Career Sidekick

26 Expert-Backed Problem Solving Examples – Interview Answers

Published: February 13, 2023

Interview Questions and Answers

Actionable advice from real experts:

picture of Biron Clark

Biron Clark

Former Recruiter

answer in problem solving

Contributor

Dr. Kyle Elliott

Career Coach

answer in problem solving

Hayley Jukes

Editor-in-Chief

Biron Clark

Biron Clark , Former Recruiter

Kyle Elliott , Career Coach

Image of Hayley Jukes

Hayley Jukes , Editor

As a recruiter , I know employers like to hire people who can solve problems and work well under pressure.

 A job rarely goes 100% according to plan, so hiring managers are more likely to hire you if you seem like you can handle unexpected challenges while staying calm and logical.

But how do they measure this?

Hiring managers will ask you interview questions about your problem-solving skills, and they might also look for examples of problem-solving on your resume and cover letter. 

In this article, I’m going to share a list of problem-solving examples and sample interview answers to questions like, “Give an example of a time you used logic to solve a problem?” and “Describe a time when you had to solve a problem without managerial input. How did you handle it, and what was the result?”

  • Problem-solving involves identifying, prioritizing, analyzing, and solving problems using a variety of skills like critical thinking, creativity, decision making, and communication.
  • Describe the Situation, Task, Action, and Result ( STAR method ) when discussing your problem-solving experiences.
  • Tailor your interview answer with the specific skills and qualifications outlined in the job description.
  • Provide numerical data or metrics to demonstrate the tangible impact of your problem-solving efforts.

What are Problem Solving Skills? 

Problem-solving is the ability to identify a problem, prioritize based on gravity and urgency, analyze the root cause, gather relevant information, develop and evaluate viable solutions, decide on the most effective and logical solution, and plan and execute implementation. 

Problem-solving encompasses other skills that can be showcased in an interview response and your resume. Problem-solving skills examples include:

  • Critical thinking
  • Analytical skills
  • Decision making
  • Research skills
  • Technical skills
  • Communication skills
  • Adaptability and flexibility

Why is Problem Solving Important in the Workplace?

Problem-solving is essential in the workplace because it directly impacts productivity and efficiency. Whenever you encounter a problem, tackling it head-on prevents minor issues from escalating into bigger ones that could disrupt the entire workflow. 

Beyond maintaining smooth operations, your ability to solve problems fosters innovation. It encourages you to think creatively, finding better ways to achieve goals, which keeps the business competitive and pushes the boundaries of what you can achieve. 

Effective problem-solving also contributes to a healthier work environment; it reduces stress by providing clear strategies for overcoming obstacles and builds confidence within teams. 

Examples of Problem-Solving in the Workplace

  • Correcting a mistake at work, whether it was made by you or someone else
  • Overcoming a delay at work through problem solving and communication
  • Resolving an issue with a difficult or upset customer
  • Overcoming issues related to a limited budget, and still delivering good work through the use of creative problem solving
  • Overcoming a scheduling/staffing shortage in the department to still deliver excellent work
  • Troubleshooting and resolving technical issues
  • Handling and resolving a conflict with a coworker
  • Solving any problems related to money, customer billing, accounting and bookkeeping, etc.
  • Taking initiative when another team member overlooked or missed something important
  • Taking initiative to meet with your superior to discuss a problem before it became potentially worse
  • Solving a safety issue at work or reporting the issue to those who could solve it
  • Using problem solving abilities to reduce/eliminate a company expense
  • Finding a way to make the company more profitable through new service or product offerings, new pricing ideas, promotion and sale ideas, etc.
  • Changing how a process, team, or task is organized to make it more efficient
  • Using creative thinking to come up with a solution that the company hasn’t used before
  • Performing research to collect data and information to find a new solution to a problem
  • Boosting a company or team’s performance by improving some aspect of communication among employees
  • Finding a new piece of data that can guide a company’s decisions or strategy better in a certain area

Problem-Solving Examples for Recent Grads/Entry-Level Job Seekers

  • Coordinating work between team members in a class project
  • Reassigning a missing team member’s work to other group members in a class project
  • Adjusting your workflow on a project to accommodate a tight deadline
  • Speaking to your professor to get help when you were struggling or unsure about a project
  • Asking classmates, peers, or professors for help in an area of struggle
  • Talking to your academic advisor to brainstorm solutions to a problem you were facing
  • Researching solutions to an academic problem online, via Google or other methods
  • Using problem solving and creative thinking to obtain an internship or other work opportunity during school after struggling at first

How To Answer “Tell Us About a Problem You Solved”

When you answer interview questions about problem-solving scenarios, or if you decide to demonstrate your problem-solving skills in a cover letter (which is a good idea any time the job description mentions problem-solving as a necessary skill), I recommend using the STAR method.

STAR stands for:

It’s a simple way of walking the listener or reader through the story in a way that will make sense to them. 

Start by briefly describing the general situation and the task at hand. After this, describe the course of action you chose and why. Ideally, show that you evaluated all the information you could given the time you had, and made a decision based on logic and fact. Finally, describe the positive result you achieved.

Note: Our sample answers below are structured following the STAR formula. Be sure to check them out!

EXPERT ADVICE

answer in problem solving

Dr. Kyle Elliott , MPA, CHES Tech & Interview Career Coach caffeinatedkyle.com

How can I communicate complex problem-solving experiences clearly and succinctly?

Before answering any interview question, it’s important to understand why the interviewer is asking the question in the first place.

When it comes to questions about your complex problem-solving experiences, for example, the interviewer likely wants to know about your leadership acumen, collaboration abilities, and communication skills, not the problem itself.

Therefore, your answer should be focused on highlighting how you excelled in each of these areas, not diving into the weeds of the problem itself, which is a common mistake less-experienced interviewees often make.

Tailoring Your Answer Based on the Skills Mentioned in the Job Description

As a recruiter, one of the top tips I can give you when responding to the prompt “Tell us about a problem you solved,” is to tailor your answer to the specific skills and qualifications outlined in the job description. 

Once you’ve pinpointed the skills and key competencies the employer is seeking, craft your response to highlight experiences where you successfully utilized or developed those particular abilities. 

For instance, if the job requires strong leadership skills, focus on a problem-solving scenario where you took charge and effectively guided a team toward resolution. 

By aligning your answer with the desired skills outlined in the job description, you demonstrate your suitability for the role and show the employer that you understand their needs.

Amanda Augustine expands on this by saying:

“Showcase the specific skills you used to solve the problem. Did it require critical thinking, analytical abilities, or strong collaboration? Highlight the relevant skills the employer is seeking.”  

Interview Answers to “Tell Me About a Time You Solved a Problem”

Now, let’s look at some sample interview answers to, “Give me an example of a time you used logic to solve a problem,” or “Tell me about a time you solved a problem,” since you’re likely to hear different versions of this interview question in all sorts of industries.

The example interview responses are structured using the STAR method and are categorized into the top 5 key problem-solving skills recruiters look for in a candidate.

1. Analytical Thinking

answer in problem solving

Situation: In my previous role as a data analyst , our team encountered a significant drop in website traffic.

Task: I was tasked with identifying the root cause of the decrease.

Action: I conducted a thorough analysis of website metrics, including traffic sources, user demographics, and page performance. Through my analysis, I discovered a technical issue with our website’s loading speed, causing users to bounce. 

Result: By optimizing server response time, compressing images, and minimizing redirects, we saw a 20% increase in traffic within two weeks.

2. Critical Thinking

answer in problem solving

Situation: During a project deadline crunch, our team encountered a major technical issue that threatened to derail our progress.

Task: My task was to assess the situation and devise a solution quickly.

Action: I immediately convened a meeting with the team to brainstorm potential solutions. Instead of panicking, I encouraged everyone to think outside the box and consider unconventional approaches. We analyzed the problem from different angles and weighed the pros and cons of each solution.

Result: By devising a workaround solution, we were able to meet the project deadline, avoiding potential delays that could have cost the company $100,000 in penalties for missing contractual obligations.

3. Decision Making

answer in problem solving

Situation: As a project manager , I was faced with a dilemma when two key team members had conflicting opinions on the project direction.

Task: My task was to make a decisive choice that would align with the project goals and maintain team cohesion.

Action: I scheduled a meeting with both team members to understand their perspectives in detail. I listened actively, asked probing questions, and encouraged open dialogue. After carefully weighing the pros and cons of each approach, I made a decision that incorporated elements from both viewpoints.

Result: The decision I made not only resolved the immediate conflict but also led to a stronger sense of collaboration within the team. By valuing input from all team members and making a well-informed decision, we were able to achieve our project objectives efficiently.

4. Communication (Teamwork)

answer in problem solving

Situation: During a cross-functional project, miscommunication between departments was causing delays and misunderstandings.

Task: My task was to improve communication channels and foster better teamwork among team members.

Action: I initiated regular cross-departmental meetings to ensure that everyone was on the same page regarding project goals and timelines. I also implemented a centralized communication platform where team members could share updates, ask questions, and collaborate more effectively.

Result: Streamlining workflows and improving communication channels led to a 30% reduction in project completion time, saving the company $25,000 in operational costs.

5. Persistence 

Situation: During a challenging sales quarter, I encountered numerous rejections and setbacks while trying to close a major client deal.

Task: My task was to persistently pursue the client and overcome obstacles to secure the deal.

Action: I maintained regular communication with the client, addressing their concerns and demonstrating the value proposition of our product. Despite facing multiple rejections, I remained persistent and resilient, adjusting my approach based on feedback and market dynamics.

Result: After months of perseverance, I successfully closed the deal with the client. By closing the major client deal, I exceeded quarterly sales targets by 25%, resulting in a revenue increase of $250,000 for the company.

Tips to Improve Your Problem-Solving Skills

Throughout your career, being able to showcase and effectively communicate your problem-solving skills gives you more leverage in achieving better jobs and earning more money .

So to improve your problem-solving skills, I recommend always analyzing a problem and situation before acting.

 When discussing problem-solving with employers, you never want to sound like you rush or make impulsive decisions. They want to see fact-based or data-based decisions when you solve problems.

Don’t just say you’re good at solving problems. Show it with specifics. How much did you boost efficiency? Did you save the company money? Adding numbers can really make your achievements stand out.

To get better at solving problems, analyze the outcomes of past solutions you came up with. You can recognize what works and what doesn’t.

Think about how you can improve researching and analyzing a situation, how you can get better at communicating, and deciding on the right people in the organization to talk to and “pull in” to help you if needed, etc.

Finally, practice staying calm even in stressful situations. Take a few minutes to walk outside if needed. Step away from your phone and computer to clear your head. A work problem is rarely so urgent that you cannot take five minutes to think (with the possible exception of safety problems), and you’ll get better outcomes if you solve problems by acting logically instead of rushing to react in a panic.

You can use all of the ideas above to describe your problem-solving skills when asked interview questions about the topic. If you say that you do the things above, employers will be impressed when they assess your problem-solving ability.

More Interview Resources

  • 3 Answers to “How Do You Handle Stress?”
  • How to Answer “How Do You Handle Conflict?” (Interview Question)
  • Sample Answers to “Tell Me About a Time You Failed”

picture of Biron Clark

About the Author

Biron Clark is a former executive recruiter who has worked individually with hundreds of job seekers, reviewed thousands of resumes and LinkedIn profiles, and recruited for top venture-backed startups and Fortune 500 companies. He has been advising job seekers since 2012 to think differently in their job search and land high-paying, competitive positions. Follow on Twitter and LinkedIn .

Read more articles by Biron Clark

About the Contributor

Kyle Elliott , career coach and mental health advocate, transforms his side hustle into a notable practice, aiding Silicon Valley professionals in maximizing potential. Follow Kyle on LinkedIn .

Image of Hayley Jukes

About the Editor

Hayley Jukes is the Editor-in-Chief at CareerSidekick with five years of experience creating engaging articles, books, and transcripts for diverse platforms and audiences.

Continue Reading

12 Expert-Approved Responses to ‘What Makes You Unique?’ in Job Interviews

15 most common pharmacist interview questions and answers, 15 most common paralegal interview questions and answers, top 30+ funny interview questions and answers, 60 hardest interview questions and answers, 100+ best ice breaker questions to ask candidates, top 20 situational interview questions (& sample answers), 15 most common physical therapist interview questions and answers.

Better resumes & cover letters

AI Resume Builder

Resume Checker

AI Cover Letters

Resume Summary Generator

Resume Bullet Generator

Resume Skills Generator

Tailor your resume to jobs

Job Tailored Resumes

Resume Keyword Scanner

Job Keyword Finder

Organize your search

Job Tracker

Contact Tracker

Interview Tracker

Job Search Metrics

Speed up your search

Job Application Autofill

Chrome Extension

  • For Organizations
  • University Career Centers
  • Career Coaches
  • Workforce Development
  • Outplacement
  • Affiliate Program

huntr logo

Targeted Resume

Beautiful, perfectly job-tailored resumes designed to make you stand out, built 10x faster with the power of AI.

  • For Organizations University Career Centers Career Coaches Bootcamps Workforce Development Outplacement Affiliate Program
  • Sign up for free

Interview Questions

50 Interview Questions About Problem Solving (With Answers)

Problems often arise at work. Can you solve them? Here are 50 interview questions about problem solving to know about.

March 22, 2024

Working in a company, you’ll be tasked with projects to work on that solve problems. Maybe you’ll need to solve a problem to help achieve a specific goal or create a solution to a problem that helps your business’ customers. Either way, problem solving is an important skill in the workplace. This post will include 50 interview questions about problem solving with answers to help you prepare for these types of interviews.

Get more interview questions.

Sign up for Huntr to access interview questions tailored to the job you're applying for.

What is problem solving?

Problem solving is a critical soft skill that involves identifying, analyzing, and resolving issues or obstacles effectively and efficiently. It encompasses a series of steps including understanding the problem, brainstorming potential solutions, evaluating the pros and cons of each option, and implementing the most viable solution. Problem solving is not only about finding quick fixes but also about foreseeing potential challenges and addressing them proactively. It requires creativity, critical thinking, and the ability to remain calm under pressure, making it a highly valued skill in personal and professional contexts alike. Whether it's navigating interpersonal conflicts, troubleshooting a technical issue, or overcoming logistical hurdles, adept problem solvers are equipped to tackle a wide array of challenges, turning obstacles into opportunities for growth and improvement.

Blog image

Why problem solving is important in the workplace

1. enhances adaptability.

In today's fast-paced work environment, the ability to engage in effective problem solving is crucial. It allows employees to adapt quickly to changes, challenges, and unexpected situations. By developing strong problem-solving skills, individuals can analyze issues, identify solutions, and implement strategies efficiently, ensuring the smooth operation of workplace processes.

2. Promotes Innovation

Problem-solving skills are the backbone of innovation within the workplace. When employees are equipped to tackle challenges creatively, they can generate novel ideas and solutions that drive the company forward. This skill not only aids in overcoming immediate obstacles but also contributes to the long-term development and competitiveness of the organization.

3. Improves Team Collaboration

Effective problem solving often requires a collaborative effort. When team members possess strong problem-solving skills, they can work together more effectively, sharing insights, and combining their expertise to find the best solutions. This not only leads to better outcomes but also fosters a culture of cooperation and mutual respect among colleagues, enhancing overall workplace morale and productivity.

Blog image

5 Tips for Answering Problem-Solving Interview Questions

Problem-solving is a critical skill that employers look for in candidates across various industries. Demonstrating your ability to tackle challenges effectively can set you apart from other applicants. Here are five tips to help you showcase your problem-solving skills during an interview:

1. Use the STAR Method

Structure your responses using the Situation, Task, Action, and Result (STAR) method. Start by describing a specific situation or problem you encountered, outline the task you needed to accomplish, detail the actions you took to address the problem, and finish with the results of your efforts. This method helps you deliver a concise and compelling story that highlights your problem-solving prowess.

2. Highlight Your Analytical Skills

Employers want to see that you can analyze a situation and come up with a logical solution. When answering questions, talk about how you break down complex problems into manageable parts. Mention any tools or techniques you use, such as SWOT analysis, root cause analysis, or critical thinking strategies, to diagnose issues and develop solutions.

3. Show Adaptability

Problem-solving often requires flexibility and the ability to adapt to changing circumstances. Share examples of how you've had to adjust your approach when faced with new information or unforeseen challenges. This demonstrates your resilience and willingness to pivot your strategy to achieve the best outcome.

4. Emphasize Collaboration

Many problems are too complex for one person to solve alone. Talk about times when you've collaborated with others to tackle a challenge. Highlight your ability to listen to different perspectives, integrate feedback, and work as part of a team to find innovative solutions. This shows that you value diverse input and can leverage collective intelligence to overcome obstacles.

5. Reflect on Lessons Learned

Finally, don't shy away from discussing problems that didn't go as planned. Reflecting on what didn't work and what you learned from those experiences can be just as powerful as showcasing your successes. It demonstrates self-awareness, accountability, and a commitment to continuous improvement. Be sure to end on a positive note by explaining how these lessons have enhanced your problem-solving abilities moving forward.

By following these tips and preparing thoughtful, specific examples, you'll be able to convincingly demonstrate your problem-solving skills in your next interview. Remember, it's not just about the problems you've solved, but how you've solved them that will impress potential employers.

Blog image

1. Can you describe a situation where you had to solve a difficult problem at work?

In a previous role, our company faced a significant customer dissatisfaction issue due to a software glitch that caused frequent service disruptions. As the project manager, I had to lead a cross-functional team to identify the root cause, develop a solution, and implement it swiftly to restore customer satisfaction. It involved collaborating with the IT team, customer support, and product development to conduct a thorough analysis, prioritize fixes, and communicate transparently with customers to manage expectations and regain trust.

2. How do you approach complex problems in your professional life?

I approach complex problems by first breaking them down into smaller manageable components to gain clarity on the issue. Then, I gather relevant data, analyze trends, and identify patterns to understand the underlying factors contributing to the problem. Collaborating with colleagues from diverse backgrounds helps gain different perspectives and insights. I also prioritize actions based on urgency and impact, continuously evaluate progress, and adapt strategies as needed to achieve effective solutions.

3. What is your process for identifying the root cause of a problem?

My process for identifying the root cause involves asking probing questions, conducting thorough research, gathering data and feedback from stakeholders, analyzing trends and patterns, and using problem-solving techniques such as root cause analysis (RCA) or the "5 Whys" method. I focus on understanding the systemic issues rather than just addressing symptoms to ensure long-term solutions.

4. Can you give an example of a creative solution you've implemented to address a challenging issue?

In a project where budget constraints were hindering progress, I proposed implementing a lean approach by prioritizing essential features, streamlining workflows, and optimizing resource allocation. This creative solution allowed us to deliver key functionalities within budget and timeline constraints while maintaining quality and stakeholder satisfaction.

5. How do you prioritize problems that need to be solved?

I prioritize problems based on their impact on organizational goals, customer experience, urgency, and resource availability. I categorize issues into immediate, short-term, and long-term priorities, considering the potential risks and benefits of solving each problem. Regularly reassessing priorities ensures alignment with evolving business needs and strategic objectives.

6. What tools or methods do you typically use for problem-solving?

I utilize various problem-solving tools and methods depending on the complexity of the issue, including brainstorming sessions, fishbone diagrams, SWOT analysis, process mapping, data analytics, and project management software. Collaborative platforms and communication tools also facilitate effective teamwork and information sharing during problem-solving processes.

7. How do you involve others in the problem-solving process?

I involve others in the problem-solving process by fostering a culture of collaboration, open communication, and shared responsibility. This includes seeking input and perspectives from team members, encouraging diverse ideas and approaches, delegating tasks based on strengths, providing support and resources, facilitating brainstorming sessions or workshops, and recognizing contributions to promote a sense of ownership and accountability in problem-solving efforts.

8. Can you share an experience where you had to solve a problem under a tight deadline?

In a previous project, we faced a critical technical issue just days before a major product launch. The issue threatened the functionality of key features, risking customer satisfaction and revenue. To solve it, I organized a rapid-response team, conducted round-the-clock troubleshooting, prioritized tasks based on impact, and delegated responsibilities efficiently. Despite the tight deadline, we collaborated effectively, leveraged expertise from various departments, and implemented a temporary workaround to ensure a successful product launch. Post-launch, we continued refining the solution for a permanent fix.

9. How do you ensure that a solution is both effective and efficient?

I ensure that a solution is both effective and efficient by aligning it with predefined objectives and success criteria, conducting thorough analysis and testing, seeking feedback from stakeholders, and evaluating its impact on key performance indicators (KPIs) such as cost, time, quality, and customer satisfaction. Continuous monitoring and iteration allow me to optimize the solution's efficiency while maintaining its effectiveness over time.

10. What is the biggest problem you have solved in your career?

One of the biggest problems I solved in my career was optimizing a supply chain process for a multinational company. The challenge involved reducing lead times, improving inventory management, and enhancing supplier relationships to meet growing customer demands and reduce costs. By implementing data-driven strategies, process improvements, and cross-functional collaboration, we achieved significant improvements in efficiency, cost savings, and customer service levels, contributing to the company's overall success.

11. How do you deal with uncertainty when solving problems?

When faced with uncertainty, I adopt a structured approach by gathering relevant information, conducting scenario analysis, identifying potential risks and opportunities, and developing contingency plans. I also leverage past experiences, seek input from subject matter experts, and remain adaptable and open to alternative solutions. Clear communication, ongoing evaluation, and agile decision-making help navigate uncertainty effectively and mitigate potential impacts.

12. Can you describe a time when you had to solve a problem without all the necessary information?

In a project where critical data was unavailable due to technical issues, I had to make decisions and implement solutions based on limited information. To address this challenge, I leveraged available data, conducted qualitative analysis, consulted with experts, and communicated transparently with stakeholders about the uncertainties and risks involved. Despite the constraints, we managed to implement a temporary solution and later refined it based on additional insights and data.

13. How do you measure the success of a solution?

I measure the success of a solution by defining clear metrics and KPIs aligned with the problem's objectives and desired outcomes. This includes assessing factors such as cost savings, time efficiency, quality improvement, customer satisfaction, revenue generation, and impact on organizational goals. Regular performance monitoring, data analysis, feedback collection, and stakeholder evaluations help track progress, identify areas for improvement, and ensure that the solution delivers measurable benefits.

14. What do you do if your initial solution to a problem fails?

If my initial solution to a problem fails, I conduct a thorough analysis to understand the root cause of the failure, gather feedback from stakeholders, and reassess the problem and its context. I then iterate and refine the solution, incorporating lessons learned, alternative approaches, and additional insights to develop a more effective and robust solution. Continuous improvement, flexibility, and resilience are key elements in overcoming setbacks and achieving successful outcomes.

15. How do you adapt your problem-solving strategies in a rapidly changing environment?

In a rapidly changing environment, I adapt my problem-solving strategies by staying informed about industry trends, technological advancements, and market dynamics. I prioritize agility, collaboration, and innovation, regularly reassessing priorities, adjusting strategies, and leveraging emerging tools and methodologies to address evolving challenges effectively. Flexibility, quick decision-making, and a proactive approach to change enable me to navigate uncertainties and drive successful problem resolution.

16. Can you give an example of a time when you had to use data analysis for problem-solving?

In a marketing campaign analysis project, we noticed a significant drop in conversion rates for a particular segment of our target audience. To identify the underlying issue, I conducted a detailed data analysis using customer demographics, behavior patterns, and campaign performance metrics. By correlating data points and conducting A/B testing, we discovered that the drop in conversion was due to a mismatch between the messaging in the campaign and the preferences of that specific audience segment. This data-driven insight allowed us to adjust our marketing strategy effectively and improve conversion rates.

17. How do you differentiate between symptoms and root causes of a problem?

To differentiate between symptoms and root causes of a problem, I employ techniques such as the "5 Whys" method, root cause analysis, and process mapping. By asking probing questions and digging deeper into the factors contributing to the problem, I can identify underlying causes rather than just addressing surface-level symptoms. This approach ensures that solutions target the root of the problem, leading to more effective and sustainable outcomes.

18. Can you discuss a time when you had to solve a problem collaboratively with a team?

In a product development project, we encountered challenges related to compatibility issues between different software components, leading to performance issues and customer complaints. To address this, I facilitated collaborative problem-solving sessions with cross-functional teams, including developers, QA testers, and product managers. Through open communication, brainstorming, and sharing of expertise, we identified the root causes, developed a coordinated action plan, and implemented solutions that resolved the compatibility issues, improving product performance and customer satisfaction.

19. How do you handle feedback on your problem-solving efforts?

I value feedback as an opportunity for growth and improvement. When receiving feedback on my problem-solving efforts, I actively listen, seek to understand perspectives, and appreciate constructive criticism. I use feedback to reflect on my approach, identify areas for enhancement, and incorporate suggestions into future problem-solving endeavors. Embracing feedback fosters continuous learning, enhances problem-solving skills, and contributes to achieving better outcomes.

20. What role does critical thinking play in your problem-solving process?

Critical thinking is integral to my problem-solving process as it enables me to analyze situations objectively, evaluate information, identify patterns, and make informed decisions. By applying logical reasoning, sound judgment, and evidence-based analysis, I can discern between relevant and irrelevant data, assess risks, consider alternative solutions, and anticipate potential outcomes. Critical thinking enhances problem-solving effectiveness by promoting thoroughness, accuracy, and strategic decision-making.

21. How do you stay motivated when faced with a complex problem?

When faced with a complex problem, I stay motivated by breaking the problem down into manageable tasks, setting realistic goals, and focusing on incremental progress. I maintain a positive mindset, leverage my problem-solving skills and past successes, seek support and collaboration from colleagues or mentors, and celebrate small victories along the way. Staying organized, maintaining a clear vision of the desired outcome, and reminding myself of the impact of solving the problem keep me motivated and determined to overcome challenges.

22. Can you describe a situation where you had to change your approach to solve a problem effectively?

In a project involving customer feedback analysis, my initial approach focused solely on quantitative data analysis to identify trends and patterns. However, I realized that the qualitative aspect of customer feedback, such as sentiments and specific comments, provided valuable insights that were missed in the quantitative analysis alone. To address this, I changed my approach by integrating qualitative data analysis techniques, such as sentiment analysis and thematic coding, into the process. This holistic approach led to a more comprehensive understanding of customer perceptions and facilitated more targeted problem-solving strategies.

23. How do you balance the need for quick solutions with the need for thorough problem-solving?

Balancing the need for quick solutions with thorough problem-solving involves prioritizing tasks based on urgency and impact. For urgent issues requiring immediate resolution, I focus on quick, interim solutions to address critical aspects and mitigate immediate risks. Simultaneously, I allocate time and resources for in-depth analysis, root cause identification, and long-term solutions to prevent recurrence and optimize outcomes. Effective time management, clear prioritization, and strategic decision-making enable me to strike a balance between speed and thoroughness in problem-solving.

24. Can you discuss a time when you had to solve a problem with limited resources?

In a project where budget constraints limited our ability to hire additional staff, I faced the challenge of increasing operational efficiency without increasing costs. To address this, I conducted a thorough analysis of existing processes, identified inefficiencies, and proposed automation solutions using available software tools. By leveraging technology and optimizing workflows, we were able to streamline operations, reduce manual tasks, and improve productivity without exceeding the budget.

25. What strategies do you use to prevent problems from occurring in the first place?

Proactive problem prevention involves risk assessment, continuous monitoring, and implementing preventive measures. I regularly conduct risk assessments to identify potential issues, develop contingency plans, and implement controls to mitigate risks. Additionally, I emphasize continuous improvement, encourage open communication within teams, and promote a culture of accountability and learning from past experiences to prevent recurring problems.

26. How do you communicate complex problems (and solutions) to stakeholders who may not be familiar with the details?

When communicating complex problems and solutions to stakeholders, I use a structured approach that involves breaking down technical information into easily understandable concepts. I focus on highlighting the impact, benefits, and relevance of the problem and proposed solutions to the stakeholders' interests and objectives. Utilizing visual aids, such as charts, graphs, and diagrams, helps clarify complex information, facilitate discussions, and ensure stakeholders grasp key points effectively.

27. Can you share an example of a time when you solved a problem that significantly benefited your organization?

In a cost optimization initiative, I identified inefficiencies in supply chain management that were leading to increased procurement costs and delayed deliveries. By analyzing data, negotiating contracts with vendors, and implementing process improvements, we streamlined supply chain operations, reduced procurement costs by 20%, and improved delivery timelines. This solution not only saved the organization substantial resources but also enhanced operational efficiency and customer satisfaction.

28. How do you incorporate feedback into your problem-solving process?

Incorporating feedback into the problem-solving process is essential for continuous improvement. I actively seek feedback from stakeholders, team members, and subject matter experts throughout the problem-solving journey. I analyze feedback to identify areas for enhancement, consider alternative perspectives and solutions, and iteratively refine strategies based on input received. Regular feedback loops ensure that solutions are well-informed, aligned with stakeholder expectations, and optimized for effectiveness.

29. What is the most unconventional problem-solving method you’ve successfully used?

In a project where traditional problem-solving methods were insufficient, I applied design thinking principles to generate innovative solutions. By conducting empathy interviews, brainstorming sessions, and prototyping ideas, we identified creative solutions that addressed user needs more effectively. This unconventional approach led to breakthrough solutions that significantly improved user experience and product performance.

30. How do you ensure that your solution aligns with the overall goals of your organization?

Aligning solutions with organizational goals involves understanding strategic objectives, engaging stakeholders, and evaluating solutions based on their impact on key performance indicators. I ensure alignment by conducting thorough analyses, seeking input from relevant stakeholders, and assessing how proposed solutions contribute to organizational priorities, such as cost reduction, revenue growth, customer satisfaction, or market expansion. Regular alignment checks and feedback loops help validate that solutions remain aligned with overarching goals.

31. Can you describe a problem you solved that had a positive impact on your team’s performance?

In a team performance improvement project, I identified communication barriers and lack of collaboration as key challenges impacting productivity. I implemented team-building activities, fostered a culture of open communication and collaboration, and facilitated regular feedback sessions. These interventions resulted in improved teamwork, enhanced morale, and increased productivity, leading to measurable performance improvements and positive outcomes for the team and organization.

32. How do you decide when to tackle a problem on your own or to seek help?

I assess the complexity, urgency, and impact of the problem to determine whether I can address it independently or if seeking help is necessary. If the problem requires specialized expertise, collaboration, or resources beyond my capabilities, I proactively seek assistance to ensure efficient and effective problem-solving. I prioritize transparency and open communication to foster a collaborative problem-solving approach.

33. How do you keep track of multiple problems that need to be solved at the same time?

To manage multiple problems simultaneously, I utilize organization tools such as task lists, project management software, and prioritization techniques. I categorize problems based on urgency, importance, and dependencies, establish clear timelines and milestones for each, and regularly review progress to adjust priorities as needed. Effective delegation, communication, and collaboration with team members also contribute to successful management of multiple problem-solving efforts.

34. Can you explain how you evaluate potential solutions to a problem?

When evaluating potential solutions, I consider factors such as feasibility, effectiveness, cost, impact, and alignment with objectives. I conduct thorough analyses, gather relevant data and insights, assess risks and benefits, and compare alternative solutions based on predefined criteria. I prioritize solutions that are practical, sustainable, and aligned with strategic goals, and I seek input from stakeholders and subject matter experts to ensure comprehensive evaluation and informed decision-making.

35. How do you deal with resistance when implementing a solution?

Addressing resistance during solution implementation requires effective communication, stakeholder engagement, and change management strategies. I proactively communicate the rationale behind the solution, its benefits, and the expected outcomes to gain buy-in and mitigate resistance. I listen to concerns, address objections, and involve stakeholders in the decision-making process to foster ownership and collaboration. I also provide support, training, and feedback mechanisms to facilitate smooth adoption and implementation of the solution.

36. Can you describe a time when you had to solve a problem outside of your area of expertise?

In a cross-functional project, I encountered a technical issue that required expertise beyond my domain. Recognizing the challenge, I collaborated with experts from relevant departments, sought their insights, and leveraged their knowledge to understand the problem thoroughly. By facilitating interdisciplinary discussions, conducting research, and learning from experts, I gained the necessary understanding to contribute effectively to problem-solving and drive successful outcomes for the project.

37. How do you ensure that your problem-solving efforts are inclusive and consider diverse perspectives?

Inclusive problem-solving involves valuing diverse perspectives, fostering open dialogue, and creating an environment where everyone's input is respected and considered. I actively seek input from team members with varying backgrounds, experiences, and expertise, encourage brainstorming sessions, and facilitate discussions that promote diverse viewpoints. I promote inclusivity by practicing active listening, empathy, and respect for different opinions, ensuring that solutions are comprehensive, innovative, and reflective of diverse perspectives.

38. Can you share a situation where you had to use emotional intelligence in problem-solving?

During a project with tight deadlines and high stakes, I encountered interpersonal conflicts among team members that were impacting productivity and morale. Recognizing the importance of emotional intelligence, I employed active listening, empathy, and conflict resolution skills to understand each team member's perspectives, address underlying issues, and foster a collaborative and supportive environment. By acknowledging emotions, managing conflicts constructively, and promoting effective communication, I successfully mitigated tensions, restored team cohesion, and achieved project objectives.

39. How do you manage stress and pressure when solving critical problems?

Managing stress and pressure during critical problem-solving involves prioritization, time management, self-care, and resilience-building strategies. I prioritize tasks based on urgency and importance, break down complex problems into manageable steps, and set realistic expectations and timelines. I practice stress-reduction techniques such as mindfulness, exercise, and time management, seek support from colleagues and mentors, and maintain a positive mindset to navigate challenges effectively and maintain focus on problem-solving objectives.

40. Can you explain a time when you had to prioritize long-term solutions over quick fixes?

In a previous project, we encountered recurring quality issues that required immediate attention. While quick fixes could address the immediate symptoms, I advocated for a comprehensive root cause analysis to identify underlying systemic issues. By prioritizing long-term solutions, we implemented process improvements, updated quality control measures, and provided training to team members. This approach not only resolved the immediate issues but also prevented future occurrences, leading to sustainable improvements and long-term success.

41. How do you adapt your problem-solving approach based on the audience or stakeholders involved?

Adapting problem-solving approaches involves understanding stakeholders' preferences, priorities, and communication styles. For technical audiences, I focus on data-driven analyses, detailed reports, and solution feasibility. When engaging non-technical stakeholders, I emphasize clear explanations, visual aids, and real-world examples to ensure understanding and alignment. I tailor communication channels, formats, and frequency to suit stakeholders' preferences, fostering collaboration, buy-in, and successful problem resolution.

42. Can you discuss a time when you had to use negotiation skills in problem-solving?

During a project negotiation, conflicting interests arose between departments regarding resource allocation and project timelines. To resolve the impasse, I facilitated negotiations by identifying common goals, exploring compromise options, and advocating for win-win solutions. Using active listening, empathy, and persuasive communication, I bridged differences, built consensus, and reached agreements that satisfied all parties' needs. This collaborative approach not only resolved conflicts but also strengthened relationships and promoted cooperation.

43. How do you incorporate new technologies or methodologies into your problem-solving toolkit?

Incorporating new technologies and methodologies into problem-solving requires continuous learning, experimentation, and adaptation. I stay updated on industry trends, attend training sessions, and seek opportunities to apply emerging technologies such as data analytics, automation, and AI in problem-solving. I collaborate with experts, conduct pilot projects, and evaluate results to assess the effectiveness and feasibility of new approaches. By embracing innovation, I enhance problem-solving capabilities, drive efficiencies, and deliver value to organizations.

44. Can you describe a time when your problem-solving led to innovation within your organization?

In a project focused on streamlining operations, I identified inefficiencies in existing processes and proposed innovative solutions leveraging digital tools and automation. By collaborating with cross-functional teams, conducting workflow analyses, and piloting new technologies, we implemented streamlined workflows, reduced manual tasks, and improved efficiency. This innovation not only optimized operations but also enhanced employee productivity, customer satisfaction, and organizational competitiveness.

45. How do you ensure that your solutions are sustainable and environmentally friendly?

Ensuring sustainable and environmentally friendly solutions involves considering ecological impacts, resource conservation, and long-term viability. I prioritize sustainable practices such as waste reduction, energy efficiency, and eco-friendly materials in solution design. I collaborate with sustainability experts, conduct lifecycle assessments, and integrate environmental considerations into decision-making processes. By promoting green initiatives, I contribute to environmental stewardship, corporate social responsibility, and positive societal impact through problem-solving efforts.

46. Can you share an example of a cross-functional problem you solved?

In a previous role, we faced a cross-functional challenge related to customer retention. The marketing team identified a decline in customer engagement, while the sales team noticed an increase in churn rates. Collaborating with both teams, I led a comprehensive analysis to identify root causes. We discovered that misaligned messaging between marketing campaigns and sales interactions was confusing customers. By implementing a unified communication strategy, aligning marketing messages with sales processes, and enhancing customer touchpoints, we improved customer satisfaction, reduced churn rates, and increased revenue.

47. How do you mentor others in developing their problem-solving skills?

Mentoring others in problem-solving involves providing guidance, encouragement, and opportunities for skill development. I mentor by sharing my problem-solving experiences, providing frameworks, and encouraging critical thinking. I offer constructive feedback, challenge assumptions, and encourage diverse perspectives to foster creativity and innovation. I also create learning opportunities such as workshops, case studies, and collaborative projects to help mentees apply problem-solving techniques in real-world scenarios and enhance their skills over time.

48. Can you describe a situation where you had to overcome significant obstacles to solve a problem?

In a complex project, we encountered unforeseen technical challenges that threatened project timelines and deliverables. Despite initial setbacks and resource constraints, I led the team in brainstorming sessions, leveraging expertise from various departments, and exploring alternative solutions. We overcame obstacles by fostering collaboration, adopting agile methodologies, and prioritizing problem-solving efforts. Through perseverance, resilience, and adaptability, we successfully resolved technical issues, met project milestones, and delivered high-quality outcomes.

49. How do you balance logic and intuition in your problem-solving process?

Balancing logic and intuition in problem-solving involves leveraging analytical thinking and creative insights. I start by gathering data, analyzing facts, and applying logical frameworks to understand the problem's scope and complexity. Then, I tap into intuition by brainstorming ideas, exploring unconventional approaches, and considering potential outcomes from different perspectives. I combine rational decision-making with gut instincts, test hypotheses, and validate solutions to ensure effectiveness and innovation in problem-solving.

50. Can you discuss a time when you had to revisit and revise a solution based on new information or outcomes?

In a strategic initiative, we implemented a new process to streamline operations and improve efficiency. However, after implementation, we noticed unexpected challenges and suboptimal results. Recognizing the need for continuous improvement, I initiated a review process, gathered feedback from stakeholders, and conducted performance evaluations. Based on new insights and outcomes, we revisited the initial solution, identified areas for enhancement, and revised the process to address gaps and achieve desired outcomes effectively. This iterative approach ensured that our solutions remained adaptive, responsive, and aligned with evolving needs and objectives.

Get More Interviews, Faster

Huntr streamlines your job search. Instantly craft tailored resumes and cover letters, fill out application forms with a single click, effortlessly keep your job hunt organized, and much more...

or learn more

Next-Generation Job Tailored Resumes

Huntr provides the most advanced job <> resume matching system in the world. Helping you match not only keywords, but responsibilities and qualifications from a job, into your resume.

Job Keyword Extractor + Resume AI Integration

Huntr extracts keywords from job descriptions and helps you integrate them into your resume using the power of AI.

Application Autofill

Save hours of mindless form filling. Use our chrome extension to fill application forms with a single click.

Move beyond basic, bare-bones job trackers. Elevate your search with Huntr's all-in-one, feature-rich management platform.

Perfectly tailored cover letters, in seconds! Our cover letter generator blends your unique background with the job's specific requirements, resulting in unique, standout cover letters.

Huntr checks your resume for spelling, length, impactful use of metrics, repetition and more, ensuring your resume gets noticed by employers.

Gorgeous Resume Templates

Stand out with one of 7 designer-grade templates. Whether you're a creative spirit or a corporate professional, our range of templates caters to every career aspiration.

Personal Job Search CRM

The ultimate companion for managing your professional job-search contacts and organizing your job search outreach.

answer in problem solving

5 Effective Problem-Solving Strategies

answer in problem solving

Got a problem you’re trying to solve? Strategies like trial and error, gut instincts, and “working backward” can help. We look at some examples and how to use them.

We all face problems daily. Some are simple, like deciding what to eat for dinner. Others are more complex, like resolving a conflict with a loved one or figuring out how to overcome barriers to your goals.

No matter what problem you’re facing, these five problem-solving strategies can help you develop an effective solution.

An infographic showing five effective problem-solving strategies

What are problem-solving strategies?

To effectively solve a problem, you need a problem-solving strategy .

If you’ve had to make a hard decision before then you know that simply ruminating on the problem isn’t likely to get you anywhere. You need an effective strategy — or a plan of action — to find a solution.

In general, effective problem-solving strategies include the following steps:

  • Define the problem.
  • Come up with alternative solutions.
  • Decide on a solution.
  • Implement the solution.

Problem-solving strategies don’t guarantee a solution, but they do help guide you through the process of finding a resolution.

Using problem-solving strategies also has other benefits . For example, having a strategy you can turn to can help you overcome anxiety and distress when you’re first faced with a problem or difficult decision.

The key is to find a problem-solving strategy that works for your specific situation, as well as your personality. One strategy may work well for one type of problem but not another. In addition, some people may prefer certain strategies over others; for example, creative people may prefer to depend on their insights than use algorithms.

It’s important to be equipped with several problem-solving strategies so you use the one that’s most effective for your current situation.

1. Trial and error

One of the most common problem-solving strategies is trial and error. In other words, you try different solutions until you find one that works.

For example, say the problem is that your Wi-Fi isn’t working. You might try different things until it starts working again, like restarting your modem or your devices until you find or resolve the problem. When one solution isn’t successful, you try another until you find what works.

Trial and error can also work for interpersonal problems . For example, if your child always stays up past their bedtime, you might try different solutions — a visual clock to remind them of the time, a reward system, or gentle punishments — to find a solution that works.

2. Heuristics

Sometimes, it’s more effective to solve a problem based on a formula than to try different solutions blindly.

Heuristics are problem-solving strategies or frameworks people use to quickly find an approximate solution. It may not be the optimal solution, but it’s faster than finding the perfect resolution, and it’s “good enough.”

Algorithms or equations are examples of heuristics.

An algorithm is a step-by-step problem-solving strategy based on a formula guaranteed to give you positive results. For example, you might use an algorithm to determine how much food is needed to feed people at a large party.

However, many life problems have no formulaic solution; for example, you may not be able to come up with an algorithm to solve the problem of making amends with your spouse after a fight.

3. Gut instincts (insight problem-solving)

While algorithm-based problem-solving is formulaic, insight problem-solving is the opposite.

When we use insight as a problem-solving strategy we depend on our “gut instincts” or what we know and feel about a situation to come up with a solution. People might describe insight-based solutions to problems as an “aha moment.”

For example, you might face the problem of whether or not to stay in a relationship. The solution to this problem may come as a sudden insight that you need to leave. In insight problem-solving, the cognitive processes that help you solve a problem happen outside your conscious awareness.

4. Working backward

Working backward is a problem-solving approach often taught to help students solve problems in mathematics. However, it’s useful for real-world problems as well.

Working backward is when you start with the solution and “work backward” to figure out how you got to the solution. For example, if you know you need to be at a party by 8 p.m., you might work backward to problem-solve when you must leave the house, when you need to start getting ready, and so on.

5. Means-end analysis

Means-end analysis is a problem-solving strategy that, to put it simply, helps you get from “point A” to “point B” by examining and coming up with solutions to obstacles.

When using means-end analysis you define the current state or situation (where you are now) and the intended goal. Then, you come up with solutions to get from where you are now to where you need to be.

For example, a student might be faced with the problem of how to successfully get through finals season . They haven’t started studying, but their end goal is to pass all of their finals. Using means-end analysis, the student can examine the obstacles that stand between their current state and their end goal (passing their finals).

They could see, for example, that one obstacle is that they get distracted from studying by their friends. They could devise a solution to this obstacle by putting their phone on “do not disturb” mode while studying.

Let’s recap

Whether they’re simple or complex, we’re faced with problems every day. To successfully solve these problems we need an effective strategy. There are many different problem-solving strategies to choose from.

Although problem-solving strategies don’t guarantee a solution, they can help you feel less anxious about problems and make it more likely that you come up with an answer.

8 sources collapsed

  • Chu Y, et al. (2011). Human performance on insight problem-solving: A review. https://docs.lib.purdue.edu/cgi/viewcontent.cgi?article=1094&context=jps
  • Dumper K, et al. (n.d.) Chapter 7.3: Problem-solving in introductory psychology. https://opentext.wsu.edu/psych105/chapter/7-4-problem-solving/
  • Foulds LR. (2017). The heuristic problem-solving approach. https://www.tandfonline.com/doi/abs/10.1057/jors.1983.205
  • Gick ML. (1986). Problem-solving strategies. https://www.tandfonline.com/doi/abs/10.1080/00461520.1986.9653026
  • Montgomery ME. (2015). Problem solving using means-end analysis. https://sites.psu.edu/psych256sp15/2015/04/19/problem-solving-using-means-end-analysis/
  • Posamentier A, et al. (2015). Problem-solving strategies in mathematics. Chapter 3: Working backwards. https://www.worldscientific.com/doi/10.1142/9789814651646_0003
  • Sarathy V. (2018). Real world problem-solving. https://www.frontiersin.org/articles/10.3389/fnhum.2018.00261/full
  • Woods D. (2000). An evidence-based strategy for problem solving. https://www.researchgate.net/publication/245332888_An_Evidence-Based_Strategy_for_Problem_Solving

Read this next

Making big decisions can be a difficult task. Setting deadlines and asking for support can help you confidently move ahead.

Dealing with a problem can fee a lot more manageable when you have a plan. Try these 5 steps for becoming a better problem-solver.

A lack of communication in relationships doesn't have to be a dealbreaker. Learn how to improve your communication skills at work and at home.

Displaced anger is the act of directing your anger toward someone or something other than its source. Identifying and addressing underlying causes can…

Matrescence is the transition to becoming a mother. It refers to mind-body changes that occur during pregnancy and may improve cognition as a person…

Sleep deprivation, stress, or underlying health conditions can lead to an inability to focus. Self-help techniques can help improve your concentration.

Positive thinking is an essential practice to improve your overall health and well-being. Discover how to incorporate positive thinking into your…

Dreaming about babies can hold different meanings for everyone. Although theories vary, biological and psychological factors may influence your dreams.

If you're seeking to boost your concentration, practicing mindfulness, chewing gum, and brain games are just a few techniques to try. Learn how they…

Creating a schedule and managing stress are ways to make your days go by faster. Changing your perception of time can also improve your overall…

Status.net

What is Problem Solving? (Steps, Techniques, Examples)

By Status.net Editorial Team on May 7, 2023 — 5 minutes to read

What Is Problem Solving?

Definition and importance.

Problem solving is the process of finding solutions to obstacles or challenges you encounter in your life or work. It is a crucial skill that allows you to tackle complex situations, adapt to changes, and overcome difficulties with ease. Mastering this ability will contribute to both your personal and professional growth, leading to more successful outcomes and better decision-making.

Problem-Solving Steps

The problem-solving process typically includes the following steps:

  • Identify the issue : Recognize the problem that needs to be solved.
  • Analyze the situation : Examine the issue in depth, gather all relevant information, and consider any limitations or constraints that may be present.
  • Generate potential solutions : Brainstorm a list of possible solutions to the issue, without immediately judging or evaluating them.
  • Evaluate options : Weigh the pros and cons of each potential solution, considering factors such as feasibility, effectiveness, and potential risks.
  • Select the best solution : Choose the option that best addresses the problem and aligns with your objectives.
  • Implement the solution : Put the selected solution into action and monitor the results to ensure it resolves the issue.
  • Review and learn : Reflect on the problem-solving process, identify any improvements or adjustments that can be made, and apply these learnings to future situations.

Defining the Problem

To start tackling a problem, first, identify and understand it. Analyzing the issue thoroughly helps to clarify its scope and nature. Ask questions to gather information and consider the problem from various angles. Some strategies to define the problem include:

  • Brainstorming with others
  • Asking the 5 Ws and 1 H (Who, What, When, Where, Why, and How)
  • Analyzing cause and effect
  • Creating a problem statement

Generating Solutions

Once the problem is clearly understood, brainstorm possible solutions. Think creatively and keep an open mind, as well as considering lessons from past experiences. Consider:

  • Creating a list of potential ideas to solve the problem
  • Grouping and categorizing similar solutions
  • Prioritizing potential solutions based on feasibility, cost, and resources required
  • Involving others to share diverse opinions and inputs

Evaluating and Selecting Solutions

Evaluate each potential solution, weighing its pros and cons. To facilitate decision-making, use techniques such as:

  • SWOT analysis (Strengths, Weaknesses, Opportunities, Threats)
  • Decision-making matrices
  • Pros and cons lists
  • Risk assessments

After evaluating, choose the most suitable solution based on effectiveness, cost, and time constraints.

Implementing and Monitoring the Solution

Implement the chosen solution and monitor its progress. Key actions include:

  • Communicating the solution to relevant parties
  • Setting timelines and milestones
  • Assigning tasks and responsibilities
  • Monitoring the solution and making adjustments as necessary
  • Evaluating the effectiveness of the solution after implementation

Utilize feedback from stakeholders and consider potential improvements. Remember that problem-solving is an ongoing process that can always be refined and enhanced.

Problem-Solving Techniques

During each step, you may find it helpful to utilize various problem-solving techniques, such as:

  • Brainstorming : A free-flowing, open-minded session where ideas are generated and listed without judgment, to encourage creativity and innovative thinking.
  • Root cause analysis : A method that explores the underlying causes of a problem to find the most effective solution rather than addressing superficial symptoms.
  • SWOT analysis : A tool used to evaluate the strengths, weaknesses, opportunities, and threats related to a problem or decision, providing a comprehensive view of the situation.
  • Mind mapping : A visual technique that uses diagrams to organize and connect ideas, helping to identify patterns, relationships, and possible solutions.

Brainstorming

When facing a problem, start by conducting a brainstorming session. Gather your team and encourage an open discussion where everyone contributes ideas, no matter how outlandish they may seem. This helps you:

  • Generate a diverse range of solutions
  • Encourage all team members to participate
  • Foster creative thinking

When brainstorming, remember to:

  • Reserve judgment until the session is over
  • Encourage wild ideas
  • Combine and improve upon ideas

Root Cause Analysis

For effective problem-solving, identifying the root cause of the issue at hand is crucial. Try these methods:

  • 5 Whys : Ask “why” five times to get to the underlying cause.
  • Fishbone Diagram : Create a diagram representing the problem and break it down into categories of potential causes.
  • Pareto Analysis : Determine the few most significant causes underlying the majority of problems.

SWOT Analysis

SWOT analysis helps you examine the Strengths, Weaknesses, Opportunities, and Threats related to your problem. To perform a SWOT analysis:

  • List your problem’s strengths, such as relevant resources or strong partnerships.
  • Identify its weaknesses, such as knowledge gaps or limited resources.
  • Explore opportunities, like trends or new technologies, that could help solve the problem.
  • Recognize potential threats, like competition or regulatory barriers.

SWOT analysis aids in understanding the internal and external factors affecting the problem, which can help guide your solution.

Mind Mapping

A mind map is a visual representation of your problem and potential solutions. It enables you to organize information in a structured and intuitive manner. To create a mind map:

  • Write the problem in the center of a blank page.
  • Draw branches from the central problem to related sub-problems or contributing factors.
  • Add more branches to represent potential solutions or further ideas.

Mind mapping allows you to visually see connections between ideas and promotes creativity in problem-solving.

Examples of Problem Solving in Various Contexts

In the business world, you might encounter problems related to finances, operations, or communication. Applying problem-solving skills in these situations could look like:

  • Identifying areas of improvement in your company’s financial performance and implementing cost-saving measures
  • Resolving internal conflicts among team members by listening and understanding different perspectives, then proposing and negotiating solutions
  • Streamlining a process for better productivity by removing redundancies, automating tasks, or re-allocating resources

In educational contexts, problem-solving can be seen in various aspects, such as:

  • Addressing a gap in students’ understanding by employing diverse teaching methods to cater to different learning styles
  • Developing a strategy for successful time management to balance academic responsibilities and extracurricular activities
  • Seeking resources and support to provide equal opportunities for learners with special needs or disabilities

Everyday life is full of challenges that require problem-solving skills. Some examples include:

  • Overcoming a personal obstacle, such as improving your fitness level, by establishing achievable goals, measuring progress, and adjusting your approach accordingly
  • Navigating a new environment or city by researching your surroundings, asking for directions, or using technology like GPS to guide you
  • Dealing with a sudden change, like a change in your work schedule, by assessing the situation, identifying potential impacts, and adapting your plans to accommodate the change.
  • How to Resolve Employee Conflict at Work [Steps, Tips, Examples]
  • How to Write Inspiring Core Values? 5 Steps with Examples
  • 30 Employee Feedback Examples (Positive & Negative)

40 problem-solving techniques and processes

Problem solving workshop

All teams and organizations encounter challenges. Approaching those challenges without a structured problem solving process can end up making things worse.

Proven problem solving techniques such as those outlined below can guide your group through a process of identifying problems and challenges , ideating on possible solutions , and then evaluating and implementing the most suitable .

In this post, you'll find problem-solving tools you can use to develop effective solutions. You'll also find some tips for facilitating the problem solving process and solving complex problems.

Design your next session with SessionLab

Join the 150,000+ facilitators 
using SessionLab.

Recommended Articles

A step-by-step guide to planning a workshop, 54 great online tools for workshops and meetings, how to create an unforgettable training session in 8 simple steps.

  • 18 Free Facilitation Resources We Think You’ll Love

What is problem solving?

Problem solving is a process of finding and implementing a solution to a challenge or obstacle. In most contexts, this means going through a problem solving process that begins with identifying the issue, exploring its root causes, ideating and refining possible solutions before implementing and measuring the impact of that solution.

For simple or small problems, it can be tempting to skip straight to implementing what you believe is the right solution. The danger with this approach is that without exploring the true causes of the issue, it might just occur again or your chosen solution may cause other issues.

Particularly in the world of work, good problem solving means using data to back up each step of the process, bringing in new perspectives and effectively measuring the impact of your solution.

Effective problem solving can help ensure that your team or organization is well positioned to overcome challenges, be resilient to change and create innovation. In my experience, problem solving is a combination of skillset, mindset and process, and it’s especially vital for leaders to cultivate this skill.

A group of people looking at a poster with notes on it

What is the seven step problem solving process?

A problem solving process is a step-by-step framework from going from discovering a problem all the way through to implementing a solution.

With practice, this framework can become intuitive, and innovative companies tend to have a consistent and ongoing ability to discover and tackle challenges when they come up.

You might see everything from a four step problem solving process through to seven steps. While all these processes cover roughly the same ground, I’ve found a seven step problem solving process is helpful for making all key steps legible.

We’ll outline that process here and then follow with techniques you can use to explore and work on that step of the problem solving process with a group.

The seven-step problem solving process is:

1. Problem identification 

The first stage of any problem solving process is to identify the problem(s) you need to solve. This often looks like using group discussions and activities to help a group surface and effectively articulate the challenges they’re facing and wish to resolve.

Be sure to align with your team on the exact definition and nature of the problem you’re solving. An effective process is one where everyone is pulling in the same direction – ensure clarity and alignment now to help avoid misunderstandings later.

2. Problem analysis and refinement

The process of problem analysis means ensuring that the problem you are seeking to solve is  the   right problem . Choosing the right problem to solve means you are on the right path to creating the right solution.

At this stage, you may look deeper at the problem you identified to try and discover the root cause at the level of people or process. You may also spend some time sourcing data, consulting relevant parties and creating and refining a problem statement.

Problem refinement means adjusting scope or focus of the problem you will be aiming to solve based on what comes up during your analysis. As you analyze data sources, you might discover that the root cause means you need to adjust your problem statement. Alternatively, you might find that your original problem statement is too big to be meaningful approached within your current project.

Remember that the goal of any problem refinement is to help set the stage for effective solution development and deployment. Set the right focus and get buy-in from your team here and you’ll be well positioned to move forward with confidence.

3. Solution generation

Once your group has nailed down the particulars of the problem you wish to solve, you want to encourage a free flow of ideas connecting to solving that problem. This can take the form of problem solving games that encourage creative thinking or techniquess designed to produce working prototypes of possible solutions. 

The key to ensuring the success of this stage of the problem solving process is to encourage quick, creative thinking and create an open space where all ideas are considered. The best solutions can often come from unlikely places and by using problem solving techniques that celebrate invention, you might come up with solution gold. 

answer in problem solving

4. Solution development

No solution is perfect right out of the gate. It’s important to discuss and develop the solutions your group has come up with over the course of following the previous problem solving steps in order to arrive at the best possible solution. Problem solving games used in this stage involve lots of critical thinking, measuring potential effort and impact, and looking at possible solutions analytically. 

During this stage, you will often ask your team to iterate and improve upon your front-running solutions and develop them further. Remember that problem solving strategies always benefit from a multitude of voices and opinions, and not to let ego get involved when it comes to choosing which solutions to develop and take further.

Finding the best solution is the goal of all problem solving workshops and here is the place to ensure that your solution is well thought out, sufficiently robust and fit for purpose. 

5. Decision making and planning

Nearly there! Once you’ve got a set of possible, you’ll need to make a decision on which to implement. This can be a consensus-based group decision or it might be for a leader or major stakeholder to decide. You’ll find a set of effective decision making methods below.

Once your group has reached consensus and selected a solution, there are some additional actions that also need to be decided upon. You’ll want to work on allocating ownership of the project, figure out who will do what, how the success of the solution will be measured and decide the next course of action.

Set clear accountabilities, actions, timeframes, and follow-ups for your chosen solution. Make these decisions and set clear next-steps in the problem solving workshop so that everyone is aligned and you can move forward effectively as a group. 

Ensuring that you plan for the roll-out of a solution is one of the most important problem solving steps. Without adequate planning or oversight, it can prove impossible to measure success or iterate further if the problem was not solved. 

6. Solution implementation 

This is what we were waiting for! All problem solving processes have the end goal of implementing an effective and impactful solution that your group has confidence in.

Project management and communication skills are key here – your solution may need to adjust when out in the wild or you might discover new challenges along the way. For some solutions, you might also implement a test with a small group and monitor results before rolling it out to an entire company.

You should have a clear owner for your solution who will oversee the plans you made together and help ensure they’re put into place. This person will often coordinate the implementation team and set-up processes to measure the efficacy of your solution too.

7. Solution evaluation 

So you and your team developed a great solution to a problem and have a gut feeling it’s been solved. Work done, right? Wrong. All problem solving strategies benefit from evaluation, consideration, and feedback.

You might find that the solution does not work for everyone, might create new problems, or is potentially so successful that you will want to roll it out to larger teams or as part of other initiatives. 

None of that is possible without taking the time to evaluate the success of the solution you developed in your problem solving model and adjust if necessary.

Remember that the problem solving process is often iterative and it can be common to not solve complex issues on the first try. Even when this is the case, you and your team will have generated learning that will be important for future problem solving workshops or in other parts of the organization. 

It’s also worth underlining how important record keeping is throughout the problem solving process. If a solution didn’t work, you need to have the data and records to see why that was the case. If you go back to the drawing board, notes from the previous workshop can help save time.

What does an effective problem solving process look like?

Every effective problem solving process begins with an agenda . In our experience, a well-structured problem solving workshop is one of the best methods for successfully guiding a group from exploring a problem to implementing a solution.

The format of a workshop ensures that you can get buy-in from your group, encourage free-thinking and solution exploration before making a decision on what to implement following the session.

This Design Sprint 2.0 template is an effective problem solving process from top agency AJ&Smart. It’s a great format for the entire problem solving process, with four-days of workshops designed to surface issues, explore solutions and even test a solution.

Check it for an example of how you might structure and run a problem solving process and feel free to copy and adjust it your needs!

For a shorter process you can run in a single afternoon, this remote problem solving agenda will guide you effectively in just a couple of hours.

Whatever the length of your workshop, by using SessionLab, it’s easy to go from an idea to a complete agenda . Start by dragging and dropping your core problem solving activities into place . Add timings, breaks and necessary materials before sharing your agenda with your colleagues.

The resulting agenda will be your guide to an effective and productive problem solving session that will also help you stay organized on the day!

answer in problem solving

Complete problem-solving methods

In this section, we’ll look at in-depth problem-solving methods that provide a complete end-to-end process for developing effective solutions. These will help guide your team from the discovery and definition of a problem through to delivering the right solution.

If you’re looking for an all-encompassing method or problem-solving model, these processes are a great place to start. They’ll ask your team to challenge preconceived ideas and adopt a mindset for solving problems more effectively.

Six Thinking Hats

Individual approaches to solving a problem can be very different based on what team or role an individual holds. It can be easy for existing biases or perspectives to find their way into the mix, or for internal politics to direct a conversation.

Six Thinking Hats is a classic method for identifying the problems that need to be solved and enables your team to consider them from different angles, whether that is by focusing on facts and data, creative solutions, or by considering why a particular solution might not work.

Like all problem-solving frameworks, Six Thinking Hats is effective at helping teams remove roadblocks from a conversation or discussion and come to terms with all the aspects necessary to solve complex problems.

The Six Thinking Hats   #creative thinking   #meeting facilitation   #problem solving   #issue resolution   #idea generation   #conflict resolution   The Six Thinking Hats are used by individuals and groups to separate out conflicting styles of thinking. They enable and encourage a group of people to think constructively together in exploring and implementing change, rather than using argument to fight over who is right and who is wrong.

Lightning Decision Jam

Featured courtesy of Jonathan Courtney of AJ&Smart Berlin, Lightning Decision Jam is one of those strategies that should be in every facilitation toolbox. Exploring problems and finding solutions is often creative in nature, though as with any creative process, there is the potential to lose focus and get lost.

Unstructured discussions might get you there in the end, but it’s much more effective to use a method that creates a clear process and team focus.

In Lightning Decision Jam, participants are invited to begin by writing challenges, concerns, or mistakes on post-its without discussing them before then being invited by the moderator to present them to the group.

From there, the team vote on which problems to solve and are guided through steps that will allow them to reframe those problems, create solutions and then decide what to execute on. 

By deciding the problems that need to be solved as a team before moving on, this group process is great for ensuring the whole team is aligned and can take ownership over the next stages. 

Lightning Decision Jam (LDJ)   #action   #decision making   #problem solving   #issue analysis   #innovation   #design   #remote-friendly   It doesn’t matter where you work and what your job role is, if you work with other people together as a team, you will always encounter the same challenges: Unclear goals and miscommunication that cause busy work and overtime Unstructured meetings that leave attendants tired, confused and without clear outcomes. Frustration builds up because internal challenges to productivity are not addressed Sudden changes in priorities lead to a loss of focus and momentum Muddled compromise takes the place of clear decision- making, leaving everybody to come up with their own interpretation. In short, a lack of structure leads to a waste of time and effort, projects that drag on for too long and frustrated, burnt out teams. AJ&Smart has worked with some of the most innovative, productive companies in the world. What sets their teams apart from others is not better tools, bigger talent or more beautiful offices. The secret sauce to becoming a more productive, more creative and happier team is simple: Replace all open discussion or brainstorming with a structured process that leads to more ideas, clearer decisions and better outcomes. When a good process provides guardrails and a clear path to follow, it becomes easier to come up with ideas, make decisions and solve problems. This is why AJ&Smart created Lightning Decision Jam (LDJ). It’s a simple and short, but powerful group exercise that can be run either in-person, in the same room, or remotely with distributed teams.

Problem Definition Process

While problems can be complex, the problem-solving methods you use to identify and solve those problems can often be simple in design. 

By taking the time to truly identify and define a problem before asking the group to reframe the challenge as an opportunity, this method is a great way to enable change.

Begin by identifying a focus question and exploring the ways in which it manifests before splitting into five teams who will each consider the problem using a different method: escape, reversal, exaggeration, distortion or wishful. Teams develop a problem objective and create ideas in line with their method before then feeding them back to the group.

This method is great for enabling in-depth discussions while also creating space for finding creative solutions too!

Problem Definition   #problem solving   #idea generation   #creativity   #online   #remote-friendly   A problem solving technique to define a problem, challenge or opportunity and to generate ideas.

The 5 Whys 

Sometimes, a group needs to go further with their strategies and analyze the root cause at the heart of organizational issues. An RCA or root cause analysis is the process of identifying what is at the heart of business problems or recurring challenges. 

The 5 Whys is a simple and effective method of helping a group go find the root cause of any problem or challenge and conduct analysis that will deliver results. 

By beginning with the creation of a problem statement and going through five stages to refine it, The 5 Whys provides everything you need to truly discover the cause of an issue.

The 5 Whys   #hyperisland   #innovation   This simple and powerful method is useful for getting to the core of a problem or challenge. As the title suggests, the group defines a problems, then asks the question “why” five times, often using the resulting explanation as a starting point for creative problem solving.

World Cafe is a simple but powerful facilitation technique to help bigger groups to focus their energy and attention on solving complex problems.

World Cafe enables this approach by creating a relaxed atmosphere where participants are able to self-organize and explore topics relevant and important to them which are themed around a central problem-solving purpose. Create the right atmosphere by modeling your space after a cafe and after guiding the group through the method, let them take the lead!

Making problem-solving a part of your organization’s culture in the long term can be a difficult undertaking. More approachable formats like World Cafe can be especially effective in bringing people unfamiliar with workshops into the fold. 

World Cafe   #hyperisland   #innovation   #issue analysis   World Café is a simple yet powerful method, originated by Juanita Brown, for enabling meaningful conversations driven completely by participants and the topics that are relevant and important to them. Facilitators create a cafe-style space and provide simple guidelines. Participants then self-organize and explore a set of relevant topics or questions for conversation.

Discovery & Action Dialogue (DAD)

One of the best approaches is to create a safe space for a group to share and discover practices and behaviors that can help them find their own solutions.

With DAD, you can help a group choose which problems they wish to solve and which approaches they will take to do so. It’s great at helping remove resistance to change and can help get buy-in at every level too!

This process of enabling frontline ownership is great in ensuring follow-through and is one of the methods you will want in your toolbox as a facilitator.

Discovery & Action Dialogue (DAD)   #idea generation   #liberating structures   #action   #issue analysis   #remote-friendly   DADs make it easy for a group or community to discover practices and behaviors that enable some individuals (without access to special resources and facing the same constraints) to find better solutions than their peers to common problems. These are called positive deviant (PD) behaviors and practices. DADs make it possible for people in the group, unit, or community to discover by themselves these PD practices. DADs also create favorable conditions for stimulating participants’ creativity in spaces where they can feel safe to invent new and more effective practices. Resistance to change evaporates as participants are unleashed to choose freely which practices they will adopt or try and which problems they will tackle. DADs make it possible to achieve frontline ownership of solutions.
Design Sprint 2.0

Want to see how a team can solve big problems and move forward with prototyping and testing solutions in a few days? The Design Sprint 2.0 template from Jake Knapp, author of Sprint, is a complete agenda for a with proven results.

Developing the right agenda can involve difficult but necessary planning. Ensuring all the correct steps are followed can also be stressful or time-consuming depending on your level of experience.

Use this complete 4-day workshop template if you are finding there is no obvious solution to your challenge and want to focus your team around a specific problem that might require a shortcut to launching a minimum viable product or waiting for the organization-wide implementation of a solution.

Open space technology

Open space technology- developed by Harrison Owen – creates a space where large groups are invited to take ownership of their problem solving and lead individual sessions. Open space technology is a great format when you have a great deal of expertise and insight in the room and want to allow for different takes and approaches on a particular theme or problem you need to be solved.

Start by bringing your participants together to align around a central theme and focus their efforts. Explain the ground rules to help guide the problem-solving process and then invite members to identify any issue connecting to the central theme that they are interested in and are prepared to take responsibility for.

Once participants have decided on their approach to the core theme, they write their issue on a piece of paper, announce it to the group, pick a session time and place, and post the paper on the wall. As the wall fills up with sessions, the group is then invited to join the sessions that interest them the most and which they can contribute to, then you’re ready to begin!

Everyone joins the problem-solving group they’ve signed up to, record the discussion and if appropriate, findings can then be shared with the rest of the group afterward.

Open Space Technology   #action plan   #idea generation   #problem solving   #issue analysis   #large group   #online   #remote-friendly   Open Space is a methodology for large groups to create their agenda discerning important topics for discussion, suitable for conferences, community gatherings and whole system facilitation

Techniques to identify and analyze problems

Using a problem-solving method to help a team identify and analyze a problem can be a quick and effective addition to any workshop or meeting.

While further actions are always necessary, you can generate momentum and alignment easily, and these activities are a great place to get started.

We’ve put together this list of techniques to help you and your team with problem identification, analysis, and discussion that sets the foundation for developing effective solutions.

Let’s take a look!

Fishbone Analysis

Organizational or team challenges are rarely simple, and it’s important to remember that one problem can be an indication of something that goes deeper and may require further consideration to be solved.

Fishbone Analysis helps groups to dig deeper and understand the origins of a problem. It’s a great example of a root cause analysis method that is simple for everyone on a team to get their head around. 

Participants in this activity are asked to annotate a diagram of a fish, first adding the problem or issue to be worked on at the head of a fish before then brainstorming the root causes of the problem and adding them as bones on the fish. 

Using abstractions such as a diagram of a fish can really help a team break out of their regular thinking and develop a creative approach.

Fishbone Analysis   #problem solving   ##root cause analysis   #decision making   #online facilitation   A process to help identify and understand the origins of problems, issues or observations.

Problem Tree 

Encouraging visual thinking can be an essential part of many strategies. By simply reframing and clarifying problems, a group can move towards developing a problem solving model that works for them. 

In Problem Tree, groups are asked to first brainstorm a list of problems – these can be design problems, team problems or larger business problems – and then organize them into a hierarchy. The hierarchy could be from most important to least important or abstract to practical, though the key thing with problem solving games that involve this aspect is that your group has some way of managing and sorting all the issues that are raised.

Once you have a list of problems that need to be solved and have organized them accordingly, you’re then well-positioned for the next problem solving steps.

Problem tree   #define intentions   #create   #design   #issue analysis   A problem tree is a tool to clarify the hierarchy of problems addressed by the team within a design project; it represents high level problems or related sublevel problems.

SWOT Analysis

Chances are you’ve heard of the SWOT Analysis before. This problem-solving method focuses on identifying strengths, weaknesses, opportunities, and threats is a tried and tested method for both individuals and teams.

Start by creating a desired end state or outcome and bare this in mind – any process solving model is made more effective by knowing what you are moving towards. Create a quadrant made up of the four categories of a SWOT analysis and ask participants to generate ideas based on each of those quadrants.

Once you have those ideas assembled in their quadrants, cluster them together based on their affinity with other ideas. These clusters are then used to facilitate group conversations and move things forward. 

SWOT analysis   #gamestorming   #problem solving   #action   #meeting facilitation   The SWOT Analysis is a long-standing technique of looking at what we have, with respect to the desired end state, as well as what we could improve on. It gives us an opportunity to gauge approaching opportunities and dangers, and assess the seriousness of the conditions that affect our future. When we understand those conditions, we can influence what comes next.

Agreement-Certainty Matrix

Not every problem-solving approach is right for every challenge, and deciding on the right method for the challenge at hand is a key part of being an effective team.

The Agreement Certainty matrix helps teams align on the nature of the challenges facing them. By sorting problems from simple to chaotic, your team can understand what methods are suitable for each problem and what they can do to ensure effective results. 

If you are already using Liberating Structures techniques as part of your problem-solving strategy, the Agreement-Certainty Matrix can be an invaluable addition to your process. We’ve found it particularly if you are having issues with recurring problems in your organization and want to go deeper in understanding the root cause. 

Agreement-Certainty Matrix   #issue analysis   #liberating structures   #problem solving   You can help individuals or groups avoid the frequent mistake of trying to solve a problem with methods that are not adapted to the nature of their challenge. The combination of two questions makes it possible to easily sort challenges into four categories: simple, complicated, complex , and chaotic .  A problem is simple when it can be solved reliably with practices that are easy to duplicate.  It is complicated when experts are required to devise a sophisticated solution that will yield the desired results predictably.  A problem is complex when there are several valid ways to proceed but outcomes are not predictable in detail.  Chaotic is when the context is too turbulent to identify a path forward.  A loose analogy may be used to describe these differences: simple is like following a recipe, complicated like sending a rocket to the moon, complex like raising a child, and chaotic is like the game “Pin the Tail on the Donkey.”  The Liberating Structures Matching Matrix in Chapter 5 can be used as the first step to clarify the nature of a challenge and avoid the mismatches between problems and solutions that are frequently at the root of chronic, recurring problems.

Organizing and charting a team’s progress can be important in ensuring its success. SQUID (Sequential Question and Insight Diagram) is a great model that allows a team to effectively switch between giving questions and answers and develop the skills they need to stay on track throughout the process. 

Begin with two different colored sticky notes – one for questions and one for answers – and with your central topic (the head of the squid) on the board. Ask the group to first come up with a series of questions connected to their best guess of how to approach the topic. Ask the group to come up with answers to those questions, fix them to the board and connect them with a line. After some discussion, go back to question mode by responding to the generated answers or other points on the board.

It’s rewarding to see a diagram grow throughout the exercise, and a completed SQUID can provide a visual resource for future effort and as an example for other teams.

SQUID   #gamestorming   #project planning   #issue analysis   #problem solving   When exploring an information space, it’s important for a group to know where they are at any given time. By using SQUID, a group charts out the territory as they go and can navigate accordingly. SQUID stands for Sequential Question and Insight Diagram.

To continue with our nautical theme, Speed Boat is a short and sweet activity that can help a team quickly identify what employees, clients or service users might have a problem with and analyze what might be standing in the way of achieving a solution.

Methods that allow for a group to make observations, have insights and obtain those eureka moments quickly are invaluable when trying to solve complex problems.

In Speed Boat, the approach is to first consider what anchors and challenges might be holding an organization (or boat) back. Bonus points if you are able to identify any sharks in the water and develop ideas that can also deal with competitors!   

Speed Boat   #gamestorming   #problem solving   #action   Speedboat is a short and sweet way to identify what your employees or clients don’t like about your product/service or what’s standing in the way of a desired goal.

The Journalistic Six

Some of the most effective ways of solving problems is by encouraging teams to be more inclusive and diverse in their thinking.

Based on the six key questions journalism students are taught to answer in articles and news stories, The Journalistic Six helps create teams to see the whole picture. By using who, what, when, where, why, and how to facilitate the conversation and encourage creative thinking, your team can make sure that the problem identification and problem analysis stages of the are covered exhaustively and thoughtfully. Reporter’s notebook and dictaphone optional.

The Journalistic Six – Who What When Where Why How   #idea generation   #issue analysis   #problem solving   #online   #creative thinking   #remote-friendly   A questioning method for generating, explaining, investigating ideas.

Individual and group perspectives are incredibly important, but what happens if people are set in their minds and need a change of perspective in order to approach a problem more effectively?

Flip It is a method we love because it is both simple to understand and run, and allows groups to understand how their perspectives and biases are formed. 

Participants in Flip It are first invited to consider concerns, issues, or problems from a perspective of fear and write them on a flip chart. Then, the group is asked to consider those same issues from a perspective of hope and flip their understanding.  

No problem and solution is free from existing bias and by changing perspectives with Flip It, you can then develop a problem solving model quickly and effectively.

Flip It!   #gamestorming   #problem solving   #action   Often, a change in a problem or situation comes simply from a change in our perspectives. Flip It! is a quick game designed to show players that perspectives are made, not born.

LEGO Challenge

Now for an activity that is a little out of the (toy) box. LEGO Serious Play is a facilitation methodology that can be used to improve creative thinking and problem-solving skills. 

The LEGO Challenge includes giving each member of the team an assignment that is hidden from the rest of the group while they create a structure without speaking.

What the LEGO challenge brings to the table is a fun working example of working with stakeholders who might not be on the same page to solve problems. Also, it’s LEGO! Who doesn’t love LEGO! 

LEGO Challenge   #hyperisland   #team   A team-building activity in which groups must work together to build a structure out of LEGO, but each individual has a secret “assignment” which makes the collaborative process more challenging. It emphasizes group communication, leadership dynamics, conflict, cooperation, patience and problem solving strategy.

What, So What, Now What?

If not carefully managed, the problem identification and problem analysis stages of the problem-solving process can actually create more problems and misunderstandings.

The What, So What, Now What? problem-solving activity is designed to help collect insights and move forward while also eliminating the possibility of disagreement when it comes to identifying, clarifying, and analyzing organizational or work problems. 

Facilitation is all about bringing groups together so that might work on a shared goal and the best problem-solving strategies ensure that teams are aligned in purpose, if not initially in opinion or insight.

Throughout the three steps of this game, you give everyone on a team to reflect on a problem by asking what happened, why it is important, and what actions should then be taken. 

This can be a great activity for bringing our individual perceptions about a problem or challenge and contextualizing it in a larger group setting. This is one of the most important problem-solving skills you can bring to your organization.

W³ – What, So What, Now What?   #issue analysis   #innovation   #liberating structures   You can help groups reflect on a shared experience in a way that builds understanding and spurs coordinated action while avoiding unproductive conflict. It is possible for every voice to be heard while simultaneously sifting for insights and shaping new direction. Progressing in stages makes this practical—from collecting facts about What Happened to making sense of these facts with So What and finally to what actions logically follow with Now What . The shared progression eliminates most of the misunderstandings that otherwise fuel disagreements about what to do. Voila!

Journalists  

Problem analysis can be one of the most important and decisive stages of all problem-solving tools. Sometimes, a team can become bogged down in the details and are unable to move forward.

Journalists is an activity that can avoid a group from getting stuck in the problem identification or problem analysis stages of the process.

In Journalists, the group is invited to draft the front page of a fictional newspaper and figure out what stories deserve to be on the cover and what headlines those stories will have. By reframing how your problems and challenges are approached, you can help a team move productively through the process and be better prepared for the steps to follow.

Journalists   #vision   #big picture   #issue analysis   #remote-friendly   This is an exercise to use when the group gets stuck in details and struggles to see the big picture. Also good for defining a vision.

Problem-solving techniques for brainstorming solutions

Now you have the context and background of the problem you are trying to solving, now comes the time to start ideating and thinking about how you’ll solve the issue.

Here, you’ll want to encourage creative, free thinking and speed. Get as many ideas out as possible and explore different perspectives so you have the raw material for the next step.

Looking at a problem from a new angle can be one of the most effective ways of creating an effective solution. TRIZ is a problem-solving tool that asks the group to consider what they must not do in order to solve a challenge.

By reversing the discussion, new topics and taboo subjects often emerge, allowing the group to think more deeply and create ideas that confront the status quo in a safe and meaningful way. If you’re working on a problem that you’ve tried to solve before, TRIZ is a great problem-solving method to help your team get unblocked.

Making Space with TRIZ   #issue analysis   #liberating structures   #issue resolution   You can clear space for innovation by helping a group let go of what it knows (but rarely admits) limits its success and by inviting creative destruction. TRIZ makes it possible to challenge sacred cows safely and encourages heretical thinking. The question “What must we stop doing to make progress on our deepest purpose?” induces seriously fun yet very courageous conversations. Since laughter often erupts, issues that are otherwise taboo get a chance to be aired and confronted. With creative destruction come opportunities for renewal as local action and innovation rush in to fill the vacuum. Whoosh!

Mindspin  

Brainstorming is part of the bread and butter of the problem-solving process and all problem-solving strategies benefit from getting ideas out and challenging a team to generate solutions quickly. 

With Mindspin, participants are encouraged not only to generate ideas but to do so under time constraints and by slamming down cards and passing them on. By doing multiple rounds, your team can begin with a free generation of possible solutions before moving on to developing those solutions and encouraging further ideation. 

This is one of our favorite problem-solving activities and can be great for keeping the energy up throughout the workshop. Remember the importance of helping people become engaged in the process – energizing problem-solving techniques like Mindspin can help ensure your team stays engaged and happy, even when the problems they’re coming together to solve are complex. 

MindSpin   #teampedia   #idea generation   #problem solving   #action   A fast and loud method to enhance brainstorming within a team. Since this activity has more than round ideas that are repetitive can be ruled out leaving more creative and innovative answers to the challenge.

The Creativity Dice

One of the most useful problem solving skills you can teach your team is of approaching challenges with creativity, flexibility, and openness. Games like The Creativity Dice allow teams to overcome the potential hurdle of too much linear thinking and approach the process with a sense of fun and speed. 

In The Creativity Dice, participants are organized around a topic and roll a dice to determine what they will work on for a period of 3 minutes at a time. They might roll a 3 and work on investigating factual information on the chosen topic. They might roll a 1 and work on identifying the specific goals, standards, or criteria for the session.

Encouraging rapid work and iteration while asking participants to be flexible are great skills to cultivate. Having a stage for idea incubation in this game is also important. Moments of pause can help ensure the ideas that are put forward are the most suitable. 

The Creativity Dice   #creativity   #problem solving   #thiagi   #issue analysis   Too much linear thinking is hazardous to creative problem solving. To be creative, you should approach the problem (or the opportunity) from different points of view. You should leave a thought hanging in mid-air and move to another. This skipping around prevents premature closure and lets your brain incubate one line of thought while you consciously pursue another.

Idea and Concept Development

Brainstorming without structure can quickly become chaotic or frustrating. In a problem-solving context, having an ideation framework to follow can help ensure your team is both creative and disciplined.

In this method, you’ll find an idea generation process that encourages your group to brainstorm effectively before developing their ideas and begin clustering them together. By using concepts such as Yes and…, more is more and postponing judgement, you can create the ideal conditions for brainstorming with ease.

Idea & Concept Development   #hyperisland   #innovation   #idea generation   Ideation and Concept Development is a process for groups to work creatively and collaboratively to generate creative ideas. It’s a general approach that can be adapted and customized to suit many different scenarios. It includes basic principles for idea generation and several steps for groups to work with. It also includes steps for idea selection and development.

Problem-solving techniques for developing and refining solutions 

The success of any problem-solving process can be measured by the solutions it produces. After you’ve defined the issue, explored existing ideas, and ideated, it’s time to develop and refine your ideas in order to bring them closer to a solution that actually solves the problem.

Use these problem-solving techniques when you want to help your team think through their ideas and refine them as part of your problem solving process.

Improved Solutions

After a team has successfully identified a problem and come up with a few solutions, it can be tempting to call the work of the problem-solving process complete. That said, the first solution is not necessarily the best, and by including a further review and reflection activity into your problem-solving model, you can ensure your group reaches the best possible result. 

One of a number of problem-solving games from Thiagi Group, Improved Solutions helps you go the extra mile and develop suggested solutions with close consideration and peer review. By supporting the discussion of several problems at once and by shifting team roles throughout, this problem-solving technique is a dynamic way of finding the best solution. 

Improved Solutions   #creativity   #thiagi   #problem solving   #action   #team   You can improve any solution by objectively reviewing its strengths and weaknesses and making suitable adjustments. In this creativity framegame, you improve the solutions to several problems. To maintain objective detachment, you deal with a different problem during each of six rounds and assume different roles (problem owner, consultant, basher, booster, enhancer, and evaluator) during each round. At the conclusion of the activity, each player ends up with two solutions to her problem.

Four Step Sketch

Creative thinking and visual ideation does not need to be confined to the opening stages of your problem-solving strategies. Exercises that include sketching and prototyping on paper can be effective at the solution finding and development stage of the process, and can be great for keeping a team engaged. 

By going from simple notes to a crazy 8s round that involves rapidly sketching 8 variations on their ideas before then producing a final solution sketch, the group is able to iterate quickly and visually. Problem-solving techniques like Four-Step Sketch are great if you have a group of different thinkers and want to change things up from a more textual or discussion-based approach.

Four-Step Sketch   #design sprint   #innovation   #idea generation   #remote-friendly   The four-step sketch is an exercise that helps people to create well-formed concepts through a structured process that includes: Review key information Start design work on paper,  Consider multiple variations , Create a detailed solution . This exercise is preceded by a set of other activities allowing the group to clarify the challenge they want to solve. See how the Four Step Sketch exercise fits into a Design Sprint

Ensuring that everyone in a group is able to contribute to a discussion is vital during any problem solving process. Not only does this ensure all bases are covered, but its then easier to get buy-in and accountability when people have been able to contribute to the process.

1-2-4-All is a tried and tested facilitation technique where participants are asked to first brainstorm on a topic on their own. Next, they discuss and share ideas in a pair before moving into a small group. Those groups are then asked to present the best idea from their discussion to the rest of the team.

This method can be used in many different contexts effectively, though I find it particularly shines in the idea development stage of the process. Giving each participant time to concretize their ideas and develop them in progressively larger groups can create a great space for both innovation and psychological safety.

1-2-4-All   #idea generation   #liberating structures   #issue analysis   With this facilitation technique you can immediately include everyone regardless of how large the group is. You can generate better ideas and more of them faster than ever before. You can tap the know-how and imagination that is distributed widely in places not known in advance. Open, generative conversation unfolds. Ideas and solutions are sifted in rapid fashion. Most importantly, participants own the ideas, so follow-up and implementation is simplified. No buy-in strategies needed! Simple and elegant!

15% Solutions

Some problems are simpler than others and with the right problem-solving activities, you can empower people to take immediate actions that can help create organizational change. 

Part of the liberating structures toolkit, 15% solutions is a problem-solving technique that focuses on finding and implementing solutions quickly. A process of iterating and making small changes quickly can help generate momentum and an appetite for solving complex problems.

Problem-solving strategies can live and die on whether people are onboard. Getting some quick wins is a great way of getting people behind the process.   

It can be extremely empowering for a team to realize that problem-solving techniques can be deployed quickly and easily and delineate between things they can positively impact and those things they cannot change. 

15% Solutions   #action   #liberating structures   #remote-friendly   You can reveal the actions, however small, that everyone can do immediately. At a minimum, these will create momentum, and that may make a BIG difference.  15% Solutions show that there is no reason to wait around, feel powerless, or fearful. They help people pick it up a level. They get individuals and the group to focus on what is within their discretion instead of what they cannot change.  With a very simple question, you can flip the conversation to what can be done and find solutions to big problems that are often distributed widely in places not known in advance. Shifting a few grains of sand may trigger a landslide and change the whole landscape.

Problem-solving techniques for making decisions and planning

After your group is happy with the possible solutions you’ve developed, now comes the time to choose which to implement. There’s more than one way to make a decision and the best option is often dependant on the needs and set-up of your group.

Sometimes, it’s the case that you’ll want to vote as a group on what is likely to be the most impactful solution. Other times, it might be down to a decision maker or major stakeholder to make the final decision. Whatever your process, here’s some techniques you can use to help you make a decision during your problem solving process.

How-Now-Wow Matrix

The problem-solving process is often creative, as complex problems usually require a change of thinking and creative response in order to find the best solutions. While it’s common for the first stages to encourage creative thinking, groups can often gravitate to familiar solutions when it comes to the end of the process. 

When selecting solutions, you don’t want to lose your creative energy! The How-Now-Wow Matrix from Gamestorming is a great problem-solving activity that enables a group to stay creative and think out of the box when it comes to selecting the right solution for a given problem.

Problem-solving techniques that encourage creative thinking and the ideation and selection of new solutions can be the most effective in organisational change. Give the How-Now-Wow Matrix a go, and not just for how pleasant it is to say out loud. 

How-Now-Wow Matrix   #gamestorming   #idea generation   #remote-friendly   When people want to develop new ideas, they most often think out of the box in the brainstorming or divergent phase. However, when it comes to convergence, people often end up picking ideas that are most familiar to them. This is called a ‘creative paradox’ or a ‘creadox’. The How-Now-Wow matrix is an idea selection tool that breaks the creadox by forcing people to weigh each idea on 2 parameters.

Impact and Effort Matrix

All problem-solving techniques hope to not only find solutions to a given problem or challenge but to find the best solution. When it comes to finding a solution, groups are invited to put on their decision-making hats and really think about how a proposed idea would work in practice. 

The Impact and Effort Matrix is one of the problem-solving techniques that fall into this camp, empowering participants to first generate ideas and then categorize them into a 2×2 matrix based on impact and effort.

Activities that invite critical thinking while remaining simple are invaluable. Use the Impact and Effort Matrix to move from ideation and towards evaluating potential solutions before then committing to them. 

Impact and Effort Matrix   #gamestorming   #decision making   #action   #remote-friendly   In this decision-making exercise, possible actions are mapped based on two factors: effort required to implement and potential impact. Categorizing ideas along these lines is a useful technique in decision making, as it obliges contributors to balance and evaluate suggested actions before committing to them.

If you’ve followed each of the problem-solving steps with your group successfully, you should move towards the end of your process with heaps of possible solutions developed with a specific problem in mind. But how do you help a group go from ideation to putting a solution into action? 

Dotmocracy – or Dot Voting -is a tried and tested method of helping a team in the problem-solving process make decisions and put actions in place with a degree of oversight and consensus. 

One of the problem-solving techniques that should be in every facilitator’s toolbox, Dot Voting is fast and effective and can help identify the most popular and best solutions and help bring a group to a decision effectively. 

Dotmocracy   #action   #decision making   #group prioritization   #hyperisland   #remote-friendly   Dotmocracy is a simple method for group prioritization or decision-making. It is not an activity on its own, but a method to use in processes where prioritization or decision-making is the aim. The method supports a group to quickly see which options are most popular or relevant. The options or ideas are written on post-its and stuck up on a wall for the whole group to see. Each person votes for the options they think are the strongest, and that information is used to inform a decision.

Straddling the gap between decision making and planning, MoSCoW is a simple and effective method that allows a group team to easily prioritize a set of possible options.

Use this method in a problem solving process by collecting and summarizing all your possible solutions and then categorize them into 4 sections: “Must have”, “Should have”, “Could have”, or “Would like but won‘t get”.

This method is particularly useful when its less about choosing one possible solution and more about prioritorizing which to do first and which may not fit in the scope of your project. In my experience, complex challenges often require multiple small fixes, and this method can be a great way to move from a pile of things you’d all like to do to a structured plan.

MoSCoW   #define intentions   #create   #design   #action   #remote-friendly   MoSCoW is a method that allows the team to prioritize the different features that they will work on. Features are then categorized into “Must have”, “Should have”, “Could have”, or “Would like but won‘t get”. To be used at the beginning of a timeslot (for example during Sprint planning) and when planning is needed.

When it comes to managing the rollout of a solution, clarity and accountability are key factors in ensuring the success of the project. The RAACI chart is a simple but effective model for setting roles and responsibilities as part of a planning session.

Start by listing each person involved in the project and put them into the following groups in order to make it clear who is responsible for what during the rollout of your solution.

  • Responsibility  (Which person and/or team will be taking action?)
  • Authority  (At what “point” must the responsible person check in before going further?)
  • Accountability  (Who must the responsible person check in with?)
  • Consultation  (Who must be consulted by the responsible person before decisions are made?)
  • Information  (Who must be informed of decisions, once made?)

Ensure this information is easily accessible and use it to inform who does what and who is looped into discussions and kept up to date.

RAACI   #roles and responsibility   #teamwork   #project management   Clarifying roles and responsibilities, levels of autonomy/latitude in decision making, and levels of engagement among diverse stakeholders.

Problem-solving warm-up activities

All facilitators know that warm-ups and icebreakers are useful for any workshop or group process. Problem-solving workshops are no different.

Use these problem-solving techniques to warm up a group and prepare them for the rest of the process. Activating your group by tapping into some of the top problem-solving skills can be one of the best ways to see great outcomes from your session.

Check-in / Check-out

Solid processes are planned from beginning to end, and the best facilitators know that setting the tone and establishing a safe, open environment can be integral to a successful problem-solving process. Check-in / Check-out is a great way to begin and/or bookend a problem-solving workshop. Checking in to a session emphasizes that everyone will be seen, heard, and expected to contribute. 

If you are running a series of meetings, setting a consistent pattern of checking in and checking out can really help your team get into a groove. We recommend this opening-closing activity for small to medium-sized groups though it can work with large groups if they’re disciplined!

Check-in / Check-out   #team   #opening   #closing   #hyperisland   #remote-friendly   Either checking-in or checking-out is a simple way for a team to open or close a process, symbolically and in a collaborative way. Checking-in/out invites each member in a group to be present, seen and heard, and to express a reflection or a feeling. Checking-in emphasizes presence, focus and group commitment; checking-out emphasizes reflection and symbolic closure.

Doodling Together  

Thinking creatively and not being afraid to make suggestions are important problem-solving skills for any group or team, and warming up by encouraging these behaviors is a great way to start. 

Doodling Together is one of our favorite creative ice breaker games – it’s quick, effective, and fun and can make all following problem-solving steps easier by encouraging a group to collaborate visually. By passing cards and adding additional items as they go, the workshop group gets into a groove of co-creation and idea development that is crucial to finding solutions to problems. 

Doodling Together   #collaboration   #creativity   #teamwork   #fun   #team   #visual methods   #energiser   #icebreaker   #remote-friendly   Create wild, weird and often funny postcards together & establish a group’s creative confidence.

Show and Tell

You might remember some version of Show and Tell from being a kid in school and it’s a great problem-solving activity to kick off a session.

Asking participants to prepare a little something before a workshop by bringing an object for show and tell can help them warm up before the session has even begun! Games that include a physical object can also help encourage early engagement before moving onto more big-picture thinking.

By asking your participants to tell stories about why they chose to bring a particular item to the group, you can help teams see things from new perspectives and see both differences and similarities in the way they approach a topic. Great groundwork for approaching a problem-solving process as a team! 

Show and Tell   #gamestorming   #action   #opening   #meeting facilitation   Show and Tell taps into the power of metaphors to reveal players’ underlying assumptions and associations around a topic The aim of the game is to get a deeper understanding of stakeholders’ perspectives on anything—a new project, an organizational restructuring, a shift in the company’s vision or team dynamic.

Constellations

Who doesn’t love stars? Constellations is a great warm-up activity for any workshop as it gets people up off their feet, energized, and ready to engage in new ways with established topics. It’s also great for showing existing beliefs, biases, and patterns that can come into play as part of your session.

Using warm-up games that help build trust and connection while also allowing for non-verbal responses can be great for easing people into the problem-solving process and encouraging engagement from everyone in the group. Constellations is great in large spaces that allow for movement and is definitely a practical exercise to allow the group to see patterns that are otherwise invisible. 

Constellations   #trust   #connection   #opening   #coaching   #patterns   #system   Individuals express their response to a statement or idea by standing closer or further from a central object. Used with teams to reveal system, hidden patterns, perspectives.

Draw a Tree

Problem-solving games that help raise group awareness through a central, unifying metaphor can be effective ways to warm-up a group in any problem-solving model.

Draw a Tree is a simple warm-up activity you can use in any group and which can provide a quick jolt of energy. Start by asking your participants to draw a tree in just 45 seconds – they can choose whether it will be abstract or realistic. 

Once the timer is up, ask the group how many people included the roots of the tree and use this as a means to discuss how we can ignore important parts of any system simply because they are not visible.

All problem-solving strategies are made more effective by thinking of problems critically and by exposing things that may not normally come to light. Warm-up games like Draw a Tree are great in that they quickly demonstrate some key problem-solving skills in an accessible and effective way.

Draw a Tree   #thiagi   #opening   #perspectives   #remote-friendly   With this game you can raise awarness about being more mindful, and aware of the environment we live in.

Closing activities for a problem-solving process

Each step of the problem-solving workshop benefits from an intelligent deployment of activities, games, and techniques. Bringing your session to an effective close helps ensure that solutions are followed through on and that you also celebrate what has been achieved.

Here are some problem-solving activities you can use to effectively close a workshop or meeting and ensure the great work you’ve done can continue afterward.

One Breath Feedback

Maintaining attention and focus during the closing stages of a problem-solving workshop can be tricky and so being concise when giving feedback can be important. It’s easy to incur “death by feedback” should some team members go on for too long sharing their perspectives in a quick feedback round. 

One Breath Feedback is a great closing activity for workshops. You give everyone an opportunity to provide feedback on what they’ve done but only in the space of a single breath. This keeps feedback short and to the point and means that everyone is encouraged to provide the most important piece of feedback to them. 

One breath feedback   #closing   #feedback   #action   This is a feedback round in just one breath that excels in maintaining attention: each participants is able to speak during just one breath … for most people that’s around 20 to 25 seconds … unless of course you’ve been a deep sea diver in which case you’ll be able to do it for longer.

Who What When Matrix 

Matrices feature as part of many effective problem-solving strategies and with good reason. They are easily recognizable, simple to use, and generate results.

The Who What When Matrix is a great tool to use when closing your problem-solving session by attributing a who, what and when to the actions and solutions you have decided upon. The resulting matrix is a simple, easy-to-follow way of ensuring your team can move forward. 

Great solutions can’t be enacted without action and ownership. Your problem-solving process should include a stage for allocating tasks to individuals or teams and creating a realistic timeframe for those solutions to be implemented or checked out. Use this method to keep the solution implementation process clear and simple for all involved. 

Who/What/When Matrix   #gamestorming   #action   #project planning   With Who/What/When matrix, you can connect people with clear actions they have defined and have committed to.

Response cards

Group discussion can comprise the bulk of most problem-solving activities and by the end of the process, you might find that your team is talked out! 

Providing a means for your team to give feedback with short written notes can ensure everyone is head and can contribute without the need to stand up and talk. Depending on the needs of the group, giving an alternative can help ensure everyone can contribute to your problem-solving model in the way that makes the most sense for them.

Response Cards is a great way to close a workshop if you are looking for a gentle warm-down and want to get some swift discussion around some of the feedback that is raised. 

Response Cards   #debriefing   #closing   #structured sharing   #questions and answers   #thiagi   #action   It can be hard to involve everyone during a closing of a session. Some might stay in the background or get unheard because of louder participants. However, with the use of Response Cards, everyone will be involved in providing feedback or clarify questions at the end of a session.

Tips for effective problem solving

Problem-solving activities are only one part of the puzzle. While a great method can help unlock your team’s ability to solve problems, without a thoughtful approach and strong facilitation the solutions may not be fit for purpose.

Let’s take a look at some problem-solving tips you can apply to any process to help it be a success!

Clearly define the problem

Jumping straight to solutions can be tempting, though without first clearly articulating a problem, the solution might not be the right one. Many of the problem-solving activities below include sections where the problem is explored and clearly defined before moving on.

This is a vital part of the problem-solving process and taking the time to fully define an issue can save time and effort later. A clear definition helps identify irrelevant information and it also ensures that your team sets off on the right track.

Don’t jump to conclusions

It’s easy for groups to exhibit cognitive bias or have preconceived ideas about both problems and potential solutions. Be sure to back up any problem statements or potential solutions with facts, research, and adequate forethought.

The best techniques ask participants to be methodical and challenge preconceived notions. Make sure you give the group enough time and space to collect relevant information and consider the problem in a new way. By approaching the process with a clear, rational mindset, you’ll often find that better solutions are more forthcoming.  

Try different approaches  

Problems come in all shapes and sizes and so too should the methods you use to solve them. If you find that one approach isn’t yielding results and your team isn’t finding different solutions, try mixing it up. You’ll be surprised at how using a new creative activity can unblock your team and generate great solutions.

Don’t take it personally 

Depending on the nature of your team or organizational problems, it’s easy for conversations to get heated. While it’s good for participants to be engaged in the discussions, ensure that emotions don’t run too high and that blame isn’t thrown around while finding solutions.

You’re all in it together, and even if your team or area is seeing problems, that isn’t necessarily a disparagement of you personally. Using facilitation skills to manage group dynamics is one effective method of helping conversations be more constructive.

Get the right people in the room

Your problem-solving method is often only as effective as the group using it. Getting the right people on the job and managing the number of people present is important too!

If the group is too small, you may not get enough different perspectives to effectively solve a problem. If the group is too large, you can go round and round during the ideation stages.

Creating the right group makeup is also important in ensuring you have the necessary expertise and skillset to both identify and follow up on potential solutions. Carefully consider who to include at each stage to help ensure your problem-solving method is followed and positioned for success.

Create psychologically safe spaces for discussion

Identifying a problem accurately also requires that all members of a group are able to contribute their views in an open and safe manner.

It can be tough for people to stand up and contribute if the problems or challenges are emotive or personal in nature. Try and create a psychologically safe space for these kinds of discussions and where possible, create regular opportunities for challenges to be brought up organically.

Document everything

The best solutions can take refinement, iteration, and reflection to come out. Get into a habit of documenting your process in order to keep all the learnings from the session and to allow ideas to mature and develop. Many of the methods below involve the creation of documents or shared resources. Be sure to keep and share these so everyone can benefit from the work done!

Bring a facilitator 

Facilitation is all about making group processes easier. With a subject as potentially emotive and important as problem-solving, having an impartial third party in the form of a facilitator can make all the difference in finding great solutions and keeping the process moving. Consider bringing a facilitator to your problem-solving session to get better results and generate meaningful solutions!

Develop your problem-solving skills

It takes time and practice to be an effective problem solver. While some roles or participants might more naturally gravitate towards problem-solving, it can take development and planning to help everyone create better solutions.

You might develop a training program, run a problem-solving workshop or simply ask your team to practice using the techniques below. Check out our post on problem-solving skills to see how you and your group can develop the right mental process and be more resilient to issues too!

Design a great agenda

Workshops are a great format for solving problems. With the right approach, you can focus a group and help them find the solutions to their own problems. But designing a process can be time-consuming and finding the right activities can be difficult.

Check out our workshop planning guide to level-up your agenda design and start running more effective workshops. Need inspiration? Check out templates designed by expert facilitators to help you kickstart your process!

Save time and effort creating an effective problem solving process

A structured problem solving process is a surefire way of solving tough problems, discovering creative solutions and driving organizational change. But how can you design for successful outcomes?

With SessionLab, it’s easy to design engaging workshops that deliver results. Drag, drop and reorder blocks  to build your agenda. When you make changes or update your agenda, your session  timing   adjusts automatically , saving you time on manual adjustments.

Collaborating with stakeholders or clients? Share your agenda with a single click and collaborate in real-time. No more sending documents back and forth over email.

Explore  how to use SessionLab  to design effective problem solving workshops or  watch this five minute video  to see the planner in action!

answer in problem solving

Over to you

The problem-solving process can often be as complicated and multifaceted as the problems they are set-up to solve. With the right problem-solving techniques and a mix of exercises designed to guide discussion and generate purposeful ideas, we hope we’ve given you the tools to find the best solutions as simply and easily as possible.

Is there a problem-solving technique that you are missing here? Do you have a favorite activity or method you use when facilitating? Let us know in the comments below, we’d love to hear from you! 

' src=

thank you very much for these excellent techniques

' src=

Certainly wonderful article, very detailed. Shared!

' src=

Your list of techniques for problem solving can be helpfully extended by adding TRIZ to the list of techniques. TRIZ has 40 problem solving techniques derived from methods inventros and patent holders used to get new patents. About 10-12 are general approaches. many organization sponsor classes in TRIZ that are used to solve business problems or general organiztational problems. You can take a look at TRIZ and dwonload a free internet booklet to see if you feel it shound be included per your selection process.

Leave a Comment Cancel reply

Your email address will not be published. Required fields are marked *

cycle of workshop planning steps

Going from a mere idea to a workshop that delivers results for your clients can feel like a daunting task. In this piece, we will shine a light on all the work behind the scenes and help you learn how to plan a workshop from start to finish. On a good day, facilitation can feel like effortless magic, but that is mostly the result of backstage work, foresight, and a lot of careful planning. Read on to learn a step-by-step approach to breaking the process of planning a workshop into small, manageable chunks.  The flow starts with the first meeting with a client to define the purposes of a workshop.…

answer in problem solving

Effective online tools are a necessity for smooth and engaging virtual workshops and meetings. But how do you choose the right ones? Do you sometimes feel that the good old pen and paper or MS Office toolkit and email leaves you struggling to stay on top of managing and delivering your workshop? Fortunately, there are plenty of great workshop tools to make your life easier when you need to facilitate a meeting and lead workshops. In this post, we’ll share our favorite online tools you can use to make your life easier and run better workshops and meetings. In fact, there are plenty of free online workshop tools and meeting…

answer in problem solving

How does learning work? A clever 9-year-old once told me: “I know I am learning something new when I am surprised.” The science of adult learning tells us that, in order to learn new skills (which, unsurprisingly, is harder for adults to do than kids) grown-ups need to first get into a specific headspace.  In a business, this approach is often employed in a training session where employees learn new skills or work on professional development. But how do you ensure your training is effective? In this guide, we'll explore how to create an effective training session plan and run engaging training sessions. As team leader, project manager, or consultant,…

Design your next workshop with SessionLab

Join the 150,000 facilitators using SessionLab

Sign up for free

InterviewPrep

Top 20 Problem Solving Skills Interview Questions & Answers

Master your responses to Problem Solving Skills related interview questions with our example questions and answers. Boost your chances of landing the job by learning how to effectively communicate your Problem Solving Skills capabilities.

answer in problem solving

Problem-solving skills are universal currency across industries and job roles. They’re the backbone of successful projects, the driving force behind effective leadership, and the hallmark of an individual who can navigate complex challenges with poise and acumen. Whether you’re applying for an entry-level position or stepping into a senior role, your ability to identify problems, analyze their components, and devise efficient solutions will be under the microscope in any interview setting.

To help you prepare for questions that probe this critical skill set, we’ve put together a comprehensive look at some of the most common problem-solving related interview inquiries. We’ll also provide strategic approaches to these queries, offering insight on how to structure your responses and share examples that demonstrate your prowess in turning obstacles into opportunities.

Common Problem Solving Skills Interview Questions

1. describe a situation where you had to solve a problem with no obvious solution..

Having strong problem-solving skills is crucial in many job roles, particularly in positions where unexpected challenges are common. This question allows the interviewer to assess if a candidate can think outside the box, remain calm under pressure, and leverage their knowledge and resources to navigate through uncertain scenarios.

When responding, it’s important to outline a specific example that illustrates your thought process and the steps you took to address the problem. Focus on articulating the nature of the problem, the various options you considered, how you weighed the risks and benefits of each, and the outcome of your actions. It’s also beneficial to highlight any unique or innovative approaches you used, how you collaborated with others if applicable, and what you learned from the experience.

Example: “ In a situation where a critical system failed unexpectedly, I was faced with a problem that had no clear solution. The system’s failure was causing significant operational delays, and the usual troubleshooting methods were not yielding any results. I started by breaking down the problem into smaller, more manageable components to isolate the failure’s root cause. I considered various unconventional options, such as repurposing similar systems, modifying existing workflows, and even developing a temporary manual process.

After evaluating the risks and benefits of each potential solution, I decided to implement a hybrid approach. This involved a temporary manual workaround to resume operations immediately, alongside a parallel effort to adapt a similar system for a more sustainable short-term fix. This approach minimized downtime and maintained productivity while we worked on a permanent solution. The outcome was successful, as it not only resolved the immediate crisis but also led to the development of a more robust contingency plan for future system failures. This experience underscored the importance of adaptability and the value of a methodical approach to problem-solving under pressure.”

2. How do you approach defining the root cause of a complex issue?

To effectively untangle intricate issues, a methodical approach is key. Your analytical abilities and systematic methodology for identifying, dissecting, and understanding the foundational aspects of a problem are what interviewers are keen to understand.

To respond effectively, outline a step-by-step process that demonstrates your logical and structured approach. You might talk about gathering and analyzing data, using tools such as the “Five Whys” technique to drill down to the underlying cause, or how you consider the broader context to ensure you’re not overlooking external factors. Emphasize your ability to remain objective, avoid assumptions, and how you collaborate with others to gain different perspectives. Highlight past experiences where you successfully identified and addressed the root cause of a complex problem, which led to a sustainable solution.

Example: “ null”

3. What strategies do you employ when faced with multiple competing solutions?

A delicate balance of creativity, logic, and the ability to weigh options against potential outcomes is what defines decisive problem-solving. Your approach can reveal your critical thinking skills, adaptability, and prioritization techniques.

When responding, outline a structured method that you use, such as listing pros and cons, assessing risks, or consulting with key team members. It’s important to demonstrate that you can make informed decisions by evaluating the evidence and considering the broader context of the problem. Make sure to provide a real-life example that illustrates your process and the successful outcome that resulted from your chosen strategy. Highlight any tools or techniques you find particularly useful, such as decision matrices or brainstorming sessions, and explain how these help you to systematically address the issue at hand.

Example: “ When faced with multiple competing solutions, I employ a structured approach that begins with defining clear criteria based on the goals and constraints of the situation. These criteria typically include factors such as feasibility, cost, time, resources, and potential impact. I then use a decision matrix to systematically evaluate each option against these criteria, scoring them to quantify their suitability. This method allows for an objective comparison and helps to minimize bias in the decision-making process.

For instance, in a scenario where I had to choose between several technical solutions to optimize a workflow, I developed a matrix that included criteria such as integration complexity, scalability, and user adoption. After scoring each solution, it became evident that one option, despite not being the most advanced, offered the best balance between ease of implementation and potential benefits. The chosen solution was successfully implemented, leading to a 30% increase in workflow efficiency. This experience underscored the importance of a methodical approach to decision-making, ensuring that choices are made based on data-driven analysis rather than intuition alone.”

4. Can you provide an example of a time when you had to adapt your problem-solving approach mid-process?

Demonstrating adaptability in problem-solving shows your ability to navigate the unpredictable and often complex landscape of challenges that arise in any role. This question delves into your flexibility, resilience, and critical thinking skills, as well as your willingness to learn from the process and collaborate with others.

When responding to this question, outline a specific situation where your original approach to a problem didn’t pan out as expected. Explain the thought process behind your initial plan, the moment you realized a change was needed, and how you adjusted your strategy. Emphasize the outcome of your adaptability and what you learned from the experience, ensuring you showcase your ability to remain composed and innovative in the face of obstacles.

Example: “ Certainly. In one instance, I was tackling a complex issue where the initial data analysis suggested a straightforward solution. However, as the implementation progressed, unexpected variables surfaced that rendered our original plan ineffective. Recognizing this, I paused the execution and gathered the team for a brainstorming session to reassess the situation. It became clear that we needed a more robust data set to understand the underlying patterns causing the discrepancy.

We pivoted our approach to include a broader range of data sources and employed advanced analytical techniques, such as predictive modeling, to gain deeper insights. This shift not only resolved the immediate problem but also provided us with a more sophisticated framework for future issues. The key takeaway from this experience was the importance of agility in problem-solving and the value of iterative processes that allow for refinement as new information becomes available. This approach ultimately led to a successful resolution that was both effective and scalable.”

5. Detail a scenario where you utilized data analysis to solve a business problem.

In roles that demand strong problem-solving skills, the ability to use data analysis is crucial. You don’t just make decisions based on gut feelings or assumptions; instead, you seek out data, which can provide objective insights and guide you towards evidence-based solutions.

When responding to this question, outline a specific situation where you identified a problem that could be addressed with data. Walk through the steps you took to gather the appropriate data, how you analyzed it, and what tools or methods you used. Explain the conclusions you drew from the data and how you formulated a plan based on your analysis. Finally, discuss the outcome of implementing your solution, including any measurable results that underscored the success of your data-driven approach. It’s important to be concise, use clear examples, and quantify your impact if possible.

Example: “ In one scenario, I identified a recurring issue with inventory shortages that were impacting customer satisfaction and sales. Recognizing that a data-driven approach could uncover the root cause, I gathered historical sales, inventory levels, and supply chain data. Using statistical analysis and predictive modeling in Python, I identified patterns indicating that demand forecasting was misaligned with actual sales trends.

After refining the forecasting model to incorporate real-time sales data and market trends, I implemented a more dynamic inventory management system. This solution reduced stockouts by 25% within the first quarter, directly boosting customer satisfaction scores by 15% and increasing sales by 10%. The success of this project was a testament to the power of leveraging data analysis to resolve complex business problems efficiently.”

6. In what ways have you leveraged cross-functional team expertise in problem-solving?

Harnessing the expertise of cross-functional teams brings together varied knowledge bases and skills, leading to more innovative and comprehensive solutions. Your understanding of the value of collaborative thinking and your adeptness at harnessing the strengths of different departments are what employers look for.

When responding to this question, highlight a specific instance where you engaged with a cross-functional team to address a complex issue. Explain the role you played in facilitating communication between departments, how you integrated different viewpoints, and the outcome of the collaborative effort. Be sure to emphasize your ability to listen, synthesize information, and navigate through potential conflicts to arrive at a successful resolution.

Example: “ In tackling a particularly complex project challenge, I orchestrated a series of cross-functional workshops that brought together experts from R&D, marketing, finance, and operations. Recognizing that each department had a unique perspective, I facilitated a structured brainstorming session to harness these diverse viewpoints. By employing techniques such as ’round-robin’ and ‘worst possible idea’ to stimulate creative thinking and encourage open communication, the team was able to break down silos and share insights that might not have surfaced in a more conventional meeting setup.

The synthesis of these insights led to a multifaceted solution that balanced technical feasibility with market appeal and financial viability. I played a pivotal role in ensuring that each department’s concerns were addressed and that their expertise was reflected in the final plan. This collaborative approach not only resolved the issue at hand but also set a precedent for cross-departmental cooperation, leading to a 15% reduction in project lead time and a significant improvement in interdepartmental relations.”

7. Share an instance where you resolved a conflict that stemmed from differing problem-solving methodologies.

Navigating through conflicts arising from diverse approaches to problem-solving is a skill that highlights your interpersonal skills, adaptability, and capacity for integrating various strategies to overcome obstacles.

When responding, it is essential to outline a specific situation clearly, emphasizing the differing approaches without placing blame. Describe the steps taken to understand each perspective, find common ground, and explain the process of crafting a solution that incorporated the strengths of each methodology. Conclude with the outcome, focusing on the positive results of the collaboration and what was learned from the experience.

Example: “ In one instance, I was part of a team where conflict arose due to a clash between a traditional, waterfall approach to project management and a more agile, iterative method. One faction was focused on extensive planning and linear execution, while the other advocated for a flexible, adaptive approach that could respond to changing requirements.

To resolve this, I initiated a dialogue to dissect the core objectives of the project, allowing each side to present their rationale. By actively listening and asking probing questions, I facilitated an understanding that both methodologies aimed to enhance efficiency and deliver quality results, albeit through different paths. I then proposed a hybrid strategy that combined thorough upfront planning for known variables with the agility to adapt to unforeseen changes. This compromise leveraged the predictability of the waterfall model and the flexibility of agile practices.

The outcome was a cohesive project plan that satisfied both parties and improved team synergy. The project was completed successfully, meeting its goals on time and within budget. The experience reinforced the value of integrating diverse problem-solving techniques to create innovative solutions and taught us the importance of flexibility and open communication in conflict resolution.”

8. How do you prioritize issues when faced with several problems at once?

Knowing how to triage issues based on urgency and impact is a critical aspect of effective problem-solving. It’s important to distinguish between what needs immediate attention and what can wait, ensuring that resources are allocated efficiently and effectively.

When responding, a candidate should demonstrate their analytical skills by outlining a clear and logical process for prioritization. This might include assessing the severity of each issue, considering the implications on stakeholders, evaluating the time-sensitivity, and determining the resources required to address each problem. Citing specific examples where they successfully applied such a strategy will provide concrete evidence of their problem-solving abilities.

Example: “ In prioritizing issues, I employ a systematic approach that begins with assessing the impact and urgency of each problem. I consider the potential consequences of not addressing each issue promptly, and I categorize them based on the severity of their outcomes. This is often in line with the Eisenhower Matrix, where tasks are evaluated in terms of urgency and importance.

Next, I evaluate the dependencies and interconnections between the problems, as solving one might alleviate others or, conversely, ignoring one might exacerbate another. I also take into account the resources at hand, including time, personnel, and financial constraints. This comprehensive assessment allows me to create a strategic plan of action that targets the most critical issues first, ensuring that efforts are efficiently allocated to mitigate risks and capitalize on opportunities for resolution. A practical application of this was when I successfully navigated a project with simultaneous deadlines, resource shortages, and stakeholder pressures by prioritizing tasks that were critical to the project’s milestones, thereby ensuring the project’s timely and successful completion.”

9. What is your process for testing and validating the effectiveness of a solution?

A systematic approach to problem-solving includes setting benchmarks, gathering data, and analyzing results to confirm a solution’s success or identify areas for improvement. This process is essential for ensuring that solutions work as intended.

When responding, outline a clear, step-by-step process that begins with understanding the problem and setting clear objectives for what a successful solution looks like. Describe how you gather data before and after implementing a solution, and how you analyze this information to evaluate success. Offer a real-world example if possible, explaining how you adjusted your approach based on the results you obtained. This demonstrates your ability to think critically and adaptively, showcasing your commitment to not just solving problems, but solving them effectively and efficiently.

Example: “ My process for testing and validating the effectiveness of a solution starts with a clear definition of the problem and the establishment of measurable objectives. I then develop a hypothesis for a potential solution and design an experiment or pilot to test this hypothesis, ensuring that the experiment is controlled and variables are accounted for. I collect baseline data to understand the current state and compare it with the data post-implementation to assess any changes.

After implementing the solution, I gather quantitative and qualitative data to evaluate its impact. This involves not only looking at the direct outcomes but also considering any unintended consequences or side effects. I use statistical analysis to determine the significance of the results, ensuring that the observed effects are due to the solution and not random variation.

For instance, in a project aimed at reducing customer service call times, I established key performance indicators, such as average handle time and customer satisfaction scores. After implementing a new call-routing system, I analyzed the data and found a reduction in handle time, but customer satisfaction had unexpectedly decreased. This prompted a secondary analysis that revealed the new system was too impersonal. I then iterated on the solution, adding a feature to personalize customer interactions, which ultimately led to improvements in both handle time and customer satisfaction. This example underscores the importance of continuous monitoring and willingness to refine solutions based on data-driven insights.”

10. Have you ever implemented a creative solution that was not initially well-received? How did you handle it?

Stepping outside the box and challenging the status quo can be met with resistance or skepticism. Your capacity to innovate under less than ideal conditions and your resilience, persuasion skills, and ability to navigate opposition are all put to the test.

When responding, articulate the situation that required a creative solution, emphasizing the challenges faced and the innovative thinking applied. Detail the steps taken to communicate the idea, gather feedback, and adapt the solution to address concerns. Highlight your commitment to the idea, your ability to listen and incorporate feedback, and the eventual outcomes, including any lessons learned or successes achieved from the implementation. This demonstrates your approach to innovation, conflict resolution, and your ability to lead through change.

Example: “ Yes, I encountered a situation where my creative solution faced initial resistance. The challenge was to streamline an outdated process that was deeply ingrained in the company’s operations. I proposed an automation strategy that would significantly reduce manual labor and error rates. Despite the clear benefits, the solution was met with skepticism due to the team’s comfort with the status quo and fear of job displacement.

To address the concerns, I initiated a series of discussions to openly communicate the vision and long-term benefits of the automation. I actively listened to the feedback and incorporated suggestions that aligned with the project goals, such as offering training for the affected employees to manage the new system. By demonstrating the value of the solution through a pilot program, showcasing improved efficiency and accuracy, the team gradually accepted the change. The successful implementation not only optimized operations but also fostered a culture more receptive to innovation. This experience reinforced the importance of empathy and engagement when driving change.”

11. Illustrate how you maintain objectivity when solving emotionally-charged problems.

Remaining impartial and avoiding letting personal feelings or biases influence the decision-making process is a sign of strong problem-solving skills. Emotional intelligence is a vital component here, as it allows you to navigate emotionally-charged situations with a clear head.

To respond effectively, showcase a methodical approach to problem-solving that includes gathering information, weighing options, considering the consequences, and consulting with relevant stakeholders if necessary. Provide a specific example where you successfully handled an emotionally-charged problem by focusing on the facts and the bigger picture, demonstrating your emotional intelligence and commitment to fairness. Emphasize your ability to stay calm, your use of strategies to mitigate emotional influences (such as taking a step back or seeking a second opinion), and your commitment to achieving the best outcome for all parties involved.

Example: “ In situations where emotions run high, I prioritize maintaining a clear, analytical mindset. For instance, when faced with a problem that stirred significant emotional response among team members, I initiated a structured problem-solving process. This began with gathering comprehensive data and insights to ensure decisions were grounded in facts rather than feelings. I then methodically evaluated the potential impact of various solutions, considering both short-term and long-term consequences for all stakeholders involved.

To ensure objectivity, I actively sought diverse perspectives, which included consulting with individuals both inside and outside the immediate team to counteract any emotional bias. By creating a decision matrix that quantified the pros and cons of each option, I was able to present a well-reasoned recommendation. This approach not only facilitated a fair resolution but also demonstrated my commitment to impartiality and rational decision-making, even when navigating the complexities of emotionally-charged situations.”

12. Tell us about a time when you had to convince stakeholders to adopt an unconventional problem-solving approach.

Thinking outside the box is often required when conventional methods fail to yield results. This question assesses whether you are not only innovative and confident in your problem-solving skills but also persuasive and adept at managing change.

When responding, it’s crucial to outline a clear scenario where you identified a unique problem that warranted an unorthodox solution. Walk the interviewer through your thought process, emphasizing how you evaluated the risks and benefits of your approach. Describe the steps you took to gain buy-in from stakeholders, such as leveraging data, aligning with overarching goals, or showcasing small wins. Conclude by sharing the impact of your solution, reinforcing how your ability to think creatively and communicate effectively led to a positive change within the organization.

Example: “ In a project where traditional strategies were failing to resolve a persistent quality control issue, I recognized that a radical shift in our approach was necessary. After a thorough analysis, I proposed the integration of a machine learning algorithm to predict and identify defects, which was unconventional within our industry at the time. Understanding the skepticism it might evoke, I conducted a small-scale pilot study, meticulously documenting the algorithm’s predictive accuracy compared to our existing methods.

To convince stakeholders, I presented a comprehensive comparison of the pilot results with our historical data, clearly demonstrating a significant reduction in defect rates and an increase in detection efficiency. I aligned my presentation with the company’s strategic objectives, highlighting potential cost savings, improved customer satisfaction, and a strengthened market position. By focusing on tangible outcomes and strategic alignment, I was able to secure the necessary support to implement the solution organization-wide.

The adoption of this innovative approach not only resolved the quality issue but also positioned us as a forward-thinking leader in our field. The success of this initiative was evident in the enhanced product quality, reduced waste, and positive feedback from clients, which ultimately contributed to an increase in market share. This experience underscored the value of embracing unconventional problem-solving techniques and the importance of effective stakeholder communication in driving organizational innovation.”

13. How do you balance speed and accuracy when resolving urgent issues?

Maintaining a delicate equilibrium between rapid response and meticulous attention to detail is a reflection of effective problem-solving skills. In high-stakes environments, the ability to act swiftly without sacrificing the quality of the work can be crucial.

When responding to this question, candidates should demonstrate their methodical approach to problem-solving. They could share a specific example where they successfully managed a time-sensitive issue, outlining the steps they took to assess the situation, prioritize actions, and ensure the accuracy of their work. It’s also beneficial to mention any tools or techniques employed to streamline the process, such as checklists or collaboration with team members, to maintain both speed and precision.

Example: “ Balancing speed and accuracy is a critical aspect of problem-solving, especially when dealing with urgent issues. My approach is to first quickly assess the scope and impact of the problem to prioritize the necessary actions. I use a triage system to determine which aspects of the issue need immediate attention and which can be addressed after the immediate threat is mitigated. This allows for a rapid response without overlooking critical details that could lead to further complications.

Once priorities are set, I employ a combination of checklists and mental models to ensure accuracy while maintaining momentum. Checklists serve as a fail-safe to prevent oversight, while mental models like the OODA loop (Observe, Orient, Decide, Act) help me to stay focused and adapt to new information as it arises. In situations where collaboration is key, I leverage the expertise of team members, ensuring that we work in parallel to address different facets of the issue efficiently. This systematic yet flexible approach ensures that solutions are both swift and sound.”

14. What measures do you take to ensure long-term success rather than just quick fixes?

A strategic approach to problem-solving that encompasses both immediate and sustainable solutions is highly valued by employers. They look for candidates who demonstrate foresight that prioritizes long-term success and stability over temporary fixes.

When responding to this question, articulate your process for evaluating problems by considering the broader context and potential future implications. Discuss how you balance the need for immediate action with the goal of preventing recurrence. Share specific examples where you’ve successfully implemented a solution that addressed the root cause of a problem, and how you monitored its effectiveness over time. Highlight your commitment to continuous improvement and your understanding that true problem-solving is an ongoing process, not a one-time task.

Example: “ To ensure long-term success, I first conduct a thorough analysis to understand the underlying causes of a problem, rather than just addressing the symptoms. This involves gathering data, consulting with stakeholders, and examining the issue from multiple angles to identify systemic issues that could lead to recurrence. Once the root cause is identified, I develop a comprehensive solution that not only resolves the immediate problem but also strengthens the system against similar future challenges.

For instance, in a situation where a software application was experiencing repeated downtime, instead of just repeatedly patching it, I led a deep dive into the codebase and infrastructure. This revealed a scalability issue that was not immediately obvious. By redesigning a key component of the system to handle higher loads, we not only fixed the immediate crashes but also improved overall performance and reliability. To ensure the effectiveness of the solution, I implemented a monitoring system that provided real-time feedback and allowed for proactive adjustments, thus demonstrating a commitment to continuous improvement and long-term resilience.”

15. How has your problem-solving technique evolved over the course of your career?

With experience, exposure to various challenges, and the acquisition of knowledge, problem-solving skills mature. Your adaptability and growth in your approach to tackling issues reveal how you learn from past experiences and refine your methods.

When responding, it’s important to give concrete examples that illustrate a progression in your problem-solving abilities. Start with an early career challenge, describe the approach taken then, and contrast it with a more recent problem and the evolved techniques used to solve it. Highlight any specific lessons learned, mentors who influenced your methods, or training that contributed to your development. This not only shows your problem-solving growth but also demonstrates self-reflection and a commitment to professional development.

Example: “ Early in my career, my problem-solving approach was primarily reactive and often relied on conventional wisdom. For instance, when faced with a challenging project, I would tackle issues as they arose, using established methods without much anticipation of potential complications. This sometimes led to inefficiencies and a piecemeal resolution of problems.

As I gained experience, I began to adopt a more proactive and systematic approach. I learned the importance of defining the problem accurately, gathering data, and analyzing it before jumping to solutions. For example, when a complex issue presented itself recently, I utilized root cause analysis to identify the underlying factors contributing to the problem. This allowed me to develop a comprehensive strategy that addressed the core of the issue rather than just its symptoms. I also started incorporating cross-disciplinary techniques, such as design thinking and agile methodologies, to foster creative solutions and adapt quickly to changing circumstances. This evolution in my problem-solving technique has resulted in more innovative and sustainable outcomes, as well as a significant reduction in the time and resources required to resolve issues.”

16. Describe an occasion where you identified a potential problem before it occurred and preemptively addressed it.

Strategic thinking and foresight are hallmarks of recognizing and mitigating potential problems before they manifest. Your ability to analyze situations, anticipate challenges, and take proactive measures is what this question aims to uncover.

When responding, outline a specific situation where you noticed warning signs or patterns that suggested a future issue. Detail the steps you took to evaluate the risk and the actions you implemented to prevent the problem. Be sure to highlight your thought process, the resources you utilized, and the outcome of your intervention. This will demonstrate not only your problem-solving skills but also your ability to act decisively and effectively under potential pressure.

Example: “ On one occasion, I recognized that a critical project was at risk due to potential supply chain disruptions. By analyzing vendor delivery patterns and global market trends, I identified a pattern of delays that could severely impact our timeline. To mitigate this risk, I proactively engaged with alternative suppliers and renegotiated terms with existing vendors to ensure priority delivery. Additionally, I implemented a real-time tracking system for supply chain management, allowing for immediate response to any logistical hiccups.

This preemptive strategy not only averted a bottleneck in our project but also strengthened our vendor relationships and improved our overall supply chain resilience. The project was completed ahead of schedule with cost savings from the more competitive supplier terms. This experience underscored the importance of vigilance and adaptability in preemptive problem-solving.”

17. When dealing with unknown variables, how do you proceed in formulating a solution?

Across all industries, problem-solving skills demonstrate an individual’s ability to adapt and find solutions in uncertain situations. Your critical thinking and analytical abilities, initiative, resourcefulness, and creativity are all assessed through this question.

When responding to this question, outline a structured approach that begins with identifying what is known and what is not. Discuss how you gather additional information, break down the problem into manageable parts, and prioritize the variables that will have the most significant impact on the solution. Mention any tools or techniques you use, such as root cause analysis or brainstorming sessions, and how you weigh potential outcomes to make informed decisions. It’s also beneficial to provide a specific example of a past situation where you successfully navigated through unknowns to resolve a problem.

Example: “ When confronted with unknown variables, my approach is to first delineate the boundaries of what is known and leverage that as a foundation. I systematically categorize the unknowns based on their potential impact and relevance to the problem at hand, prioritizing them to efficiently direct my research and analysis efforts. I employ tools such as root cause analysis to drill down into the underlying issues and utilize brainstorming sessions with stakeholders to explore diverse perspectives and solutions.

In a past situation, I was faced with a complex problem where the cause was obscured by several interacting unknown variables. By applying a combination of Pareto analysis and iterative hypothesis testing, I was able to isolate the most significant factors. This methodical approach allowed me to not only identify the root cause but also to develop a targeted solution that addressed the core of the problem rather than just its symptoms. The result was a sustainable resolution that prevented recurrence and optimized system performance.”

18. Can you recount a project where iterative problem-solving was crucial to success?

Recognizing when a problem is complex enough to require iterative techniques is a sign of effective problem-solving. This question delves into your endurance, adaptability, and analytical skills, as well as your willingness to engage in continuous improvement.

When responding, candidates should select a project that had multiple layers of complexity, requiring them to apply an iterative approach. It’s important to describe the initial problem, the steps taken to address it, and how feedback or results from each phase guided subsequent actions. Highlighting collaboration with team members and how their input shaped the evolving solution can demonstrate both teamwork and leadership skills. Remember to articulate the lessons learned and how the experience has honed your problem-solving abilities for future challenges.

Example: “ In a project aimed at optimizing a complex supply chain network, iterative problem-solving was essential due to the dynamic nature of logistics and varying market demands. The initial problem was a bottleneck in the distribution process, which led to delays and increased costs. The initial strategy involved analyzing the existing workflow and identifying key areas where efficiency could be improved. However, it quickly became apparent that the solution required a deeper, multi-stage approach.

The first iteration of the solution involved implementing a new inventory tracking system to provide real-time data on stock levels. While this improved visibility, it didn’t fully resolve the bottleneck. Feedback from this phase indicated that transportation scheduling was also a contributing factor. The second iteration focused on developing a more flexible scheduling system that could adapt to changing demands. This significantly reduced the delays, but there was still room for improvement. Further iterations involved cross-functional collaboration to refine the process, incorporating insights from the sales, procurement, and warehouse teams to align all aspects of the supply chain.

Each phase of the solution was informed by the results and feedback of the previous one, leading to a comprehensive and adaptive system that significantly improved overall efficiency. This iterative approach not only resolved the immediate issue but also provided a framework for continuous improvement in the supply chain. The experience underscored the importance of flexibility, cross-functional collaboration, and the willingness to adapt solutions based on iterative feedback, all of which have become integral to my problem-solving toolkit.”

19. What role does customer feedback play in your problem-solving strategy?

Customer feedback is critical in understanding what is working well and what needs improvement from the user’s perspective. How you value direct input from users and integrate that information into a responsive and adaptive problem-solving process is what organizations look for.

To respond effectively, candidates should articulate a structured approach that showcases active listening, critical analysis of feedback, and integration of actionable insights into problem-solving efforts. They should give examples of how they have used customer feedback to identify the root cause of a problem, generate solutions, and measure the effectiveness of those solutions post-implementation. Additionally, candidates can discuss how they maintain a feedback loop with customers to ensure continuous improvement and customer satisfaction.

Example: “ Customer feedback is integral to my problem-solving strategy as it provides direct insight into the user experience and highlights areas that may not align with our intended outcomes. By actively listening to feedback, I can identify patterns and specific pain points that customers face. This information is critical for root cause analysis, allowing me to pinpoint the underlying issues rather than just addressing surface-level symptoms.

Incorporating customer feedback, I prioritize issues based on their impact and frequency, developing solutions that are both effective and efficient. Post-implementation, I leverage feedback to assess the success of the solution, ensuring it meets customer expectations and resolves the identified problems. This creates a feedback loop that fosters continuous improvement and demonstrates to customers that their input is valued and drives change, ultimately enhancing customer satisfaction and loyalty.”

20. How do you distinguish between symptoms and underlying causes when analyzing a problem?

Differentiating between what is immediately observable and the root issues that trigger these manifestations denotes a person’s analytical prowess and capacity for critical thinking. Your methodology in dissecting issues is tested, ensuring long-term solutions rather than temporary fixes.

When responding, it’s essential to articulate a structured approach: start by describing how you gather and analyze information to identify patterns or commonalities that may point to underlying causes. Provide examples from past experiences where you successfully identified the root of a problem, perhaps by using specific tools or frameworks like the “5 Whys” method. Emphasize your attention to detail and your persistence in exploring beyond the obvious, highlighting how your approach led to effective and sustainable solutions.

Top 20 Autonomy Interview Questions & Answers

Top 20 dental hygiene interview questions & answers, you may also be interested in..., top 20 quantitative trading interview questions & answers, top 20 manufacturing process interview questions & answers, top 20 organizational culture interview questions & answers, top 20 spreadsheet interview questions & answers.

Homebuilding Executive Recruiting Search Firm

Top Answers for Problem Solving Interview Questions

answer in problem solving

Conquer the Interview: Master the Art of Problem-Solving with MatchBuilt

Interviews can be unpredictable, but your problem-solving responses don’t have to be. At MatchBuilt, we’re not just about finding jobs—we’re about empowering you to navigate the toughest interview questions with confidence and poise.

Navigate Your Interview With Precision:

  • Unpack the Challenge: Learn to identify and articulate the problem with finesse.
  • Craft Your Methodology: Develop a clear, logical approach to potential solutions.
  • Lead with Solutions: Share your thought process and the steps to your successful outcome.
  • Soft Skills Spotlight: Balance your technical savvy with the soft skills that showcase you as a complete candidate.

As the founder of MatchBuilt, I’ve seen how problem-solving skills can make or break an interview. They’re the litmus test of your practical know-how and your ability to collaborate under pressure.

Why Entrust Your Career to MatchBuilt?

  • Depth of Experience: Insights honed from years of career coaching and talent development.
  • Industry Acumen: Advice that’s informed by the realities of today’s job landscape.
  • Tactical Expertise: Strategies that have placed numerous candidates in roles where they thrive.

Dive into this guide to turn every problem-solving question from a hurdle into a highlight of your interview. Join me, and let MatchBuilt illuminate the path to your next career milestone.

Strategies for Answering Problem Solving Interview Questions

In order to effectively answer problem-solving interview questions, candidates must demonstrate their critical thinking skills and ability to approach unexpected challenges. Here are some strategies to help candidates navigate these types of questions:

Understand the Problem

The first step in solving any problem is to understand it fully. Candidates should listen carefully to the hiring manager’s description of the situation and ask questions to clarify any uncertainties. This will help them identify potential problems and determine the best way to approach the situation.

Brainstorm and Analyze Options

Once the problem is clear, candidates should brainstorm possible solutions and analyze the pros and cons of each option. They can draw from their past experiences to identify similar situations they’ve faced and the solutions they implemented. This demonstrates their ability to think critically and come up with new ideas .

Implementation

After evaluating their options, candidates must make a decision and explain the reasoning behind it. It’s important to emphasize how they would implement their chosen solution, taking into account potential obstacles and risks. This demonstrates their ability to make informed decisions and execute plans effectively.

Emphasize Soft Skills

In addition to technical skills, hiring managers are often interested in a candidate’s soft skills . This includes communication, teamwork, adaptability, and problem-solving abilities . Candidates should showcase how their soft skills complement their technical abilities and help them handle unexpected challenges.

Candidates can use the STAR method (Situation, Task, Action, Result) to structure their answers and clearly and concisely explain their experience . By using this method, they can provide a sample answer that effectively demonstrates their approach to problem-solving.

In summary, candidates can effectively showcase their problem-solving abilities during the hiring process by understanding the problem, brainstorming and analyzing options, making informed decisions, and emphasizing soft skills. By preparing for various types of questions and drawing from past experiences, candidates can provide specific and effective example answers that demonstrate their ability to solve potential problems in a specific role.

using star method to answer problem solving questions

Top 8 Problem Solving Interview Questions and Example Answers

1. describe a situation when you faced a difficult problem at work and how you solved it.

Hiring managers ask this question to understand how candidates handle unexpected challenges and difficult situations. An example answer to this question could be:

In my previous job as a software engineer, we encountered an unexpected challenge during a product launch. Our team had worked on a feature for months, but it wasn’t functioning as expected during the testing phase. After analyzing the code and conducting further tests, we realized that there was a compatibility issue with a third-party software we had integrated. To solve the problem, I led a cross-functional team to identify a workaround that would allow us to launch the product on time. We prioritized features and made strategic decisions to ensure the most critical functionality worked smoothly. By collaborating closely with team members and thinking creatively, we overcame this challenge and delivered a successful product launch.

2. Explain a Time When You Had to Make a Decision Without All the Information You Needed

This question helps hiring managers evaluate candidates’ ability to make informed decisions under pressure. An example answer to this question could be:

In my previous role as a project manager, we were working on a tight deadline for a client deliverable. During the final stages, we encountered a technical issue that we hadn’t anticipated, and we realized that we didn’t have all the information we needed to solve the problem. I had to make a decision quickly to keep the project on track. I relied on my technical expertise and consulted with team members to gather as much information as possible. Based on the available information, I made a decision that allowed us to complete the project on time. Afterward, we conducted a post-mortem analysis to identify the root cause of the problem and make changes to our process to prevent similar issues in the future.

3. Tell Me About a Time When You Had to Work with Someone Who Was Difficult to Get Along With

This question helps hiring managers evaluate candidates’ ability to collaborate and handle conflict . An example answer to this question could be:

In my previous job, I worked on a cross-functional team with a colleague with a very different work style than mine. We had difficulty communicating effectively, which resulted in misaligned priorities and conflicting expectations. To solve the problem, I set up a one-on-one meeting with the colleague to discuss our differences and find common ground. We identified areas where we could collaborate more effectively and established a communication plan to prevent similar misunderstandings in the future. By taking the initiative to address the problem and finding a mutually beneficial solution, we were able to work together more effectively and achieve our goals.

4. Describe a Time When You Had to Analyze Information and Make a Recommendation

This question helps hiring managers evaluate candidates’ analytical skills and ability to make informed recommendations . An example answer to this question could be:

In my previous role as a financial analyst, I was tasked with analyzing a large data set and making a recommendation on a new investment opportunity. I conducted extensive research and analysis, taking into account market trends, industry benchmarks, and financial projections. Based on my findings, I recommended that we pursue the investment opportunity, highlighting the potential for high returns and the alignment with our company’s long-term strategy. The executive team ultimately approved my recommendation, and the investment turned out to be very successful.

job interview questions about challenging problems

5. Tell Me About a Time When You Had to Solve a Problem with Limited Resources

This question is often asked to test a candidate’s ability to work under pressure and come up with creative solutions. One example answer could be:

In my previous role, we had a tight budget and a deadline to launch a new product. Our team was struggling to come up with a cost-effective solution. I suggested we contact local universities to see if any students were interested in working on the project as part of their coursework. We were able to find a group of talented students who worked on the project for a fraction of the cost and delivered exceptional results on time.

6. Explain How You Would Approach a Problem That You Have Never Encountered Before

This question assesses a candidate’s critical thinking and problem-solving skills. An example answer could be:

If I encountered a problem I’d never seen before; my first step would be gathering as much information as possible. I would research the problem and try to understand its root cause. Then, I would brainstorm possible solutions and evaluate each one’s feasibility and potential impact. Finally, I would seek input from other team members and stakeholders to make an informed decision.

7. Describe a Time When You Had to Manage a Crisis Situation

This question tests a candidate’s ability to remain calm and composed in high-pressure situations. An example answer could be:

During my time as a project manager, we experienced a sudden system outage that lasted for several hours. I immediately gathered my team and assigned specific tasks to each member to investigate and resolve the issue. I also communicated regularly with our clients, keeping them informed of our progress and estimated timeline for resolution. By working together and maintaining open communication, we resolved the issue and minimized its impact on our clients.

8. Explain a Time When You Identified a Problem Before Anyone Else and Took Steps to Solve It

This question tests a candidate’s proactive thinking and initiative. An example answer could be:

In my previous role, I noticed that our team was spending excessive time on manual data entry. I researched possible solutions and presented a proposal to my supervisor, suggesting we invest in automation software. After gaining approval, I worked with our IT team to implement the new software, which significantly increased efficiency and productivity for our team.

how to answer problem solving interview questions

Problem-Solving Interview Questions: Your Top FAQs Answered

As problem-solving skills are highly valued in most job roles, it’s no surprise that problem-solving interview questions are common in job interviews. However, these types of questions can be challenging to answer effectively, leaving job seekers with a lot of questions. In this section, we’ll address some of the most frequently asked questions about problem-solving interview questions and provide answers to help candidates confidently navigate these types of questions.

Interviewers ask problem-solving questions to assess candidates' ability to think critically, handle challenges, and develop creative solutions. These skills are often essential in the workplace, and employers want to ensure they hire someone who can contribute to the company's success.

Candidates may find it challenging to come up with specific examples that showcase their problem-solving skills. Additionally, they may struggle to articulate their thought process or communicate their solutions effectively.

The best way to answer problem-solving questions is to use a structured approach. Begin by clarifying the problem, brainstorming possible solutions, evaluating each option, and selecting the best solution. It's also essential to communicate your process and reasoning effectively.

You can expect a range of problem-solving questions in a job interview, from general questions about past experiences to hypothetical scenarios that test your critical thinking skills. Examples may include questions about difficult situations you've faced, how you've solved problems with limited resources, or how you approach problems you've never encountered before.

To prepare for problem-solving questions, review the job description and identify any skills or experiences that are essential for the role. Think of specific examples from your past experiences that showcase your problem-solving skills. Practice using the STAR method to structure your answers and communicate your thought process clearly.

It's okay if you don't know the answer to a problem-solving question. Instead, focus on your approach to problem-solving and how you would go about finding a solution. Employers are often more interested in how you think than whether you have all the answers.

If you don't have work experience, think of examples from your academic or personal life that showcase your problem-solving skills. For example, you may have solved a complex problem in a group project, volunteered for a community organization, or participated in a hackathon.

Problem-solving questions can be used for all types of jobs, from entry-level to executive positions. Employers want to ensure that their employees can handle challenges and come up with innovative solutions regardless of their job titles or responsibilities.

best problem solving answers

Mastering Problem-Solving Interview Questions: Key Takeaways for Job Seekers

Problem-solving interview questions provide hiring managers with valuable insights into a candidate’s problem-solving skills and ability to handle challenging situations. As a candidate, answering these questions with a specific example of a time when you faced a potential problem and describing your problem-solving process can give you a good chance of impressing the hiring manager.

While technical interviews often focus on a candidate’s technical abilities, behavioral questions, including problem-solving questions, provide insight into a candidate’s approach and experience with relevant work-related problems. By emphasizing your problem-solving process and relevant information, you can demonstrate to the hiring manager that you are among the right candidates for the job.

At MatchBuilt Executive Search, we understand the importance of problem-solving interview questions in hiring. Our expertise in helping clients find candidates with the right skills and experience has given us a unique perspective on the best way to approach these questions. By providing innovative and creative solutions to unexpected challenges, our candidates have a better chance of succeeding in the interview process and landing their dream job.

So, whether you are a hiring manager looking for the right candidates or a job seeker looking to showcase your problem-solving skills, understanding the importance of problem-solving interview questions and how to approach them can make all the difference.

Looking for more career advice and tips? Check out our blog for articles on dealing with difficult coworkers, finding the best careers for your personality type , and being more assertive at work . We also cover industry-specific topics, such as the best-paying jobs in homebuilding . Browse our blog for insights and strategies that can help you achieve career success.

Mastering Problem-Solving Interview Questions: Expert Tips and Video Guides

answer in problem solving

About Mark Matyanowski

As the founder of MatchBuilt, with over 18 years of recruiting and coaching experience and 8+ years in executive roles at a leading Fortune 100 company, I am deeply committed to guiding professionals in their career paths.

Our team at MatchBuilt offers expert support in enhancing resumes, optimizing LinkedIn profiles, and preparing for interviews. Our blog, drawing on our rich experience and industry insights, is a valuable resource for job seekers.

We take pride in successfully guiding job candidates to top-tier company roles while empowering individuals to achieve their career ambitions, irrespective of their background or educational level.

Read more

How it works

Transform your enterprise with the scalable mindsets, skills, & behavior change that drive performance.

Explore how BetterUp connects to your core business systems.

We pair AI with the latest in human-centered coaching to drive powerful, lasting learning and behavior change.

Build leaders that accelerate team performance and engagement.

Unlock performance potential at scale with AI-powered curated growth journeys.

Build resilience, well-being and agility to drive performance across your entire enterprise.

Transform your business, starting with your sales leaders.

Unlock business impact from the top with executive coaching.

Foster a culture of inclusion and belonging.

Accelerate the performance and potential of your agencies and employees.

See how innovative organizations use BetterUp to build a thriving workforce.

Discover how BetterUp measurably impacts key business outcomes for organizations like yours.

A demo is the first step to transforming your business. Meet with us to develop a plan for attaining your goals.

Request a demo

  • What is coaching?

Learn how 1:1 coaching works, who its for, and if it's right for you.

Accelerate your personal and professional growth with the expert guidance of a BetterUp Coach.

Types of Coaching

Navigate career transitions, accelerate your professional growth, and achieve your career goals with expert coaching.

Enhance your communication skills for better personal and professional relationships, with tailored coaching that focuses on your needs.

Find balance, resilience, and well-being in all areas of your life with holistic coaching designed to empower you.

Discover your perfect match : Take our 5-minute assessment and let us pair you with one of our top Coaches tailored just for you.

Find your Coach

Research, expert insights, and resources to develop courageous leaders within your organization.

Best practices, research, and tools to fuel individual and business growth.

View on-demand BetterUp events and learn about upcoming live discussions.

The latest insights and ideas for building a high-performing workplace.

  • BetterUp Briefing

The online magazine that helps you understand tomorrow's workforce trends, today.

Innovative research featured in peer-reviewed journals, press, and more.

Founded in 2022 to deepen the understanding of the intersection of well-being, purpose, and performance

We're on a mission to help everyone live with clarity, purpose, and passion.

Join us and create impactful change.

Read the buzz about BetterUp.

Meet the leadership that's passionate about empowering your workforce.

For Business

For Individuals

10 Problem-solving strategies to turn challenges on their head

Find my Coach

Jump to section

What is an example of problem-solving?

What are the 5 steps to problem-solving, 10 effective problem-solving strategies, what skills do efficient problem solvers have, how to improve your problem-solving skills.

Problems come in all shapes and sizes — from workplace conflict to budget cuts.

Creative problem-solving is one of the most in-demand skills in all roles and industries. It can boost an organization’s human capital and give it a competitive edge. 

Problem-solving strategies are ways of approaching problems that can help you look beyond the obvious answers and find the best solution to your problem . 

Let’s take a look at a five-step problem-solving process and how to combine it with proven problem-solving strategies. This will give you the tools and skills to solve even your most complex problems.

Good problem-solving is an essential part of the decision-making process . To see what a problem-solving process might look like in real life, let’s take a common problem for SaaS brands — decreasing customer churn rates.

To solve this problem, the company must first identify it. In this case, the problem is that the churn rate is too high. 

Next, they need to identify the root causes of the problem. This could be anything from their customer service experience to their email marketing campaigns. If there are several problems, they will need a separate problem-solving process for each one. 

Let’s say the problem is with email marketing — they’re not nurturing existing customers. Now that they’ve identified the problem, they can start using problem-solving strategies to look for solutions. 

This might look like coming up with special offers, discounts, or bonuses for existing customers. They need to find ways to remind them to use their products and services while providing added value. This will encourage customers to keep paying their monthly subscriptions.

They might also want to add incentives, such as access to a premium service at no extra cost after 12 months of membership. They could publish blog posts that help their customers solve common problems and share them as an email newsletter.

The company should set targets and a time frame in which to achieve them. This will allow leaders to measure progress and identify which actions yield the best results.

team-meeting-problem-solving-strategies

Perhaps you’ve got a problem you need to tackle. Or maybe you want to be prepared the next time one arises. Either way, it’s a good idea to get familiar with the five steps of problem-solving. 

Use this step-by-step problem-solving method with the strategies in the following section to find possible solutions to your problem.

1. Identify the problem

The first step is to know which problem you need to solve. Then, you need to find the root cause of the problem. 

The best course of action is to gather as much data as possible, speak to the people involved, and separate facts from opinions. 

Once this is done, formulate a statement that describes the problem. Use rational persuasion to make sure your team agrees .

2. Break the problem down 

Identifying the problem allows you to see which steps need to be taken to solve it. 

First, break the problem down into achievable blocks. Then, use strategic planning to set a time frame in which to solve the problem and establish a timeline for the completion of each stage.

3. Generate potential solutions

At this stage, the aim isn’t to evaluate possible solutions but to generate as many ideas as possible. 

Encourage your team to use creative thinking and be patient — the best solution may not be the first or most obvious one.

Use one or more of the different strategies in the following section to help come up with solutions — the more creative, the better.

4. Evaluate the possible solutions

Once you’ve generated potential solutions, narrow them down to a shortlist. Then, evaluate the options on your shortlist. 

There are usually many factors to consider. So when evaluating a solution, ask yourself the following questions:

  • Will my team be on board with the proposition?
  • Does the solution align with organizational goals ?
  • Is the solution likely to achieve the desired outcomes?
  • Is the solution realistic and possible with current resources and constraints?
  • Will the solution solve the problem without causing additional unintended problems?

woman-helping-her-colleague-problem-solving-strategies

5. Implement and monitor the solutions

Once you’ve identified your solution and got buy-in from your team, it’s time to implement it. 

But the work doesn’t stop there. You need to monitor your solution to see whether it actually solves your problem. 

Request regular feedback from the team members involved and have a monitoring and evaluation plan in place to measure progress.

If the solution doesn’t achieve your desired results, start this step-by-step process again.

There are many different ways to approach problem-solving. Each is suitable for different types of problems. 

The most appropriate problem-solving techniques will depend on your specific problem. You may need to experiment with several strategies before you find a workable solution.

Here are 10 effective problem-solving strategies for you to try:

  • Use a solution that worked before
  • Brainstorming
  • Work backward
  • Use the Kipling method
  • Draw the problem
  • Use trial and error
  • Sleep on it
  • Get advice from your peers
  • Use the Pareto principle
  • Add successful solutions to your toolkit

Let’s break each of these down.

1. Use a solution that worked before

It might seem obvious, but if you’ve faced similar problems in the past, look back to what worked then. See if any of the solutions could apply to your current situation and, if so, replicate them.

2. Brainstorming

The more people you enlist to help solve the problem, the more potential solutions you can come up with.

Use different brainstorming techniques to workshop potential solutions with your team. They’ll likely bring something you haven’t thought of to the table.

3. Work backward

Working backward is a way to reverse engineer your problem. Imagine your problem has been solved, and make that the starting point.

Then, retrace your steps back to where you are now. This can help you see which course of action may be most effective.

4. Use the Kipling method

This is a method that poses six questions based on Rudyard Kipling’s poem, “ I Keep Six Honest Serving Men .” 

  • What is the problem?
  • Why is the problem important?
  • When did the problem arise, and when does it need to be solved?
  • How did the problem happen?
  • Where is the problem occurring?
  • Who does the problem affect?

Answering these questions can help you identify possible solutions.

5. Draw the problem

Sometimes it can be difficult to visualize all the components and moving parts of a problem and its solution. Drawing a diagram can help.

This technique is particularly helpful for solving process-related problems. For example, a product development team might want to decrease the time they take to fix bugs and create new iterations. Drawing the processes involved can help you see where improvements can be made.

woman-drawing-mind-map-problem-solving-strategies

6. Use trial-and-error

A trial-and-error approach can be useful when you have several possible solutions and want to test them to see which one works best.

7. Sleep on it

Finding the best solution to a problem is a process. Remember to take breaks and get enough rest . Sometimes, a walk around the block can bring inspiration, but you should sleep on it if possible.

A good night’s sleep helps us find creative solutions to problems. This is because when you sleep, your brain sorts through the day’s events and stores them as memories. This enables you to process your ideas at a subconscious level. 

If possible, give yourself a few days to develop and analyze possible solutions. You may find you have greater clarity after sleeping on it. Your mind will also be fresh, so you’ll be able to make better decisions.

8. Get advice from your peers

Getting input from a group of people can help you find solutions you may not have thought of on your own. 

For solo entrepreneurs or freelancers, this might look like hiring a coach or mentor or joining a mastermind group. 

For leaders , it might be consulting other members of the leadership team or working with a business coach .

It’s important to recognize you might not have all the skills, experience, or knowledge necessary to find a solution alone. 

9. Use the Pareto principle

The Pareto principle — also known as the 80/20 rule — can help you identify possible root causes and potential solutions for your problems.

Although it’s not a mathematical law, it’s a principle found throughout many aspects of business and life. For example, 20% of the sales reps in a company might close 80% of the sales. 

You may be able to narrow down the causes of your problem by applying the Pareto principle. This can also help you identify the most appropriate solutions.

10. Add successful solutions to your toolkit

Every situation is different, and the same solutions might not always work. But by keeping a record of successful problem-solving strategies, you can build up a solutions toolkit. 

These solutions may be applicable to future problems. Even if not, they may save you some of the time and work needed to come up with a new solution.

three-colleagues-looking-at-computer-problem-solving-strategies

Improving problem-solving skills is essential for professional development — both yours and your team’s. Here are some of the key skills of effective problem solvers:

  • Critical thinking and analytical skills
  • Communication skills , including active listening
  • Decision-making
  • Planning and prioritization
  • Emotional intelligence , including empathy and emotional regulation
  • Time management
  • Data analysis
  • Research skills
  • Project management

And they see problems as opportunities. Everyone is born with problem-solving skills. But accessing these abilities depends on how we view problems. Effective problem-solvers see problems as opportunities to learn and improve.

Ready to work on your problem-solving abilities? Get started with these seven tips.

1. Build your problem-solving skills

One of the best ways to improve your problem-solving skills is to learn from experts. Consider enrolling in organizational training , shadowing a mentor , or working with a coach .

2. Practice

Practice using your new problem-solving skills by applying them to smaller problems you might encounter in your daily life. 

Alternatively, imagine problematic scenarios that might arise at work and use problem-solving strategies to find hypothetical solutions.

3. Don’t try to find a solution right away

Often, the first solution you think of to solve a problem isn’t the most appropriate or effective.

Instead of thinking on the spot, give yourself time and use one or more of the problem-solving strategies above to activate your creative thinking. 

two-colleagues-talking-at-corporate-event-problem-solving-strategies

4. Ask for feedback

Receiving feedback is always important for learning and growth. Your perception of your problem-solving skills may be different from that of your colleagues. They can provide insights that help you improve. 

5. Learn new approaches and methodologies

There are entire books written about problem-solving methodologies if you want to take a deep dive into the subject. 

We recommend starting with “ Fixed — How to Perfect the Fine Art of Problem Solving ” by Amy E. Herman. 

6. Experiment

Tried-and-tested problem-solving techniques can be useful. However, they don’t teach you how to innovate and develop your own problem-solving approaches. 

Sometimes, an unconventional approach can lead to the development of a brilliant new idea or strategy. So don’t be afraid to suggest your most “out there” ideas.

7. Analyze the success of your competitors

Do you have competitors who have already solved the problem you’re facing? Look at what they did, and work backward to solve your own problem. 

For example, Netflix started in the 1990s as a DVD mail-rental company. Its main competitor at the time was Blockbuster. 

But when streaming became the norm in the early 2000s, both companies faced a crisis. Netflix innovated, unveiling its streaming service in 2007. 

If Blockbuster had followed Netflix’s example, it might have survived. Instead, it declared bankruptcy in 2010.

Use problem-solving strategies to uplevel your business

When facing a problem, it’s worth taking the time to find the right solution. 

Otherwise, we risk either running away from our problems or headlong into solutions. When we do this, we might miss out on other, better options.

Use the problem-solving strategies outlined above to find innovative solutions to your business’ most perplexing problems.

If you’re ready to take problem-solving to the next level, request a demo with BetterUp . Our expert coaches specialize in helping teams develop and implement strategies that work.

Boost your productivity

Maximize your time and productivity with strategies from our expert coaches.

Elizabeth Perry, ACC

Elizabeth Perry is a Coach Community Manager at BetterUp. She uses strategic engagement strategies to cultivate a learning community across a global network of Coaches through in-person and virtual experiences, technology-enabled platforms, and strategic coaching industry partnerships. With over 3 years of coaching experience and a certification in transformative leadership and life coaching from Sofia University, Elizabeth leverages transpersonal psychology expertise to help coaches and clients gain awareness of their behavioral and thought patterns, discover their purpose and passions, and elevate their potential. She is a lifelong student of psychology, personal growth, and human potential as well as an ICF-certified ACC transpersonal life and leadership Coach.

8 creative solutions to your most challenging problems

5 problem-solving questions to prepare you for your next interview, what are metacognitive skills examples in everyday life, 31 examples of problem solving performance review phrases, what is lateral thinking 7 techniques to encourage creative ideas, learn what process mapping is and how to create one (+ examples), leadership activities that encourage employee engagement, how much do distractions cost 8 effects of lack of focus, can dreams help you solve problems 6 ways to try, make the most of your time with the best time management tools, how to create a work plan (with template), 3 problem statement examples and steps to write your own, impression management: developing your self-presentation skills, 3 ways to solve your performance management problems, adjusting your vision for 2024, how to get permission for taking a sabbatical from work, the only guide you'll need to create effective cascading goals, 20 best productivity books and why you should read them, stay connected with betterup, get our newsletter, event invites, plus product insights and research..

3100 E 5th Street, Suite 350 Austin, TX 78702

  • Platform Overview
  • Integrations
  • Powered by AI
  • BetterUp Lead™
  • BetterUp Manage™
  • BetterUp Care®
  • Sales Performance
  • Diversity & Inclusion
  • Case Studies
  • Why BetterUp?
  • About Coaching
  • Find your Coach
  • Career Coaching
  • Communication Coaching
  • Personal Coaching
  • News and Press
  • Leadership Team
  • Become a BetterUp Coach
  • BetterUp Labs
  • Center for Purpose & Performance
  • Leadership Training
  • Business Coaching
  • Contact Support
  • Contact Sales
  • Privacy Policy
  • Acceptable Use Policy
  • Trust & Security
  • Cookie Preferences

LifeHack

Brain Power

5 steps (and 4 techniques) for effective problem solving.

' src=

Problem solving is the process of reviewing every element of an issue so you can get to a solution or fix it. Problem solving steps cover multiple aspects of a problem that you can bring together to find a solution. Whether that’s in a group collaboratively or independently, the process remains the same, but the approach and the steps can differ.

To find a problem solving approach that works for you, your team, or your company, you have to take into consideration the environment you’re in and the personalities around you.

Knowing the characters in the room will help you decide on the best approach to try and ultimately get to the best solution.

Table of Contents

5 problem solving steps, 4 techniques to encourage problem solving, the bottom line.

No matter what the problem is, to solve it, you nearly always have to follow these problem solving steps. Missing any of these steps can cause the problem to either resurface or the solution to not be implemented correctly.

Once you know these steps, you can then get creative with the approach you take to find the solutions you need.

1. Define the Problem

You must define and understand the problem before you start, whether you’re solving it independently or as a group. If you don’t have a single view of what the problem is, you could be fixing something that doesn’t need fixing, or you’ll fix the wrong problem.

Spend time elaborating on the problem, write it down, and discuss everything, so you’re clear on why the problem is occurring and who it is impacting.

Once you have clarity on the problem, you then need to start thinking about every possible solution . This is where you go big and broad, as you want to come up with as many alternative solutions as possible. Don’t just take the first idea; build out as many as you can through active listening, as the more you create, the more likely you’ll find a solution that has the best impact on the team.

3. Decide on a Solution

Whichever solution you pick individually or as a team, make sure you think about the impact on others if you implement this solution. Ask questions like:

  • How will they react to this change?
  • Will they need to change anything?
  • Who do we need to inform of this change?

4. Implement the Solution

At this stage of problem solving, be prepared for feedback, and plan for this. When you roll out the solution, request feedback on the success of the change made.

5. Review, Iterate, and Improve

Making a change shouldn’t be a one time action. Spend time reviewing the results of the change to make sure it’s made the required impact and met the desired outcomes.

Make changes where needed so you can further improve the solution implemented.

Each individual or team is going to have different needs and may need a different technique to encourage each of the problem solving steps. Try one of these to stimulate the process.

1-2-4 All Approach + Voting

The 1-2-4-All is a good problem solving approach that can work no matter how large the group is. Everyone is involved, and you can generate a vast amount of ideas quickly.

Ideas and solutions are discussed and organized rapidly, and what is great about this approach is the attendees own their ideas, so when it comes to implementing the solutions, you don’t have more work to gain buy-in.

As a facilitator, you first need to present the group with a question explaining the problem or situation. For example, “What actions or ideas would you recommend to solve the company’s lack of quiet working areas?”

With the question clear for all to see, the group then spends 5 minutes to reflect on the question individually. They can jot down their thoughts and ideas on Post-Its.

Now ask the participants to find one or two other people to discuss their ideas and thoughts with. Ask the group to move around to find a partner so they can mix with new people.

Ask the pairs to spend 5 minutes discussing their shared ideas and thoughts.

Next, put the group into groups of two or three pairs to make groups of 4-6. Each group shouldn’t be larger than six as the chances of everyone being able to speak reduces.

Ask the group to discuss one interesting idea they’ve heard in previous rounds, and each group member shares one each.

The group then needs to pick their preferred solution to the problem. This doesn’t have to be voted on, just one that resonated most with the group.

Then ask for three actions that could be taken to implement this change.

Bring everyone back together as a group and ask open questions like “What is the one thing you discussed that stood out for you?” or “Is there something you now see differently following these discussions?”

By the end of the session, you’ll have multiple approaches to solve the problem, and the whole group will have contributed to the future solutions and improvements.

The Lightning Decision Jam

The Lightning Decision Jam is a great way to solve problems collaboratively and agree on one solution or experiment you want to try straight away. It encourages team decision making, but at the same time, the individual can get their ideas and feedback across. [1]

If, as a team, you have a particular area you want to improve upon, like the office environment, for example, this approach is perfect to incorporate in the problem solving steps.

The approach follows a simple loop.

Make a Note – Stick It on The Wall – Vote – Prioritize

Using sticky notes, the technique identifies major problems, encourages solutions, and opens the group up for discussion. It allows each team member to play an active role in identifying both problems and ways to solve them.

Mind Mapping

Mind mapping is a fantastic visual thinking tool that allows you to bring problems to life by building out the connections and visualizing the relationships that make up the problem.

You can use a mind map to quickly expand upon the problem and give yourself the full picture of the causes of the problem, as well as solutions [2] .

Problem Solving with Mind Maps (Tutorial) - Focus

The goal of a mind map is to simplify the problem and link the causes and solutions to the problem.

To create a mind map, you must first create the central topic (level 1). In this case, that’s the problem.

Next, create the linked topics (level 2) that you place around and connect to the main central topic with a simple line.

If the central topic is “The client is always changing their mind at the last minute,” then you could have linked topics like:

  • How often does this happen?
  • Why are they doing this?
  • What are they asking for?
  • How do they ask for it?
  • What impact does this have?

Adding these linking topics allows you to start building out the main causes of the problem as you can begin to see the full picture of what you need to fix. Once you’re happy that you’ve covered the breadth of the problem and its issues, you can start to ideate on how you’re going to fix it with the problem solving steps.

Now, start adding subtopics (level 3) linking to each of the level 2 topics. This is where you can start to go big on solutions and ideas to help fix the problem.

For each of the linked topics (level 2), start to think about how you can prevent them, mitigate them, or improve them. As this is just ideas on paper, write down anything that comes to mind, even if you think the client will never agree to it!

The more you write down, the more ideas you’ll have until you find one or two that could solve the main problem.

Once you run out of ideas, take a step back and highlight your favorite solutions to take forward and implement.

The 5 Why’s

The five why’s can sound a little controversial, and you shouldn’t try this without prepping the team beforehand.

Asking “why” is a great way to go deep into the root of the problem to make the individual or team really think about the cause. When a problem arises, we often have preconceived ideas about why this problem has occurred, which is usually based on our experiences or beliefs.

Start with describing the problem, and then the facilitator can ask “Why?” fives time or more until you get to the root of the problem. It’s tough at first to keep being asked why, but it’s also satisfying when you get to the root of the problem [3] .

The 5 Whys

As a facilitator, although the basic approach is to ask why, you need to be careful not to guide the participant down a single route.

To help with this, you can use a mind map with the problem at the center. Then ask a why question that will result in multiple secondary topics around the central problem. Having this visual representation of the problem helps you build out more useful why questions around it.

Once you get to the root of the problem, don’t forget to be clear in the actions to put a fix in place to resolve it.

Learn more about how to use the five why’s here .

To fix a problem, you must first be in a position where you fully understand it. There are many ways to misinterpret a problem, and the best way to understand them is through conversation with the team or individuals who are experiencing it.

Once you’re aligned, you can then begin to work on the solutions that will have the greatest impact through effective problem solving steps.

For the more significant or difficult problems to solve, it’s often advisable to break the solution up into smaller actions or improvements.

Trial these improvements in short iterations, and then continue the conversations to review and improve the solution. Implementing all of these steps will help you root out the problems and find useful solutions each time.

[1]^UX Planet:
[2]^Focus:
[3]^Expert Program Management:

how to use a planner

How to Use a Planner Effectively

how to be a better planner

How to Be a Better Planner: Avoid the Planning Fallacy

delegation tools

5 Best Apps to Help You Delegate Tasks Easily

delegating leadership style

Delegating Leadership Style: What Is It & When To Use It?

hesitate to delegate

The Fear of Delegating Work To Others

importance of delegation in leadership

Why Is Delegation Important in Leadership?

best tools for prioritizing tasks

7 Best Tools for Prioritizing Work

how to deal with competing priorities

How to Deal with Competing Priorities Effectively

rice prioritization model

What Is the RICE Prioritization Model And How Does It Work?

exercises to improve focus

4 Exercises to Improve Your Focus

chronic procrastination

What Is Chronic Procrastination and How To Deal with It

procrastination adhd

How to Snap Out of Procrastination With ADHD

depression procrastination

Are Depression And Procrastination Connected?

procrastination and laziness

Procrastination And Laziness: Their Differences & Connections

bedtime procrastination

Bedtime Procrastination: Why You Do It And How To Break It

best books on procrastination

15 Books on Procrastination To Help You Start Taking Action

productive procrastination

Productive Procrastination: Is It Good or Bad?

how does procrastination affect productivity

The Impact of Procrastination on Productivity

anxiety and procrastination

How to Cope With Anxiety-Induced Procrastination

How to Break the Perfectionism-Procrastination Loop

How to Break the Perfectionism-Procrastination Loop

work life balance books

15 Work-Life Balance Books to Help You Take Control of Life

Work Life Balance for Women

Work Life Balance for Women: What It Means & How to Find It

career mindset

6 Essential Mindsets For Continuous Career Growth

career move

How to Discover Your Next Career Move Amid the Great Resignation

lee-cockerell

The Key to Creating a Vibrant (And Magical Life) by Lee Cockerell

how to disconnect from work

9 Tips on How To Disconnect From Work And Stay Present

work life integration VS balance

Work-Life Integration vs Work-Life Balance: Is One Better Than the Other?

self-advocacy in the workplace

How To Practice Self-Advocacy in the Workplace (Go-to Guide)

answer in problem solving

How to Boost Your Focus And Attention Span

answer in problem solving

What Are Distractions in a Nutshell?

answer in problem solving

What Is Procrastination And How To End It

answer in problem solving

Prioritization — Using Your Time & Energy Effectively

answer in problem solving

Delegation — Leveraging Your Time & Resources

answer in problem solving

Your Guide to Effective Planning & Scheduling

answer in problem solving

The Ultimate Guide to Achieving Goals

answer in problem solving

How to Find Lasting Motivation

answer in problem solving

Complete Guide to Getting Back Your Energy

answer in problem solving

How to Have a Good Life Balance

Explore the time flow system.

answer in problem solving

About the Time Flow System

answer in problem solving

Key Philosophy I: Fluid Progress, Like Water

answer in problem solving

Key Philosophy II: Pragmatic Priorities

answer in problem solving

Key Philosophy III: Sustainable Momentum

answer in problem solving

Key Philosophy IV: Three Goal Focus

answer in problem solving

How the Time Flow System Works

What Are Problem-Solving Skills? (Examples Included)

Mike Simpson 0 Comments

answer in problem solving

By Mike Simpson

Problem-solving skills are important not just for work. In the words of Karl Popper , “All life is problem-solving.”

What on earth does that mean? Simply that being alive means facing challenges. With problem-solving skills, you can navigate issues with greater ease, making hard times, well, less hard.

But what are problem-solving skills? How do you know if you have them or not? Why do they matter to your job search? And what should you do if you don’t feel yours are up to snuff? Luckily, we’re about to get into all of that.

If you’re curious about the world of problem-solving skills, here’s what you need to know.

What Are Problem-Solving Skills?

Before we dig into any examples, let’s focus first on an important question: what are problem-solving skills.

To answer that question, let’s start with the barebones basics. According to Merriam-Webster , problem-solving is “the process or act of finding a solution to a problem.” Why does that matter? Well, because it gives you insight into what problem-solving skills are.

Any skill that helps you find solutions to problems can qualify. And that means problem-solving skills aren’t just one capability, but a toolbox filled with soft skills and hard skills that come together during your time of need.

The ability to solve problems is relevant to any part of your life. Whether your writing a grocery list or dealing with a car that won’t start, you’re actually problem-solving.

The same is true at work, too. Most tasks actually involve a degree of problem-solving. Really? Really.

Think about it this way; when you’re given an assignment, you’re being asked, “Can you do this thing?” Doing that thing is the problem.

Then, you have to find a path that lets you accomplish what you need to do. That is problem-solving.

Yes, sometimes what you need to handle isn’t “challenging” in the difficulty sense. But that doesn’t mean it doesn’t count.

Besides, some of what you need to do will legitimately be hard. Maybe you’re given a new responsibility, or something goes wrong during a project. When that happens, you’ll have to navigate unfamiliar territory, gather new information, and think outside of the box. That’s problem-solving, too.

That’s why hiring managers favor candidates with problem-solving skills. They make you more effective in your role, increasing the odds that you can find solutions whenever the need arises.

How Are Problem-Solving Skills Relevant to a Job Search?

Alright, you probably have a good idea of what problem-solving skills are. Now, it’s time to talk about why they matter to your job search.

We’ve already touched on one major point: hiring managers prefer candidates with strong problem-solving skills. That alone makes these capabilities a relevant part of the equation. If you don’t show the hiring manager you’ve got what it takes to excel, you may struggle to land a position.

But that isn’t the only reason these skills matter. Problem-solving skills can help you during the entire job search process. After all, what’s a job search but a problem – or a series of problems – that needs an answer.

You need a new job; that’s the core problem you’re solving. But every step is its own unique challenge. Finding an opening that matches your skills, creating a resume that resonates with the hiring manager, nailing the interview, and negotiating a salary … those are all smaller problems that are part of the bigger one.

So, problem-solving skills really are at the core of the job search experience. By having strong capabilities in this area, you may find a new position faster than you’d expect.

Okay, you may be thinking, “If hiring managers prefer candidates with problem-solving skills, which ones are they after? Are certain problem-solving capabilities more important today? Is there something I should be going out of my way to showcase?”

While any related skills are worth highlighting, some may get you further than others. Analysis, research, creativity, collaboration , organization, and decision-making are all biggies. With those skills, you can work through the entire problem-solving process, making them worthwhile additions to your resume.

But that doesn’t mean you have to focus there solely. Don’t shy away from showcasing everything you bring to the table. That way, if a particular hiring manager is looking for a certain capability, you’re more likely to tap on what they’re after.

How to Highlight Problem-Solving Skills for Job Search

At this point, it’s ridiculously clear that problem-solving skills are valuable in the eyes of hiring managers. So, how do you show them that you’ve got all of the capabilities they are after? By using the right approach.

When you’re writing your resume or cover letter , your best bet is to highlight achievements that let you put your problem-solving skills to work. That way, you can “show” the hiring manager you have what it takes.

Showing is always better than telling. Anyone can write down, “I have awesome problem-solving skills.” The thing is, that doesn’t really prove that you do. With a great example, you offer up some context, and that makes a difference.

How do you decide on which skills to highlight on your resume or cover letter? By having a great strategy. With the Tailoring Method , it’s all about relevancy. The technique helps you identify skills that matter to that particular hiring manager, allowing you to speak directly to their needs.

Plus, you can use the Tailoring Method when you answer job interview questions . With that approach, you’re making sure those responses are on-point, too.

But when do you talk about your problem-solving capabilities during an interview? Well, there’s a good chance you’ll get asked problem-solving interview questions during your meeting. Take a look at those to see the kinds of questions that are perfect for mentioning these skills.

However, you don’t have to stop there. If you’re asked about your greatest achievement or your strengths, those could be opportunities, too. Nearly any open-ended question could be the right time to discuss those skills, so keep that in mind as you practice for your interview.

How to Develop Problem-Solving Skills If You Don’t Have Them

Developing problem-solving skills may seem a bit tricky on the surface, especially if you think you don’t have them. The thing is, it doesn’t actually have to be hard. You simply need to use the right strategy.

First, understand that you probably do have problem-solving skills; you simply may not have realized it. After all, life is full of challenges that you have to tackle, so there’s a good chance you’ve developed some abilities along the way.

Now, let’s reframe the question and focus on how to improve your problem-solving skills. Here’s how to go about it.

Understand the Problem-Solving Process

In many cases, problem-solving is all about the process. You:

  • Identify the problem
  • Analyze the key elements
  • Look for potential solutions
  • Examine the options for viability and risk
  • Decide on an approach
  • Review the outcome for lessons

By understanding the core process, you can apply it more effectively. That way, when you encounter an issue, you’ll know how to approach it, increasing the odds you’ll handle the situation effectively.

Try Puzzles and Games

Any activity that lets you take the steps listed above could help you hone your problem-solving skills. For example, brainteasers, puzzles, and logic-based games can be great places to start.

Whether it’s something as straightforward – but nonetheless challenging – as Sudoku or a Rubik’s Cube, or something as complex as Settlers of Catan, it puts your problem-solving skills to work. Plus, if you enjoy the activity, it makes skill-building fun, making it a win-win.

Look for Daily Opportunities

If you’re looking for a practical approach, you’re in luck. You can also look at the various challenges you face during the day and think about how to overcome them.

For example, if you always experience a mid-day energy slump that hurts your productivity, take a deep dive into that problem. Define what’s happening, think about why it occurs, consider various solutions, pick one to try, and analyze the results.

By using the problem-solving approach more often in your life, you’ll develop those skills further and make using these capabilities a habit. Plus, you may find ways to improve your day-to-day living, which is a nice bonus.

Volunteer for “Stretch” Projects

If you’re currently employed, volunteering for projects that push you slightly outside of your comfort zone can help you develop problem-solving skills, too. You’ll encounter the unknown and have to think outside of the box, both of which can boost critical problem-solving-related skills.

Plus, you may gain other capabilities along the way, like experience with new technologies or tools. That makes the project an even bigger career booster, which is pretty awesome.

List of Problem-Solving Skills

Alright, we’ve taken a pretty deep dive into what problem-solving skills are. Now, it’s time for some problem-solving skills examples.

As we mentioned above, there are a ton of capabilities and traits that can support better problem-solving. By understanding what they are, you can showcase the right abilities during your job search.

So, without further ado, here is a quick list of problem-solving skill examples:

  • Collaboration
  • Organization
  • Decision-Making
  • Troubleshooting
  • Self-Reliance
  • Self-Motivation
  • Communication
  • Attention to Detail
  • Brainstorming
  • Forecasting
  • Active Listening
  • Accountability
  • Open-Mindedness
  • Critical Thinking
  • Flexibility

Do you have to showcase all of those skills during your job search individually? No, not necessarily. Instead, you want to highlight a range of capabilities based on what the hiring manager is after. If you’re using the Tailoring Method, you’ll know which ones need to make their way into your resume, cover letter, and interview answers.

Now, are there other skills that support problem-solving? Yes, there certainly can be.

Essentially any skill that helps you go from the problem to the solution can, in its own right, be a problem-solving skill.

All of the skills above can be part of the equation. But, if you have another capability that helps you flourish when you encounter an obstacle, it can count, too.

Reflect on your past experience and consider how you’ve navigated challenges in the past. If a particular skill helped you do that, then it’s worth highlighting during a job search.

If you would like to find out more about skills to put on a resume , we’ve taken a close look at the topic before. Along with problem-solving skills, we dig into a variety of other areas, helping you choose what to highlight so that you can increase your odds of landing your perfect job.

Putting It All Together

Ultimately, problem-solving skills are essential for professionals in any kind of field. By honing your capabilities and showcasing them during your job search, you can become a stronger candidate and employee. In the end, that’s all good stuff, making it easier for you to keep your career on track today, tomorrow, and well into the future.

answer in problem solving

Co-Founder and CEO of TheInterviewGuys.com. Mike is a job interview and career expert and the head writer at TheInterviewGuys.com.

His advice and insights have been shared and featured by publications such as Forbes , Entrepreneur , CNBC and more as well as educational institutions such as the University of Michigan , Penn State , Northeastern and others.

Learn more about The Interview Guys on our About Us page .

About The Author

Mike simpson.

' src=

Co-Founder and CEO of TheInterviewGuys.com. Mike is a job interview and career expert and the head writer at TheInterviewGuys.com. His advice and insights have been shared and featured by publications such as Forbes , Entrepreneur , CNBC and more as well as educational institutions such as the University of Michigan , Penn State , Northeastern and others. Learn more about The Interview Guys on our About Us page .

Copyright © 2024 · TheInterviewguys.com · All Rights Reserved

  • Our Products
  • Case Studies
  • Interview Questions
  • Jobs Articles
  • Members Login

answer in problem solving

  • Bipolar Disorder
  • Therapy Center
  • When To See a Therapist
  • Types of Therapy
  • Best Online Therapy
  • Best Couples Therapy
  • Best Family Therapy
  • Managing Stress
  • Sleep and Dreaming
  • Understanding Emotions
  • Self-Improvement
  • Healthy Relationships
  • Student Resources
  • Personality Types
  • Guided Meditations
  • Verywell Mind Insights
  • 2024 Verywell Mind 25
  • Mental Health in the Classroom
  • Editorial Process
  • Meet Our Review Board
  • Crisis Support

Problem-Solving Strategies and Obstacles

JGI / Jamie Grill / Getty Images

  • Application
  • Improvement

From deciding what to eat for dinner to considering whether it's the right time to buy a house, problem-solving is a large part of our daily lives. Learn some of the problem-solving strategies that exist and how to use them in real life, along with ways to overcome obstacles that are making it harder to resolve the issues you face.

What Is Problem-Solving?

In cognitive psychology , the term 'problem-solving' refers to the mental process that people go through to discover, analyze, and solve problems.

A problem exists when there is a goal that we want to achieve but the process by which we will achieve it is not obvious to us. Put another way, there is something that we want to occur in our life, yet we are not immediately certain how to make it happen.

Maybe you want a better relationship with your spouse or another family member but you're not sure how to improve it. Or you want to start a business but are unsure what steps to take. Problem-solving helps you figure out how to achieve these desires.

The problem-solving process involves:

  • Discovery of the problem
  • Deciding to tackle the issue
  • Seeking to understand the problem more fully
  • Researching available options or solutions
  • Taking action to resolve the issue

Before problem-solving can occur, it is important to first understand the exact nature of the problem itself. If your understanding of the issue is faulty, your attempts to resolve it will also be incorrect or flawed.

Problem-Solving Mental Processes

Several mental processes are at work during problem-solving. Among them are:

  • Perceptually recognizing the problem
  • Representing the problem in memory
  • Considering relevant information that applies to the problem
  • Identifying different aspects of the problem
  • Labeling and describing the problem

Problem-Solving Strategies

There are many ways to go about solving a problem. Some of these strategies might be used on their own, or you may decide to employ multiple approaches when working to figure out and fix a problem.

An algorithm is a step-by-step procedure that, by following certain "rules" produces a solution. Algorithms are commonly used in mathematics to solve division or multiplication problems. But they can be used in other fields as well.

In psychology, algorithms can be used to help identify individuals with a greater risk of mental health issues. For instance, research suggests that certain algorithms might help us recognize children with an elevated risk of suicide or self-harm.

One benefit of algorithms is that they guarantee an accurate answer. However, they aren't always the best approach to problem-solving, in part because detecting patterns can be incredibly time-consuming.

There are also concerns when machine learning is involved—also known as artificial intelligence (AI)—such as whether they can accurately predict human behaviors.

Heuristics are shortcut strategies that people can use to solve a problem at hand. These "rule of thumb" approaches allow you to simplify complex problems, reducing the total number of possible solutions to a more manageable set.

If you find yourself sitting in a traffic jam, for example, you may quickly consider other routes, taking one to get moving once again. When shopping for a new car, you might think back to a prior experience when negotiating got you a lower price, then employ the same tactics.

While heuristics may be helpful when facing smaller issues, major decisions shouldn't necessarily be made using a shortcut approach. Heuristics also don't guarantee an effective solution, such as when trying to drive around a traffic jam only to find yourself on an equally crowded route.

Trial and Error

A trial-and-error approach to problem-solving involves trying a number of potential solutions to a particular issue, then ruling out those that do not work. If you're not sure whether to buy a shirt in blue or green, for instance, you may try on each before deciding which one to purchase.

This can be a good strategy to use if you have a limited number of solutions available. But if there are many different choices available, narrowing down the possible options using another problem-solving technique can be helpful before attempting trial and error.

In some cases, the solution to a problem can appear as a sudden insight. You are facing an issue in a relationship or your career when, out of nowhere, the solution appears in your mind and you know exactly what to do.

Insight can occur when the problem in front of you is similar to an issue that you've dealt with in the past. Although, you may not recognize what is occurring since the underlying mental processes that lead to insight often happen outside of conscious awareness .

Research indicates that insight is most likely to occur during times when you are alone—such as when going on a walk by yourself, when you're in the shower, or when lying in bed after waking up.

How to Apply Problem-Solving Strategies in Real Life

If you're facing a problem, you can implement one or more of these strategies to find a potential solution. Here's how to use them in real life:

  • Create a flow chart . If you have time, you can take advantage of the algorithm approach to problem-solving by sitting down and making a flow chart of each potential solution, its consequences, and what happens next.
  • Recall your past experiences . When a problem needs to be solved fairly quickly, heuristics may be a better approach. Think back to when you faced a similar issue, then use your knowledge and experience to choose the best option possible.
  • Start trying potential solutions . If your options are limited, start trying them one by one to see which solution is best for achieving your desired goal. If a particular solution doesn't work, move on to the next.
  • Take some time alone . Since insight is often achieved when you're alone, carve out time to be by yourself for a while. The answer to your problem may come to you, seemingly out of the blue, if you spend some time away from others.

Obstacles to Problem-Solving

Problem-solving is not a flawless process as there are a number of obstacles that can interfere with our ability to solve a problem quickly and efficiently. These obstacles include:

  • Assumptions: When dealing with a problem, people can make assumptions about the constraints and obstacles that prevent certain solutions. Thus, they may not even try some potential options.
  • Functional fixedness : This term refers to the tendency to view problems only in their customary manner. Functional fixedness prevents people from fully seeing all of the different options that might be available to find a solution.
  • Irrelevant or misleading information: When trying to solve a problem, it's important to distinguish between information that is relevant to the issue and irrelevant data that can lead to faulty solutions. The more complex the problem, the easier it is to focus on misleading or irrelevant information.
  • Mental set: A mental set is a tendency to only use solutions that have worked in the past rather than looking for alternative ideas. A mental set can work as a heuristic, making it a useful problem-solving tool. However, mental sets can also lead to inflexibility, making it more difficult to find effective solutions.

How to Improve Your Problem-Solving Skills

In the end, if your goal is to become a better problem-solver, it's helpful to remember that this is a process. Thus, if you want to improve your problem-solving skills, following these steps can help lead you to your solution:

  • Recognize that a problem exists . If you are facing a problem, there are generally signs. For instance, if you have a mental illness , you may experience excessive fear or sadness, mood changes, and changes in sleeping or eating habits. Recognizing these signs can help you realize that an issue exists.
  • Decide to solve the problem . Make a conscious decision to solve the issue at hand. Commit to yourself that you will go through the steps necessary to find a solution.
  • Seek to fully understand the issue . Analyze the problem you face, looking at it from all sides. If your problem is relationship-related, for instance, ask yourself how the other person may be interpreting the issue. You might also consider how your actions might be contributing to the situation.
  • Research potential options . Using the problem-solving strategies mentioned, research potential solutions. Make a list of options, then consider each one individually. What are some pros and cons of taking the available routes? What would you need to do to make them happen?
  • Take action . Select the best solution possible and take action. Action is one of the steps required for change . So, go through the motions needed to resolve the issue.
  • Try another option, if needed . If the solution you chose didn't work, don't give up. Either go through the problem-solving process again or simply try another option.

You can find a way to solve your problems as long as you keep working toward this goal—even if the best solution is simply to let go because no other good solution exists.

Sarathy V. Real world problem-solving .  Front Hum Neurosci . 2018;12:261. doi:10.3389/fnhum.2018.00261

Dunbar K. Problem solving . A Companion to Cognitive Science . 2017. doi:10.1002/9781405164535.ch20

Stewart SL, Celebre A, Hirdes JP, Poss JW. Risk of suicide and self-harm in kids: The development of an algorithm to identify high-risk individuals within the children's mental health system . Child Psychiat Human Develop . 2020;51:913-924. doi:10.1007/s10578-020-00968-9

Rosenbusch H, Soldner F, Evans AM, Zeelenberg M. Supervised machine learning methods in psychology: A practical introduction with annotated R code . Soc Personal Psychol Compass . 2021;15(2):e12579. doi:10.1111/spc3.12579

Mishra S. Decision-making under risk: Integrating perspectives from biology, economics, and psychology . Personal Soc Psychol Rev . 2014;18(3):280-307. doi:10.1177/1088868314530517

Csikszentmihalyi M, Sawyer K. Creative insight: The social dimension of a solitary moment . In: The Systems Model of Creativity . 2015:73-98. doi:10.1007/978-94-017-9085-7_7

Chrysikou EG, Motyka K, Nigro C, Yang SI, Thompson-Schill SL. Functional fixedness in creative thinking tasks depends on stimulus modality .  Psychol Aesthet Creat Arts . 2016;10(4):425‐435. doi:10.1037/aca0000050

Huang F, Tang S, Hu Z. Unconditional perseveration of the short-term mental set in chunk decomposition .  Front Psychol . 2018;9:2568. doi:10.3389/fpsyg.2018.02568

National Alliance on Mental Illness. Warning signs and symptoms .

Mayer RE. Thinking, problem solving, cognition, 2nd ed .

Schooler JW, Ohlsson S, Brooks K. Thoughts beyond words: When language overshadows insight. J Experiment Psychol: General . 1993;122:166-183. doi:10.1037/0096-3445.2.166

By Kendra Cherry, MSEd Kendra Cherry, MS, is a psychosocial rehabilitation specialist, psychology educator, and author of the "Everything Psychology Book."

Problem-Solving Interview Questions & Answers

problem-solving

  • Updated July 13, 2023
  • Published March 9, 2020

A job interview is a great moment for interviewers to evaluate how candidates approach challenging work situations . They do this by asking problem-solving questions. These types of questions are commonly asked during interviews since problem-solving skills are essential in most jobs. In any workplace, there are challenges, and when hiring new personnel, hiring managers look for candidates who are equipped to deal with this.

Problem-solving questions are so-called behavioral interview questions . Behavioral interview questions are strategic type of questions that require you to provide an answer that includes an example situation that you experienced in your career. These questions focus on specific work situations that you experienced and how you responded.

A basic example of a behavior question about problem-solving is ‘ tell me about a time you solved a problem at work .’ As you can see, the interviewer is looking for you to explain a situation and how you approach it, and how you solved it. Furthermore, the interviewer is interested in what you learned from that experience. Answering behavioral questions requires some work because you need to provide the interviewer with a strong answer to convince them that you’re the right person for the job.

The rationale behind asking problem-solving questions is to discover how you approach complex and challenging situations and if you can provide an effective solution. Interview questions about your past behavior might sound challenging, but they are actually a great opportunity for you to show that you’re a fit for the position. With the right preparation, you can use your answers to problem-solving questions to your advantage.

What Are Problem-solving Interview Questions?

Basically, problem-solving skills relate to your ability to identify problems, issues, obstacles, challenges, and opportunities and then come up with and implement effective and efficient solutions. However, this is a broad definition of problem-solving abilities. Depending on the position and field you’re applying for a position in the interviewer can focus on different aspects of problem-solving.

Examples of problem-solving competencies are:

Taking initiative.

Taking initiative means that you step up to the plate when needed and that you take action without being asked to do so. People who take the initiative demonstrate that they can think for themselves and take action whenever necessary. Furthermore, you actively look for opportunities to make a difference in the workplace.

Creative thinking

Creative thinking means that you’re able to look at something in a new way to find a solution. People who are creative have the ability to come up with new ways to carry out their tasks, solve problems, and meet challenges. Creative people are original thinkers and are able to bring unorthodox perspectives to their work.

Resourcefulness

Resourcefulness is the ability to find quick and clever ways to overcome challenges in your work. Furthermore, people who are resourceful are original in their ways of thinking to overcome challenges.

Analytical thinking skills

These skills refer to the ability to gather data, break down a complex problem, weigh pros and cons, and make logical decisions. People who possess analytical thinking skills help the company overcome challenges and are able to spot potential issues before they become actual problems.

Determination

Determination can be described as the firmness of purpose or resoluteness. Specifically, people who are determined are persistent and do not give up easily or when they have a setback. Determination gives these people the motivation to push through and keep moving forward.

Results-oriented

People who are result-oriented have their full focus on getting to the desired outcome.

Problem-solving behavioral interview questions

As discussed in the introduction, problem-solving questions fall into the behavioral category of interview questions . These questions ask you to provide specific examples of past work experiences. For interviewers, understanding your past professional performance is the best way to gauge your future job performance.

Behavioral questions are focused on the desired skills or competency area, such as in this case, problem-solving. Other common competency areas for which behavioral questions are used are teamwork , communication , time management , creative thinking skills , leadership , adaptability , conflict resolution , etc.

Behavioral job interview questions usually start with the following:

  • Give me an example of
  • Tell me about a time when you
  • What do you do when
  • Describe a situation where

Examples of problem-solving behavioral interview questions:

  • Give me an example of a time you had to solve a difficult problem at work.
  • Tell me about a time when you identified and fixed a problem before it became urgent.
  • What was the best idea you came up with at your last position?
  • Describe a situation where you find a creative way to overcome an obstacle.

As you can see, the questions mentioned above require you to discuss your past behavior in a professional work environment. The reason for asking behavioral job interview questions instead of just asking traditional ones is that the most accurate predictor of future performance is your past performance in similar situations .

The interviewer wants to discuss previous work situations and wants you to elaborate on them to get to know you better. Solid interview preparation will help you give the answers that the interviewer is looking for. This starts with doing your research and thoroughly reviewing the job description . Doing so can help you understand what type of problem-solving skills are required to successfully perform the job you’re interviewing for.

By preparing example scenarios to questions you expect based on your research , you can give exactly the information that he or she is looking for. In other words, you need to relate your answers to the job requirements and company culture of the organization where you’re applying for a position.

To answer behavioral questions about your problem-solving skills, you need to ensure that you provide the interviewer with specific details about the situation you were in, your task in that situation, the action you took, and the specific results that came out of those actions. In short, this is called the STAR-method of providing an answer. The STAR method is discussed in more detail later on in this article.

Why Interviewers Ask Problem-solving Interview Questions

Problem-solving skills are required in most job positions. This means that a lot of hiring managers will try to assess your problem-solving skills during your interview. The main reason for asking you about situations in which you used your problem-solving skills is to get an understanding of how you work.

The interviewers want to get questions answered, such as:

  • Are you results-oriented, and are you proactively involved in your work?
  • Do you look for different ways to contribute?
  • Are you an individual that others can count on to increase team performance?
  • Are you a self-starter, or do you need someone to give you instructions?

Most likely, the interviewers look for a self-starting person with general problem-solving skills that can be used in different situations. A proven track record of solving problems such as those required in the position you’re interviewing for will definitely help convince the interviewer. Therefore, make sure you prepare answers to questions you expect in advance.

For example, someone who works as a customer service representative should be able to deal with a frustrated or angry customer . They need to be able to solve these problems and know how to handle such situations. Other examples of positions where problem-solving skills are essential are, for instance, accounts or project managers. They need to be flexible in their approaches and should be able to handle a change in deadlines. Another example is, for instance, a logistic manager who should be able to fix an inefficient logistics process.

The Interviewers’ Goal When Assessing Your Problem-solving Skills

There are several underlying reasons why interviewers use behavioral questions to assess your problem-solving skills. The main one, of course, is that they want to hire a person who’s able to perform the job.

Instead of hiring the person that they ‘like’ they need something better to figure out which candidate is the right fit for the job. By analyzing your behavior in past situations that are similar to the ones that are required in the role that you’re applying for, they try to do just that. Below we discuss a couple of important elements employers consider when making a hiring decision.

Costs of making a bad hiring decision

Employers want to make sure that they hire the right person for the job. For a company, making a bad hiring decision is not only about losing money, but it can also lead to a decrease in productivity and morale. Hiring a bad candidate could lead to leaving a bad impression on customers/clients, but also with coworkers.

Furthermore, time will be lost if the company needs to search for another candidate after a bad hire. Therefore, employers do everything to avoid such situations. Behavioral questions are regarded as a preventative way to make sure that the right person with the right fit for the company is hired .

Specific details of your behavior

By asking behavioral questions about your problem-solving skills, the interviewers try to uncover specific details of your behavior. They want to find out if you are able to clearly identify a problem and if you are able to come up with an efficient and effective solution when needed.

Of course, they got your resume already and maybe even a motivational letter or letter of recommendation . Still, the interviewer can only assess your hard skills and educational levels based on these documents.

Essential soft skills , such as problem-solving, are easier to assess during job interviews with the help of behavioral interview questions. Therefore, include real-life work scenarios in your answers that demonstrate how you have used the skills required for the position that you’re interviewing for. The interviewer wants to assess if you possess the skills required to perform the day-to-day tasks and deal with challenges that you will encounter in the workplace.

Your (past) behavior as a predictor of your future job performance

Questions about your problem-solving skills and the answers you give are used to determine the chances of your future success in the job that you’re interviewing for.

Specific behavioral problem-solving questions such as ‘ tell me about a time you had to solve a problem at work. What steps did you take before deciding on how to solve the problem, and why? ‘ give the interviewer more insight into your professional behavior and in turn, your future job performance .

Another way to assess your behavior is by asking hypothetical questions. If you, for instance, do not have certain experience yet, the interviewer could ask you a question along the lines of ‘ What would you do if you were caught off-guard by a problem that you had not foreseen? Which steps would you take to address the problem? ‘. As you can see, this question is hypothetical in nature. The interviewer wants to hear which steps you would take to address a possible complexity in your work. Based on your answer, the interviewer will assess if your approach is suitable for the position for which you’re interviewing.

It’s therefore important that you prepare for frequently asked interview questions that you can expect during your interview. By preparing the right example answers on how you have solved problems in your previous jobs and how you would solve problems in the job you’re applying for, you can provide a concise answer without missing important details.

Avoid making a wrong hiring decision

Questions that gauge your professional behavior help employers assess your future job performance. In other words, this helps them make a better hiring decision. A perfect resume or cover letter is not enough to impress seasoned interviewers.

By asking behavioral problem-solving questions, the interviewer tries to uncover your previous work patterns. The information in your answers gives them more insight into your approach to critical situations and if this approach matches the ones required for the position you’re applying for.

By preparing the right way, you can make sure that your example answer situations include aspects of the most important job requirements. Of course, the interviewer is looking for candidates that fit the job description , so make sure that your answers relate to the job requirements.

What Interviewers Look for in Successful Candidates

In short, interviewers look for candidates who have the right work approach to succeed within their company and in that particular position. This is also why we can’t emphasize the importance of being able to demonstrate your skills through solid example scenarios enough .

The right preparation will help you get there. Your goal is to demonstrate that you are capable of taking on the day-to-day tasks required for the position and have the potential to grow . For example, if you are able to work in and deal with transitions in fast-paced environments such as financial markets . And can you handle the complex situations that you will encounter? Are you able to deal with such transitions effectively? In this case, you need to show adaptability and problem-solving skills through example scenarios of how you did so in the past.

Problem-solving behavioral questions are used to get insights into how you approach problems at work, if you take the initiative, and if you possess the right creative and critical thinking skills . Basically, the interviewers want to get the following questions answered:

  • Do you take the initiative?
  • Can you communicate effectively?
  • Are you able to adequately respond to problems or issues that occur during your work?
  • Can you perform in stressful and unexpected situations?
  • Are you able to adjust to changing work environments?
  • Can you assist your coworkers or team when needed?
  • Are you flexible in your approaches to situations at work?

Red Flags for Interviewers Assessing Your Problem-solving Skills

When answering questions about your problem-solving skills, there are certain things you need to look out for. Below we discuss a couple of warning signs that interviewers consider when you answer their questions. Ensure that you avoid these at all costs to avoid making the wrong impression.

1. Not answering the question or not providing enough detail

If you answer a question with ‘I can’t recall a situation where I encountered such a problem ,’ this is considered a red flag. This could mean that you did not prepare well and that you’re not taking the interview seriously. Furthermore, the interviewer could interpret such an answer as you may avoid dealing with challenging situations.

If you cannot provide specific details or examples about what you claim in your resume or cover letter, this can be considered a red flag too. If you, for instance, claim that you have successfully solved problems and used critical thinking skills in your work, you need to make sure you’re able to back this up through clear examples of times you did so. Failing to do so could lead to a quick elimination of your candidacy for the position. If the interviewer has trouble verifying your employment history, this is considered a warning sign.

2. Canned responses to questions

Preparing answers is key to success for any interview. However, this means preparing original, effective, and relevant answers that are related to the position you’re interviewing for.

Generic answers to behavioral problem-solving questions such as ‘ tell me about a time you had to solve an issue with a customer ‘ are considered warning signs. An example of a generic answer to that particular question is ‘t his one time I had to deal with an angry customer who had complaints about the pricing of a product. I calmed her down and made the sale ‘. As you can see, this answer does not provide much insight into your problem solving skills, thought process, and how you approach the situation.

If you give a generic answer, you can expect more follow-up questions from the interviewer. However, it’s better to prepare strong answers to impress the interviewer that you actually possess the required skills for the job.

3. Answers that focus on problems, not solutions

The reason for asking specific behavioral-problem solving questions is to assess how you approach and solve problems. It’s, therefore, important that your answers focus on the solution, not the problem . Of course, it’s important that you are able to spot and identify problems, but finding a solution is essential. If your answers focus on problems too much, you can come across as too negative for the job.

Negativity, in any form, in your answers, is considered a red flag. This can be talking negatively about a problem you had to solve but also talking inappropriately about previous employers or co-workers. Negative undertones never impress interviewers the right way. Therefore, focus on how you solve problems and put yourself in the best light possible.

4. Too stressed or uncomfortable during an interview

Interviewers know that almost everybody is slightly uncomfortable when put on the spot during a job interview. However, when you’re too stressed to provide a good answer, this can be viewed as an indicator that you do not handle stressful situations well. Of course, remaining calm under pressure while still being able to solve problems is essential for positions in which problem-solving skills are required.

5. Failing to respond effectively

Failing to respond effectively to interview questions comes across weak. It’s therefore important that you prepare for your interview by thoroughly analyzing the job description and try to understand what kind of problems you will be solving in the position that you applied for. This research will help you choose the right examples from your past that are most likely to impress the interviewer.

Therefore, research the job and organization and make notes of the required skills and experiences you think the company values. This allows you to tailor your answers to your situation.

Also, think about possible follow-up questions the interviewer might ask you. Because you already know what examples you will use in your answers to questions you expect , if you prepare the right way, you can figure out which follow-up questions are likely to be asked. For instance, if you’re preparing for the interview question, ‘ tell me about a time you solved a problem at work ,’ you can expect the interviewer to follow up with, ‘ what steps did you take to solve the situation?’.

6. Not taking responsibility or minimizing the significance of a problem

When a problem is identified but not addressed, this could quickly escalate into a bigger problem. Employees who do not take responsibility or those who leave things for later might not be result-oriented and engaged in their work.

Another way of taking responsibility is to show self-awareness. It’s common for interviewers to ask you about a time that you failed, especially in situations where you needed to solve problems. They are interested in what went wrong in a work situation, if you took responsibility for your actions, and what you learned from that situation. Not taking responsibility for, for instance, a  project that may have failed , is considered a warning sign.

Self-awareness and being to reflect on situations is an important characteristic to possess in the workplace. Interviewers want to hire candidates that can admit errors or who made thoughtful mistakes trying to solve problems in the past and tried to fix them. Employers know that candidates are human and make mistakes, just like everybody else. It’s important that your answers show that you take responsibility for situations and describe the actions you took to repair any problems or challenges.

Frequently Asked Problem-solving Interview Questions

Below you can find commonly asked behavioral problem-solving questions . These questions are divided into regular questions and hypothetical questions. Learn everything you need to know about common interview questions that are frequently asked during job interviews .

Problem-solving interview questions:

  • Tell me about a time you faced an unexpected challenge at work and how you dealt with it.
  • Describe a time a customer approached you with a problem or concern. How did you respond?
  • Tell me about a time you had to change your planned course of action at the last moment. How did you re-evaluate your priorities?
  • What was the best idea you came in in your last position?
  • Tell me about a time you had to solve a difficult problem.
  • What’s the most significant improvement that you have made in the last year?
  • Tell me about the most innovative new idea that you have implemented in the workplace.
  • Have you ever improved the workflow of a project based on your analysis? How did you do this?
  • Describe a situation in which you anticipated a potential problem and applied preventive measures.
  • Tell me about a time you faced a significant obstacle you had to overcome to succeed in a project.
  • When you’re working on several projects, it’s tricky to deliver excellent service to all of them. How do you go about prioritizing the needs of a client?
  • Describe a situation in which you had to analyze information and make a recommendation.
  • What do you consider your greatest achievement in the workplace? Why?
  • Describe a situation in which you needed to motivate others to get something done.

Hypothetical problem-solving interview questions:

  • How would you approach a situation in which you had to analyze information to make a recommendation to a client?
  • Tell me how you would handle a situation in which you have a deadline you cannot meet.
  • How would you handle a conflict with a co-worker?
  • A frustrated client calls you to discuss a problem. How do you deal with such a situation?
  • How would you handle a situation in which you would need to convince someone to change their decision?

Preparing Answers to Problem-solving Interview Questions

There are several steps that you can take to prepare for problem-solving questions. Here you can find a job interview checklist . To get started, you can consider the following steps.

Step 1: Research

Before your interview, it’s important that you thoroughly research the position and company. Read the job description carefully to find specific skills that a candidate needs to possess to successfully perform the job. Think of skills such as adaptability , communication , and problem-solving. Also, read the company website to get more information about their mission statement and who their main clients are. Furthermore, check their LinkedIn pages and other content/news related to the company.

Your research will help you identify the required skills, qualities, and experience for the position. In turn, you can use this information to make an educated guess about what kind of interview questions you can expect .

Step 2: Write down the required skills, competencies, and experience

Behavioral questions such as those about problem-solving skills are a great opportunity for you to show why you’re the perfect candidate for the job. Based on the skills and competencies that you have identified during your research, you can start preparing answers. Rank the skills on importance in relation to the requirements for the position.

Step 3: Create a list of past work experiences related to the position’s requirements

Everybody knows that it’s hard to come up with strong answers when you’re put on the spot during an interview. Therefore, come up with strong examples to questions you expect ahead of your interview.

Create a list of past work experiences and tailor them to the required skills and competencies for the job—highlight successful situations where you demonstrated behavior related to these required skills and competencies . Focus on delivering a concise and to-the-point answer.

Step 4: Prepare successful and challenging answer examples

Effective problem-solving skills are essential in the workplace. Therefore, your answers must demonstrate that you have successfully identified problems, proposed solutions, evaluated several options, and finally implemented a solution. However, it’s also likely that the interviewer will ask you about a time you have failed to solve a problem . Interviewers ask you about failures to assess whether or not you learn from your mistakes and if you’re self-aware enough to acknowledge times you failed. Also, it helps them identify if you take calculated and smart risks.

Step 5: Use the STAR method to structure your answers

The STAR method allows you to concisely provide the interviewer an answer by logically walking them through the situation. STAR is an acronym that stands for a situation ( S ), your task ( T ) in that situation, the actions ( A ) you took, and what results ( R ) you got based on your actions. These are the basic steps you take in your walkthrough.

Below we discuss the STAR interview technique in more detail.

STAR Interview Technique For Problem-Solving Interview Questions & Answers

By using the STAR method, you can give an answer that includes exactly what the interviewer is looking for. Below, the STAR acronym is broken down into each step.

Start your answer by explaining the situation that you faced. The start of your answer ‘story’ should answer questions such as:

  • What was the situation/problem?
  • Who was involved?
  • Why did the situation happen at that time?

It’s important to provide context around what problem needed to be solved. Furthermore, make sure to provide relevant details.

Next, explain your specific role in the task ahead. Include important details, such as specific responsibilities. Focus on giving the interviewer an understanding of your task in solving the problem. This part of your answer should answer questions such as:

  • Why were you involved in that specific situation?
  • What’s the background story?

After you describe your task, it’s time to specifically discuss the actions you took to solve the problem. Give the interviewer a step-by-step description of the actions you took. This part of your answer should answer questions such as:

  • What steps did you take to resolve the situation you were in?
  • Why did you choose to complete your tasks this way?

Finish your answer by discussing the results you got from your actions. Detail the outcomes of your actions and ensure to highlight your strengths . Also, make sure to take credit for your behavior that led to the result. Focus on positive results and positive learning experiences. This part of your answer ‘story’ should answer questions such as:

  • What exactly happened?
  • What did you accomplish?
  • How did you feel about the results you got?
  • What did you learn from the situation?
  • How did this particular situation influence who you are as a professional today?

Sample Answers to Problem-solving Questions

Below you will find some example questions. The examples are already written in STAR format so that you can clearly see how you can structure your answers. However, these are ‘general’ examples. Do not forget to structure your own answers in a way that includes enough detail to convince the interviewer that you’re the right person for the job!

Problem-Solving Example 1: ‘ Tell me about a time you had to resolve a disagreement with a coworker.’

‘Personally, I believe that communication is essential in such a situation to find a way that works for both of us. Finding a compromise is the main goal to get the work done to the best of our ability.

Task & Action

In my current position as a financial consultant, I encountered such a situation recently. A colleague disagreed with the way I wanted to handle an issue that we encountered along the way. To address this issue, I scheduled a meeting with him to discuss the situation. I asked him about his points of view and how he thought we should go about the project.

Even though we had differences in the way we felt like how the project should be approached, we quickly came to the conclusion that our goal was the same; providing our client with a high-quality final product within the set deadline.

We talked about the project and the specific aspect about which we had a difference. I explained my point of view and that I had already encountered a similar issue in the past. Ultimately, my colleague agreed to tackle the issue using my proposed method. His insights gave me a good suggestion which we incorporated into the project. After that, we successfully worked together and finalized the project in time and according to the quality level that we both were proud of.’

Why this is a strong answer:

  • The provided example is concise and relevant to the workplace where problem-solving skills are important
  • This answer shows important skills such as being proactive, problem-solving, persuasion, and adaptability .
  • The answer shows that you’re a team player as well and that you listen to the input of others for the better of a project’s result.

Note : There’s always a chance that interviewers ask you follow-up questions about how you convinced your colleague. Make sure that you are able to answer those questions as well.

Problem-Solving Example 2: ‘ Tell me about a time you had to solve a challenging problem at work .’

‘In my position as a business development manager at ABC Software, I’m responsible for organizing all client events and conferences. ABC Software is a major player in the IT market, and during our events, we invite industry experts to speak on market developments. These events are used to attract new clients but also to maintain our relationship with our existing ones.

Over the last two years, we analyzed our attendee data and found that our event attendance dropped by almost 10%. Furthermore, we discovered that the retention rate of our clients also decreased. When we had to plan the next event, my team and I knew that we had to get our attendance levels back up in order for the events to stay successful. The goal was to get our networking event popular and recurring again.

I had an idea why the attendance levels dropped but to get more information, I interviewed several sales consultants as well. The main feedback was that we should focus more on attracting new clients through social media channels. I communicated this with our marketing team, and we decided to also reach out to our client base and ask them what they would like to see on our future events. This led to interesting new insights on topics and speakers that we could invite, plus we also received input on how to improve networking possibilities during our events. Based on our research and feedback, I created a new plan of action to market our events through our social media channels to increase exposure.

After launching our marketing campaign, we immediately gained online traction, leading to an increase in advance registrations. For that specific event, we saw a total increase in attendance of 20% in comparison to the previous year. An online survey showed that the attendees were happy with how the way the new event was structured, and 80% of respondents said that it would be likely that they would recommend our events within their network.

My approach to increasing attendance at our events did not go unnoticed. I was asked by my department director to make a presentation about how I tackled this problem and present this to the board.’

  • This example shows that you can identify issues and understand your responsibility to address them.
  • The provided example is related but also relevant to the workplace. It’s also concise, which is perfect.
  • This answer shows important skills, such as being proactive, teamwork , adaptability , problem-solving skills, and creativity .
  • Taking responsibility to find out why the event attendance dropped and subsequently taking action turned out successful gives more weight to the situation.

Problem Solving Example 3: ‘Describe a time a customer approached you with a problem or concern. How did you respond?’

‘In one instance, a customer came to me with an issue. She had recently purchased a product from our store, which broke shortly after she got it home. She was understandably upset and wanted to know what could be done.

In response, I apologized for any inconvenience and asked her to explain what had happened. After hearing her story, I promised to help her as much as possible. Next, I checked the item’s warranty status in our system.

I was able to offer her a replacement or a refund since the product was still under warranty , and I helped her find an identical item in our store and processed the exchange for her. The customer decided she wanted a replacement, so I explained our return policy to her in case this ever happened again in the future.

My customer thanked me for my help and seemed more satisfied at the end of the transaction; I was glad I was able to turn a potentially negative experience into a positive one.’

  • This example shows that you understand what great customer service is.
  • The provided example is concise and to the point; it describes a situation and the actions you took to resolve it.
  • This answer shows essential skills, such as being proactive, customer service, and problem-solving skills.

Rate this article

Your page rank:

Interview categories

  • Interview Questions
  • Cover Letter
  • Interview Tips

Megainterview/Contact

  • Career Interview Questions
  • Write For Megainterview!
  • Editorial Policy
  • Privacy Policy / GDPR
  • Terms & Conditions
  • Contact: [email protected]

Sign-up for our newsletter

🤝 We’ll never spam you or sell your data

Popular Topics

  • Accomplishments
  • Adaptability
  • Career Change
  • Career Goals
  • Communication
  • Conflict Resolution
  • Creative Thinking
  • Critical Thinking
  • Cultural Fit
  • Customer Service
  • Entry-Level & No Experience
  • Growth Potential
  • Honesty & Integrity
  • Job Satisfaction
  • Negotiation Skills
  • Performance Based
  • Phone Interview
  • Problem-Solving
  • Questions to Ask the Interviewer
  • Salary & Benefits
  • Situational & Scenario-Based
  • Stress Management
  • Time Management & Prioritization
  • Uncomfortable
  • Work Experience

Popular Articles

  • What Is The Most Challenging Project You Have Worked On?
  • Tell Me About a Time You Had to Deal With a Difficult Customer
  • What Have You Done To Improve Yourself In The Past Year?
  • Interview Question: How Do You Deal With Tight Deadlines?
  • Describe a Time You Demonstrated Leadership
  • Tell Me About a Time When You Took Action to Resolve a Problem
  • Job Interview Questions About Working in Fast-Paced Environments
  • Job Interview: What Areas Need Improvement? (+ Answers)
  • Tell Me About a Time You Were On a Team Project That Failed
  • Tell Me About a Time You Managed an Important Project

Our mission is to

Help you get hired.

Hofplein 20

3032 AC, Rotterdam, the Netherlands

Turn interviews into offers

Every other Tuesday, get our Chief Coach’s best job-seeking and interviewing tips to land your dream job. 5-minute read.

  • Resume Templates
  • Resume Examples
  • Free Resume Builder
  • How to Write a Resume
  • Resume Format
  • Resume Packs
  • Cover Letter Templates
  • Cover Letter Examples
  • Free Cover Letter Generator
  • How To Write a Cover Letter
  • CV Templates
  • CV Examples
  • Free CV Maker
  • Resume Help
  • Cover Letter Help
  • Job Interview
  • Career Advice

25 Problem-Solving Interview Questions And Sample Answers

Elena Prokopets

Every day we face a ton of mishaps — from a glitching messenger app to a compliance update, sending your industry into chaos. Compound this with rapid technology change and shifting customer behaviors, and it becomes apparent that strong problem-solving skills are highly important in the workplace.

So much so that 60% of employees want to see evidence of problem-solving skills when evaluating candidates. In interviews, candidates will be asked problem-solving questions . 

In this post, we provide common problem-solving interview questions employers use to screen candidates (with sample answers!). But first, let’s recap the basics. 

What Are Problem-Solving Interview Questions?

Problem-solving interview questions assess critical thinking, data analysis, and decision-making abilities. Candidates face hypothetical situations or case problems to test their analytical , critical thinking , and conceptual skills . 

Nail the problem-solving questions, and you’re likely to get the role: 70% of employers consider strong critical thinking abilities as a huge indicator of job success. 

Why Would Employers Ask Problem-Solving Questions During an Interview?

Problem-solving questions are a good way to evaluate your ability to overcome work challenges. Most employers want to be sure you can resolve issues and move past bottlenecks independently.

In other words: They want to see how you apply deductive reasoning or analytical frameworks to determine the root cause of the problem and then determine the best solution for troubleshooting.

The purpose of interview questions for problem-solving may also vary depending on the role. 

  • For customer-facing roles, problem-solving questions are a great way to assess conflict management and issue-resolution skills. 
  • For management roles , they provide insights into the candidate’s strategic thinking and planning abilities. 
  • For technical roles , these help evaluate your approaches to issue troubleshooting and process optimization. 

In every case, the employer expects to see how you apply your cognitive, analytical, communication, and decision-making skills.

Popular Types of Problem-Solving Interview Questions (and Answers) 

Because problem-solving assumes using a range of hard and soft skills , there are multiple types of interview questions employers may ask. To help you practice, we organized popular problem-solving interview questions into different groups. 

Situational Interview Questions

Situational interview questions ask you to describe your line of thinking and actions in a certain setting. Most ‘mock’ situations will be directly related to your role. For example, as a social media marketing manager you may get asked “What would you do if you noticed a typo in an update 10 hours after publishing when people have already been commenting on it? 

The best approach to situational problem-solving questions is using the STAR interview method . First, describe the situation. Next, talk about the task (problem) you’ve had. Then explain what actions you took. Finally, conclude with an outcome (result) gained. 

Here are several sample problem-solving questions with answers for this category. 

A customer asks for a product, but it’s out of stock. They’re unhappy. How would you respond? 

For customer-facing roles, you may be probed with a problem-solving interview question presenting some sort of a customer issue. Such questions are also common in the hospitality, restaurant, and retail industries among others.

Your goal is to showcase your stellar customer service and conflict resolution skills. 

Sample answer: 

First, I’d ask the customer if they’d be open to some alternatives — and provide a range of similar products we currently have in stock. If neither works for them, I’d look up the restock information and offer to put them on a notification list. Or, if they are open to that — suggest placing a backorder. If they are still not happy, I’d politely ask them to wait for a moment and approach the manager about the possibility of issuing a discount for them or offering free expedited shipping once the product is back in stock.

You are last to leave the office, but can’t find your keys. No one else is around. What would you do?

This is another sample situational interview question, prompting you to talk about your approaches to responding to unexpected circumstances. The other party wants to understand whether you’d be following the protocol or acting erratic (or unprofessional).

Here’s how you should answer this question:

Well, I’d first re-check if I haven’t misplaced my keys and search all my belongings. If I truly don’t have them on me, I can’t leave the office without properly securing it, right? So I’d try calling my manager to see if they could help — or another employee, whom I know to leave close by. I believe one of them would be able to come and help me out or direct me towards the right HR person to contact about this.

You’ve hatched a detailed plan. But there were some last-minute changes from the senior stakeholder, affecting your timeline. How would you respond?

Not all projects go as planned. The purpose of this question is to test your adaptability skills. The interviewer also wants to understand whether you’ll push back on the change or try to implement it even if that would result in extra work for you. 

Sample answer:

 This happened quite a few times in my last job, where the CEO liked to propose last-moment tweaks to investor reports. At first, I just went along and adjusted the copy and design myself before publishing. After the second time, I started sending an investor report draft to the CEO 7 days before the publishing date and set a hard deadline for her edits. This helped fix the issue. 

You and your team are stuck in a traffic jam. You are running late for an important client meeting. What would you do?

That’s another common situation, that plenty of people can relate to. The interviewer wants to see a demo of your communication and on-the-sport decision-making skills. 

Assuming I’d be still late if I walk or use public transport, I’d do this: Phone in the client with my apologies. Then propose to either order lunch/refreshments for them while they’re waiting or propose to start the meeting on video conferencing from the car if that’s possible. 

Scenario-Based Interview Questions 

Scenario-based interview questions present you with a specific problem the interviewer asks you to solve.  Rather than assessing your immediate response, problem-solving scenarios aim to test your and ability to strategize.  In most cases, there’s no right or wrong answer to such questions. Your goal is to demonstrate your thought process. 

Below are several examples of problem-solving scenarios for interviews. 

You have two vendors: One has lower prices and another proposes faster shipping. Which one would you pick and why? 

Many interviewers like to pose such questions to evaluate a candidate’s decision-making skills. The interviewer wants to understand how you access different options when making operational calls. Give them a walkthrough.

I’d check two metrics first — planned deadlines and current budgets. If a later delivery doesn’t affect the manufacturing schedule, I’d go with a cheaper vendor. If the materials are time-sensitive, I’d approach the CFO regarding the matter and explain why paying a higher supply price is more favorable than risking manufacturing delays (and bearing direct and indirect costs of that). To make my case, I’d use ERP data and a business intelligence app to model different scenarios.

You need to kick off the project but don’t have full data. What are your next steps? 

For most companies, the current economic realities are rather volatile — from ongoing supply chain disruptions to rapid changes in consumer preferences. Thus, operational decisions have to be taken fast, often with incomplete data. 

By posing this question, the interviewer likely wants to assess your general business acumen skills, as well as approaches to strategic planning. 

Sample answer

As a marketing manager, I fully understand that good data may not always be available. In such cases, I try to generate my own data and test assumptions. First, I try to split-test different types of creative and run them by a sample target audience group. Based on the response rates (e.g. average click-through rates), I then select the main creative to use in the campaign.

A senior colleague asks for your recommendation on a new policy. How would you prepare it?

A good answer to this problem-solving interview question will include a list of steps you’d follow and the type of resources you’d use to make an informed decision. Explain how you usually collect data, make assessments, and synthesize findings to present them to others. 

I’d kick things off with an impact assessment to understand the context, objectives, and outcomes of the proposed change. I’d model different scenarios using a custom script on Power BI that I’ve made. Once I have the hard numbers (e.g., impact on revenue, efficiency, cost savings), I’d analyze the cultural impact of policy change. That usually involves conversations with other stakeholders and department heads. I’d incorporate their feedback and provide summarized findings to the colleague. 

You’re asked to identify cost-saving opportunities for a company. As you review the financial statements, you notice that operating expenses have increased significantly over the past quarter. How would you approach this? 

This is an example of a precise scenario-based question you may get for a financial analyst or accounting role . Other positions also receive similar questions, based around a difficult on-the-job situation. Your goal is to demonstrate your approach to issue resolution. 

I would first analyze all the groups of expenses to determine what drives the increase. If the company is spending more to grow, I’d calculate the ROI of that spending to justify it or, on the contrary — challenge it. If the cost inflation is due to excessive spending on low-value initiatives, I’d suggest several optimization strategies.

Behavioral Problem-Solving Interview Questions 

Behavioral problem-solving questions aim to learn more about your personality. They encourage you to provide examples of how you’ve acted in the past and showcase your general attitudes towards different challenging situations.

These provide room to demonstrate your self-management skills and mental resilience. So be sure to prepare some problem-solving examples for interviews beforehand. 

Tell me about the time you’ve faced a major challenge at work

This question can be more context-specific. For example, the interviewer may prompt you to talk about meeting an unrealistic deadline, resolving a professional mishap, or dealing with another type of out-of-the-ordinary work situation. In every case, you must not just describe the problem, but communicate what you’ve done to resolve it. 

My sales team spent 6+ months preparing for a major demo for this manufacturing client. It was an important strategic deal for Acme Inc. Two days before the presentation, the main Account Manager fell sick with COVID-19 and couldn’t do the meeting. Since I worked closely with him, I volunteered to moderate the presentation and facilitate the discussion. We’ve notified the client team about the changes and I’ve invited their management to a quick lunch a day ahead to meet up and “break the ice”. Then helped negotiate. We’ve successfully closed this deal.

What’s your standard approach to resolving blockers at work? 

The answer to this problem-solving interview question will be somewhat different for regular employees and managers. As a regular employee, you should focus your reply on your organizational skills . As a manager, you should lean more towards your administrative and leadership skills . 

Below is a sample answer from a manager’s perspective: 

I’d describe my management style as a facilitator. As a UX Design Lead, I spend a lot of time prioritizing our backlog in line with the company-wide product roadmap and collecting regular input from other teams. Based on it, I set different levels of priorities for design tasks and map dependencies between them. Then I communicate the main priorities in this Sprint to the design team every 2-3 months. Weekly, I go through the work backlog to analyze progress and reach out to individual members on status reports. If the person is stuck, I try to figure out the root cause for that first, then get back to them with different suggestions on how to move forward.

What does “being resourceful” mean for you? 

Employers want autonomous go-getters, who know how to accomplish tasks, given the existing constraints. The hiring manager wants to understand how you make the max out of the available resources. Illustrate this with a quick example.

I treat constraints as an opportunity to be creative and innovate with frugality. I maintain an inventory of all creative assets available to me and like recycling content for different channels. For example, one podcast episode = 1 more blog post, 5 social media updates, and a collection of quotes the team can then use for other marketing assets without bothering the SME again. 

Could you exemplify your “self-sufficiency” abilities? How do you ensure high personal performance? 

This interview question prompts you to talk about your approach to staying motivated and methodical in your work. The interviewer wants to understand how you solve problems on your own and ensure that temporary setbacks don’t affect your performance.

I’ve been working remotely for three years now and my current employer prioritizes async communication, so I’m used to solving issues on my own. When I’m dealing with a coding problem, I usually head to Stackflow exchange to see if there are existing threads, plus search for reference architecture of similar solutions. There’s so much information available these days, so it’s easy to find answers to even the most niche problems. 

Problem-Solving Questions for Teamwork

A lot of issues arise due to misunderstanding and interpersonal dynamics. The employer wants to understand that you can diffuse tensions, handle arguments, and prevent conflicts professionally. So be prepared to answer some problem-solving interview questions around teamwork.

Your colleague proposes an alternative approach. The team can’t decide between your idea and theirs. What would you do? 

Here the interviewer wants to see how you reach consensus. Few teams like managers with their “my way or the highway” attitude. Your goal is to show that you’re not making decisions with your ego. 

I’d once again analyze both approaches holistically together with the team, pointing out the strengths and weaknesses of each. I always encourage everyone to probe my ideas, even though I’m a Senior Architect. Then ask again to contribute further thoughts and vote for the best option.

How do you usually handle workplace conflict between employees of the same level? 

A variation of such interview questions is common for managerial roles. After all, much of your job involves team-building. A good answer will include an example from your experience, demonstrating your conflict resolution strategies. 

I would have informal conversations with both at first to understand the source of animosity. In my last role, I had a UX designer and front-end developer constantly clash due to differences in communication styles. The developer lacked active listening skills and the designer wasn’t best at expressing their thoughts verbally. I’ve suggested they start a shared handover documentation file, where both documented the requirements from each side and leave helpful notes on design animation or tech constraints/compromises. Matters got better after this.

One of your team members is underperforming. This negatively impacts the group dynamic. How would you address this issue?

According to Gallup, 70% of the variance in team engagement is determined by the manager. Hence, employers want to ensure that you can identify and effectively address performance issues. The best answer to this interview question will include an example from your past work. 

A couple of months ago, I noticed that one of our senior developer’s velocity fell by almost 20%. Her code commits were also getting rejected by our CI/CD pipeline at the unit testing stage more often, slowing up the release cycle. Sarah was going through some family issues as learned in a 1:1. I suggested she take a 7-day PTO and also reminded her that we have free mental counseling available. She signed on for a couple of sessions and returned to her best in two weeks.

A stakeholder comments on the quality issues in your project. But these are not your team’s fault. How would you address their concerns while maintaining high team morale? 

Lack of appreciation and recognition of efforts from senior stakeholders can weigh heavily on the teams’ morale and, by proxy, performance. The purpose of this question is to test how you can advocate for your team. Your answer must demonstrate high emotional intelligence and professionalism in managing expectations. 

I would have a private conversation with the person to better understand the source of their concerns about quality. Then explain to them what part of work my team is responsible for and how this relates to the quality issues origination. I would then reassure them that I would speak to the manager, responsible for that line of work myself, and we’ll jointly work on optimizing this problem.

Problem-Solving Exercises 

Some interviewers also like to throw in a couple of weird interview questions , aimed at challenging your on-the-stop problem-solving skills. For example, Jeff Bezos once asked an interviewee to try counting the number of windows in Seattle. While the question may sound absurd, it gives the interviewer a good idea of how you structure your reasoning and employ logical thinking skills. 

In other cases, an interview may include several problem-solving exercises — cognitive puzzles or quiz-style questions you need to complete within a certain time. Some of these may require you to do some arithmetics to arrive at a precise answer. Others are just meant to test your logical reasoning abilities. 

Examples of problem-solving exercises for an interview: 

  • Can you count how many tennis balls would fit into this room? 
  • As a pizza delivery man, how would you benefit from scissors?
  • You have 1000 bottles of wine, and one of them is poisoned. You also have ten rats to test which bottles are poisoned. What’s the fastest way to find a poisoned bottle?
  • You have 3 critical production tasks, requiring the same specialized equipment, but you can only afford to rent one at a time. How will you prioritize and schedule tasks to optimize resources? 
  • You have 3 containers with 20 balls. You have enough room to sort all balls of the same color into separate containers. How will you make sure that each container only has balls of the same time and that no two balls of the same type end up in different containers? 

Case Studies 

Case studies (or case problems) are context-rich, mock business scenarios, designed to test your problem-solving skills. They are common for roles in the consulting and financial sectors. However, many IT companies have also adopted them into their interviewing process.

Generally, you have 15 minutes to review the case study and ask supporting questions from the interviewer. Then another 15 to 30 minutes to prepare your answer. These tasks demand good business acumen — an understanding of the typical business goals and commercial awareness of the market and operating model. 

Your goal is to demonstrate that you understand the key issues and have a structured approach to finding the solution. You need to demonstrate which factors you’ve considered and their implication for the business. Then provide high-level recommendations, based on the data you have. 

Sample case studies for an interview: 

  • If you were a competitor entering a new regional market, how would you convince customers to select our product? 
  • A sports brand wants to launch an online employee advocacy program, where employees act as micro-brand ambassadors — showcase the goods on their social media and provide customer advisory. How would you recommend them to structure this initiative? 
  • A French wine producer wants to enter the Australian market. Prepare a summary, explaining why the market may be a good choice for them and which products may have the highest chance of success. 
  • A friend asks for your advice: They want to launch a new vegan DTC cosmetics brand. What type of go-to-market strategy would you recommend?

You can also find more sample case study interviews to practice at websites from big consulting firms like Bain , BCG , or Deloitte .   

How To Approach Problem-Solving Interview Questions?

When presented with any type of a problem-solving interview question your main goal is to narrate how you’ll use your analytics, situational analysis, and critical-thinking skills to best navigate the matter. You should always clearly communicate what you plan to do and why. Then highlight the outcome you’d aim to achieve. 

Demonstrate structured thinking and a logical progression in your response:

  • Reiterate the problem and ask clarifying questions if necessary. 
  • Explain your first action. Mention why you’ve chosen it over the others. 
  • Be precise with your arguments. State what data you’ve used for decision-making.
  • Explain your next steps and/or alternative course of action if the first option fails to work. 
  • Summarize the outcome you’ve achieved or expect to achieve as a result.  

Remember: the interviewer doesn’t expect you to come up with a highly elaborate multi-step roadmap. They just want to hear how you’ve solved similar issues in the past and how you might react to new challenges!

Elena Prokopets

Elena runs content operations at Freesumes since 2017. She works closely with copywriters, designers, and invited career experts to ensure that all content meets our highest editorial standards. Up to date, she wrote over 200 career-related pieces around resume writing, career advice... more

you might also like

asking the right questions during interview

35 Questions to Ask the Hiring Manager During the Interview

personal values for job interview

6 Examples of Personal Values to Bring Up in an Interview

job interview above and beyond

Swipe-Worthy Going Above And Beyond Examples For Your Interview

cultural fit

14 Culture Fit Interview Questions with Sample Answers

flight attendant

18 Flight Attendant Interview Questions (With Winning Answers)

what can you do, if you l were fired from your job.? can you leave it off your resume? what do you mention in the interview about leaving your job, or having an employment gap?

Leave a Response Cancel reply

MockQuestions

Problem Solving Mock Interview

To help you prepare for your job interview, here are 25 interview questions that will test your problem solving ability.

Get More Information About Our Problem Solving Interview Questions

Question 1 of 25

Tell me about the most challenging problem you have encountered in your professional career.

How to Answer

Answer example.

Everyone has had their share of challenges in their career. The interviewer knows that you are not perfect; however, they need to know that you can professionally overcome work-related roadblocks. Maybe you had a significant project that almost went sideways. Perhaps you had a conflict in the workplace that you could have handled more professionally. Explain your approach to resolving the issue and be sure to highlight the steps you took to reach that resolution.

"The most challenging problem I have encountered in my professional career was with my most recent employer. I had an incredibly important project that made up the majority of my annual budget. The client was challenging to work with as he was rarely available for comment, due to extensive international travel. I needed this deal to work out so, for the 6-month span of the project, I made my work hours reflect his time zone. This shift allowed us to communicate via Skype on a daily basis which meant a fair share of late night and early morning calls for me! It was a sacrifice, and I would do it again. I understand that sacrifices need to happen to gain successful outcomes."

"The most challenging problem that I encountered in my career was when my former company experienced a major merger. It was a lot to adjust to but, after some time, I was able to get a good pace again."

"The most significant challenge I have faced as a manager would be the labor dispute and lockout that our company went through in 2016. Many of our permanent employees are union based. We could not come to a new collective agreement, and so I ended up having to utilize a lot of temporary staffing options during that time. It was a lot of re-training, and strain on the company culture overall."

"The biggest challenge that I face as a marketer, and it's an ongoing challenge, is to manage my expectations on projects. I lean on the side of perfectionism and often put more pressure on myself than even a client would. The positive side of this; however, is that I always deliver an immaculate product."

"I'd say the most challenging problem I have encountered was when my manager suddenly resigned. I was then in charge of the department. Now, I was mostly ready for the responsibility, as the assistant manager in the department. However, I had never completed inventory reconciliation, and on the first day, this was my first task. I was asked to give projections so that our buyer could stock us for next season. I had no idea what to do, so I researched until I came up with the answer. Also, other managers in other departments helped to guide me. Ultimately the work paid off because our next season projections were perfect. Since then, I've learned more effective ways to do our inventory management and projections, but I don't think I've ever learned anything as quickly as I did that week."

"The most challenging problem I've encountered is the misstep of taking my current role. The initial pitch to me on company growth and my duties is not my reality. This factor has been a challenge to my career growth. I know that even if it was a misstep, there are lessons to be learned, and I approach each day with interest and a positive attitude to try to learn those lessons and grow professionally."

"The most significant challenge I've faced is nearly having my department eliminated due to budgetary cuts. I was lucky to have an active parent community rally behind me and the department which saved the program, in the end. The other challenge that comes to mind was getting back into the swing of teaching after taking a few years off to be home with my children. There was a learning curve on getting up to speed with curriculum and the lesson planning, but my love for teaching made it all that much easier!"

Next Question

25 Problem Solving Interview Questions & Answers

Below is a list of our Problem Solving interview questions. Click on any interview question to view our answer advice and answer examples. You may view 5 answer examples before our paywall loads. Afterwards, you'll be asked to upgrade to view the rest of our answers.

1. Tell me about the most challenging problem you have encountered in your professional career.

2. in your opinion, what makes you a great problem solver.

Employers want to know that you have a methodical approach to problem-solving. Consider the skills and qualities that help you successfully face problems. Perhaps you have a keen eye for detail. Maybe you can see opportunity when others can only focus on the issue. Share your strengths as a problem solver, and your ability to come up with innovative solutions. Strong problem solvers are: - Systematic thinkers - Open minded - Okay with being wrong sometimes - Always researching and exploring - Able to identify the core problem - Able to reverse engineer a challenge to avoid future issues - Able to come up with multiple avenues that work well for all stakeholders - Are do-ers and not worriers

"I am a great problem solver because I can compartmentalize all aspects of a problem before studying it. I also like to bring more experienced team members in to add to the solution. I will never try to be a hero and solve a complicated problem without tapping into the resources around me."

"What makes me a great problem solver is that I have a keen ability to research, read, and explore so that my recommendations are based on fact and study rather than guesses."

"I have been told that I am an excellent problem solver and I believe this is because I have a bit of an engineering mind. I can take the issue, work backward to solve it, and then use that resolution as a basis for avoiding future issues to come up. I am also a big-picture thinker which allows me to come up with various resolutions per problem."

"I am a great problem solver because I do not allow stress to cloud my judgment and mute my creativity. I am a keen observer with a great memory which allows me to recall unique solutions or ideas."

"I am a great problem solver because I draw from the experience of others, whether solicited advice or through my prior observations and then I improve upon that, where possible. My memory and years in the industry have exposed me to many types of situations and problems, so I feel I have a vast amount of experience to draw from, allowing me to be creative and effective in the way I approach any challenge. Not to mention, I'm not afraid to ask for help or advice along the way. I know that I don't know everything, so I like to ask for input when I feel I am not fully equipped to do the job alone. There is no shame in that."

"I believe I am a great problem solver because I am sure to gather as many facts as possible, I look at the problem and its potential solutions from multiple angles, and I am not afraid to make a creative decision, that might seem off the beaten path."

"I consider myself a great problem solver and believe my skills are in my emotional intelligence. I can be really in tune with the tone of the group, who is feeling what, and how they are each best reached. This skill applies to both adults and children, so it is beneficial both inside of the classroom and out! By being aware of what is at the heart of the matter and how each person needs his or her needs met, I'm able to accomplish a lot while avoiding many common landmines."

Anonymous Interview Answers with Professional Feedback

Anonymous Answer

answer in problem solving

Cindy's Feedback

3. Tell me about a time when you discovered a problem and went beyond regular expectations to fix it.

Your innovative approach may be exciting and unconventional, but can you implement it realistically? Ideas are one thing, but putting them into practice and providing measurable results is where you can add genuine value. Think of a time you worked long hours and made sacrifices to overcome a challenging problem. Demonstrate your impact and the significance of your solution.

"During our busy tax season I noticed that one of our primary spreadsheets was not formulated properly. I am not an expert with Excel; however, with everyone being in peak stress mode - I decided it was something I could learn on my own. I watched a few online tutorials and ended up resolving the issue without the need to involve the rest of the team."

"When I worked as an admin assistant at my last job, I was in charge of purchasing office and kitchen supplies. I noticed we had been spending quite a bit of money on paper and plastic-ware. I compared the cost of disposables to the cost of buying permanent dishes and utensils for the kitchen. It turned out we were able to save the company hundreds of dollars each year by simply investing in dishes and silverware!"

"I had a staff member who was stealing supplies. Rumors were going around that she was dishonest; however, there was no evidence. I carefully waited and, after two days, the rumored infractions were caught on camera. At that point, I was able to terminate her employment. I went beyond regular expectations by gaining evidence before terminating her. I knew this would prevent a human resources issue down the road, and it also saved my company from having to pay this employee any severance pay."

"Our agency performed a major client launch last month that tested well. Upon implementing, I noticed that their new website was not functioning correctly. I wanted our client to be happy with our services, so I worked late into the night with our IT team to troubleshoot the site and ensure that by morning, there were no more kinks to work out. In the end, our client was thrilled with my dedication, and they ended up writing an amazing review online and even mentioned me in the review!"

"I managed a coat department previously and, depending on the season; these coats were very high ticket items. I had two salespeople who were consistently battling for the sale. It was unbecoming, to say the least, and impacted the department's morale. To incentivize everyone to go for the sale, I made a sales contest on non-coat merchandise. The more items they upsold, despite being a smaller sale, the more tickets they received towards various other compensation incentives like gift cards or extra time for breaks. The other sales reps felt reinvigorated, and it pushed my two coat-fighters to step outside of their perceived territory."

"In my first role, there was a regular lane of shipments that was difficult to cover. The issue didn't cause us to fall short as far as the customer was concerned. However, we were in danger of potentially having the customer poached due to waiting times. After several late nights attempting to come through for a key customer, I got tired of running in a hamster wheel. I decided to find some carriers that could assist. Long story short, after staying late many days and making some creative calls to find a backhaul, I was able to secure a new carrier, at a great rate, and keep the customer happy."

"When I was reworking lesson plans, I noticed that there was a gap between the programs and some policy. So, rather than hand them back to the team to fix, I took it upon myself to write the remaining lessons and tweak the existing ones to make them cohesive. It took about seven days of working on my own time, but it was worth it when I saw the lessons in action during the school year."

4. Tell me about a time where you had to analyze a set of data and then make a recommendation.

Talk about your attention to detail and sharp focus when it comes to data and statistics. You may not consider yourself a highly analytical person. However, this is a skill that you have indeed exercised in the past.

"I worked for a financial firm last year and had a client who was looking for investment recommendations. I gathered data on the stocks they were interested in, sorting through 12-month trends and further historical data to determine the most promising returns. The client was happy with my findings, and my manager was quite impressed with the research that I conducted."

"My boss recently asked me to make a case for Oracle on Demand versus SAP Business ByDesign. Our business was growing so fast, and we needed a new CRM fast. I called both companies who took me through a webinar and a couple of online tutorials. I then gathered the data and made an informative PowerPoint presentation. My boss was very impressed with how thorough I was, and I was happy to learn something new!"

"Each time I onboard a new client, I analyze a set of data before I make any recommendations on their strategy. This data includes their current analytics, primary sales sources, key customers, and more. I have a formula that I follow for the most part to help me assess and then give the best strategic recommendations that I can."

"My current employer wanted to know the exact impact our social media campaigns were making. I gathered our Facebook analytics for him and created a short PowerPoint presentation from the data. My recommendation was to increase our keywords in the geographical areas where our ads received the highest click-through rates. My research and recommendations certainly helped as our Facebook reach grew exponentially."

"As department manager, I'm responsible for forecasting what our sales will be for the upcoming season so that our buyer can accurately purchase the proper inventory. I have to look at our current inventory, last year's trends, YOY growth, and what the industry is doing as a whole, especially with the impact of online retailers. I then make a recommendation and forecast that will either set us up for success or not. If I under or over forecast, we end up with not enough inventory or too much to sell through and the cost is either opportunity in missed sales, or having to discount unnecessary items. To date, I've been nearly exact in my predictions."

"When doing annual reviews with my clients, I would analyze the past year's shipments, trends, and overall data. I would then make recommendations for improved efficiencies, rates, and better service contracts in the upcoming year. I would make not only carrier recommendations based on service level and pricing, but also made suggestions on new routes or ways in which we could be creative, like consolidating the shipments in our warehouses, to save cost when possible. I managed two of the most significant accounts in the office, so my recommendations were fundamental to our bottom line, and I'm happy to report that they were consistently adopted, resulting in more business."

"I am responsible for analyzing the results of our unit tests given across the department quarterly. I had not only to compile the results and make recommendations as to what units to keep and what to remove for the following year but also diagnose what ineffective and how we could remedy that. This task is a critical one as it shapes the future of the department and our efficacy as teachers."

answer in problem solving

Stephanie's Feedback

5. When a problem requires a quick solution, how do you respond?

When it comes to complex problem solving, decisions are not always readily reached. It takes practice, experience, and confidence to learn what sorts of decisions yield the best results. Walk the interviewer through your process when it comes to making quick decisions. Do you rely on past experiences? Perhaps you go with a gut feeling. Maybe you have read case studies that you lean on in these instances. Problems that require you to act quickly can be emergency situations such as knowing where the fire extinguisher is and grabbing it fast enough to put out a small grease fire in the company kitchen. Other quick decisions could be if you are asked to take on a new responsibility and are only given five minutes to decide if it's something you are prepared to take on. Going with your gut is a skill, and the more you learn to trust your intuition, the easier it becomes to make these types of decisions. Demonstrate that you are confident and able to react swiftly when the need arises.

"Our Controller recently came down with pneumonia on a week where we had a major client presentation to give. He sent me what he had prepared, and I had to fill in the blanks. As an Analyst it was a bit out of my wheelhouse, being in a client facing role, but I adapted quickly, and reminded myself that my team needed me."

"When an urgent problem arises at work, I always try to respond in a calm and assuring manner. I am a natural leader which means that my team often looks to me for answers. One instance of my fast-thinking was just last week when we had an administrative employee no-show on a significant day for us. I called a temp agency, and they had the position filled in just one hour."

"In logistics, there are often split-second decisions that can either get the freight to a customer on time or cause a shut-down of a production line. Sometimes, these decisions have to be made after hours. On more than one occasion, I've received a phone call from our central dispatch asking me how to handle a late driver. I have to remember the details of the particular shipper or receiver, my customer, and the actual load in question but also get creative with how they can make sure to meet customer expectations. Due to the urgent nature of the business, as well as the drivers, it has to be a very quick decision to be successfully resolved. Luckily, due to following my gut, I've been able to make very fast, split-second decisions in the best interest of the branch and customer."

"As a Marketing Director, I need to make a multitude of decisions, on the fly, for varying projects. I rely partially on the instinct that I have built as an expert in the marketing industry and part in past experiences that may be similar. I am sure always to exude an air of control when making decisions."

"I thrive under pressure and always have, so when I'm given a time-sensitive situation to address, I light up and get down to business. I am more impactful and even more creative when I have little time to do much besides jump in and take charge. This ability to make fast decisions is especially helpful in my role as manager when there is an inventory, personnel, or customer issue."

"Just like with negotiations, I react swiftly in emergency situations. Perhaps my skills come from my years as a parent, having to think fast and put out fires! If a quick solution is required, I will do a fast overview of the facts and make a decision based on risk factors considering the potential financial loss."

"I am certainly a take charge and tackle a project kind of gal - as a teacher and a mom, too! I feel I have a powerful and accurate intuitive sense and I follow it instinctively. It's very rarely steered me wrong."

6. When it comes to problem solving, are you a strong collaborator?

Show off your teamwork skills by giving an example of when you successfully collaborated with your coworkers. Be sure to demonstrate how you communicated your thoughts or opinions. Highlight how your contributions, or ability to ask for help, made a difference. Explain how you are a team player who enjoys working alongside others.

"Last month, I recruited a couple of coworkers to help me solve a problem for a client. We were looking at their financials, but something didn't add up, and I didn't have the analysis background that these two co-workers had. Together we molded our areas of expertise and created a bulletproof financial plan for our client. I enjoyed the collaboration and would do it again in a heartbeat."

"I am most certainly a strong collaborator! Being an executive assistant, I am often in need of strong collaboration to complete a project for the VP who I support. I love learning new things from my coworkers and those who I report to."

"I love having impromptu brainstorm sessions with my team. It keeps everyone on their toes! When an issue comes to light, I will approach the problem with the entire team and open the floor, at the end of the meeting, for suggestions."

"In marketing, it is imperative to collaborate and gain different sides of the story, and new opinions. I try to seek out my team's opinions on projects all the time. I find everyone has something to contribute and can help me see a problem or strategy in a way that I may not have ever considered."

"I would consider myself an active collaborator and believe that two heads are almost always better than one. Three is the best, in my opinion. This way the team is odd-numbered, so if there's a dispute you can take a vote on it! Multiple viewpoints are almost always a great idea."

"I am a strong collaborator. I am always willing to listen to others' opinions, hear their perspective, and work together to build a solution that will fit for everyone. I am always looking to draw from others' experience and expertise to bring about the best solution for the client and the branch as a whole. When drafting a pitch for a client, I am always sure to bring on a manager or carrier sales rep so that I will have multiple perspectives to help bring us to the best collaborative solution."

"I believe I'm a skilled collaborator and am confident that my coworkers would agree. I come to our bi-weekly department meetings full of ideas and with an open spirit, ready to collaborate with the rest of the team. We always have engaging discussions that result in great takeaways for the teachers as well as our students."

7. When you cannot seem to find the right solution to a problem, how do you deal?

Sometimes, problems just seem too impossible to solve, at first glance. Your creative problem-solving skills may be at a stand-still from time to time, and the interviewer wants to know how you deal with that. Taking a brief break and stepping away from the problem can help you to see things from a different perspective. When you are in a rut, you can waste time plugging away at something, resulting in a decline in productivity. Discuss with the interviewer how you handle being in a rut like this.

"If I am stuck on a particular problem, I will take a break from trying to figure out what's wrong and ask a coworker for advice. Getting another person's perspective when you start to feel like you're hitting a wall can help one to see a problem with a fresh set of eyes. As humans, sometimes we overthink! The biggest hurdle can be asking for help, and I am not above asking for help when I'm stuck."

"If time allows - I will sleep on it! When faced with tough decisions where an answer does not come to me easily, I will take a moment to feel the issue out. When necessary I will also bring in the opinion of the administrators in a different department."

"If I cannot come to a solution that feels right I will check in with other leaders whom I work with and, depending on the situation, my business mentor. It's important to check in with those that I admire as they have unique ideas and some have more industry tenure as well."

"As a marketer, I am hired to find the solution for others. As you can imagine, when that solution seems elusive, it is incredibly challenging for me to accept. For this reason, I love brainstorm sessions with my team. I will also look to the outside in the form of resources online such as blogs and forums by other marketing professionals."

"It can be frustrating when a solution does not come fluidly. However, sometimes trying a solution and seeing it fail, will lead you to a lightbulb moment. I am an active person, so I like to walk and talk things out. Usually, as I do that, I don't filter my ideas. This way, something slips out that I would have edited out as "ridiculous" if I were writing down a list. I have found that this free-flowing problem-solving session often leads to the most creative and impactful solutions which I would have nixed from the get-go had another not failed."

"If I'm stuck on a problem, I try to take some time away from the issue, ideally by taking a step away from the screen and get my blood flowing. Walking away seems to help me get reinvigorated and more creative. I also find it valuable to talk it out with someone, even if that person is not a stakeholder in the situation."

"If I am stuck in a rut or can't seem to figure out the best approach, I am fortunate enough that I have so many other tasks and classes that I can focus on. Usually, if I clear my mind and fill it with something else, a great idea hits me when I least expect it. If I am stuck on a problem and cannot take the time to step away, I usually rely on my students to help me shake it off!"

8. When faced with a problem, how do you decide on the best solution?

There may be more than one solution to a problem, and the interviewer would like to know how you make a final choice when you're in a situation like that. Effectively comparing and contrasting, or weighing the pros and cons, is essential when choosing the best way to solve a problem. The interviewer wants to see that you are capable when it comes to calculating risk vs. reward. Think about a time when you have compared the risk and reward to a potential solution.

"If I have a problem with multiple solutions, I always go back to the classic pros vs. cons method. I fully understand that although no solution is perfect, and some solutions offer lesser sacrifice while others pose potential loss. I have been trained to take the solution that is 'closest to the money' which means that if I am stuck between a rock and a hard place, I will choose the solution that is most beneficial to the company's bottom line."

"When it comes to problem-solving, I will always weigh the pros and cons before making a decision. I will also bounce my thoughts off of some co-workers if I still feel conflicted after that."

"My decisions are always based on three factors. One, what is best for the company. Two, what is best for our clients. Three, what will boost employee morale. Now, not all decisions will be popular with all three groups, and I do keep that in mind. In those instances, it is my job to watch our bottom line but ensure customer satisfaction at all times."

"Rock, paper, scissors! Kidding - of course! Our team will collaborate on tough decisions, and we often vote. Majority wins in our office for many creative decisions."

"When I face a problem, I am sure to draw on previous experiences both as a customer and an employee in retail. I then use these experiences to make the most informed decision that I can about the problem at hand. Generally speaking, if I've already seen or experienced a very comparable situation, I can be impactful and exact in my approach by drawing from those experiences."

"As I consider a problem and its solutions, I make a note of what my gut tells me what to do. Then I take a step back and reflect on times that I have faced the situation before. I recall the actions that I took, the outcome, and then pivot as necessary. I trust my instinct because I am heavily knowledgeable in this industry, but I believe in relying on fact as well."

"I am typically a follow-my-gut type of person, so I follow my instinct when possible. I make a note of what my initial inclination was and then I make sure to compare and contrast solutions. Once I have identified the best solution, I check in to see if it feels right. More often than not, my initial instinct is correct. Of course, I am sure to be analytical as I weigh out each decision."

9. How do you prioritize multiple projects when they all seem equally important?

Prioritizing is a skill that requires practice. There are many approaches you can take. Here are some suggestions: 1) Make a list. By thinking through and writing down each item that needs completion, you can see it on paper. 2) Mark what is urgent or essential. Take into account deadlines and meetings. 3) Order each task based on effort and estimated value. 4) Consider due dates and how long it will take to do each item. When answering this question, show the interviewer that you have a system in place that helps you to think through what needs to happen, and when. The better you can prioritize, the more productive you will be, making you an asset to their company!

"I aim to be as effective and efficient as possible and make sure I can use all minutes of a day for a project. I have a few things going at once most of the time. I am the lead on some, the delegator on others, and the reviewer on another, for instance. This way, by splitting up the work to the appropriate parties, both my team and I can be the most efficient with our time."

"I often have multiple projects due at a time, since I am the assistant to three different executives. I ask my executives to rank their need from 1-5 in the level of urgency, including its due date. I start my work on that list. If there is more than one urgent need, I will work overtime, or through my lunch, to ensure that I deliver everything on time."

"I had to utilize creative problem solving last month when we found ourselves short-staffed and unable to hire new employees due to budget cuts. I changed our schedule to include some split shifts and received approval for a small amount of overtime spending. The problem is solved, at least temporarily, until our company comes out of our spending freeze."

"In my current department, we are very systematic in our customer delivery promises; however, that is not to say that doubling up on client deliveries does not happen. When situations occur where I have to prioritize, I will do so by the size of the client and budget. It may seem unfair at times; however, our largest clients with the most significant spend always rule out."

"I prioritize based on urgency and time required for the project. I have a list of what needs to be done, by when, and how long I estimate that it will take to accomplish. I am great under pressure, but try to make sure that I don't get myself or my team into a sticky situation by not allotting enough time for any particular project."

"I love to keep running lists of everything that I need to do, big or small. Mostly because I love crossing things off of the to-do list, but also because it helps me keep track of everything. Lately, I've started utilizing a free project management software that I use to make those lists, categorize the tasks, and mark them by the level of urgency. I take care of the most time-sensitive issues first and then move along to the equally important, but perhaps less time-sensitive to-dos. I also estimate how long each task will take, so if I have a few minutes in between projects, I can tackle the quick to dos and use that time effectively, rather than use it to figure out 'what's next.'"

"I follow the tried and true practice of making lists and assigning each item a priority and tackling the list that way. I love to check things off my list, as it gives me a feeling of accomplishment. Also, I am a believer in following my intuition. If I feel that something lower on the to-do list needs to be bumped up in priority, I will tackle that right away. As a teacher, there are always a lot of simultaneous to do items, so in addition to prioritizing, I have to be good at multitasking; something I find I do quite well as both a teacher and a mom."

answer in problem solving

Marcie's Feedback

10. Tell me about a recurring problem that you run into in your current position, and how you handle it.

The interviewer wants to see that, despite this recurring problem, you take action to find a resolution. They want to make sure they aren't hiring a chronic complainer who is easily defeated! Be careful to avoid complaining about your current (or most recent) position. A recurring problem could be a glitchy software system, an employee who is regularly late, or even an unpredictable work schedule. Remain optimistic in your reply!

"A recurring problem that I have in my current position is the fact that our client management software is not user-friendly. Any entry that I need to make is incredibly time-consuming which poses a real problem when a deadline is present, or when we have clients waiting for an answer. I have found that the best workaround for this is not to allow my paperwork to build up. The more proactive I am, the better I can keep ahead of schedule."

"A problem that I am currently running into is a lack of office supplies. My boss has been running very lean, financially speaking, since our industry took a downturn. I have to time my ordering with client invoices at this point. This situation has certainly helped me to become more aware of spending and budgets, that's for sure!"

"The greatest issue in my current position is that we have so much employee turnover. It started to feel like I was constantly training new staff. I came up with an employee referral bonus program which greatly helped. For every successful referral, our employees get $400 plus another $400 after their referral stays for three months. I believe this has been successful because the quality of our employees has greatly increased."

"In my current office, we have more clients than we can handle - which is a great thing! However, it's been tough to find the best marketers to join our team because we are a small organization. This hiring situation has meant a lot of overtime hours, which I am certainly happy to do for the most part. I do look forward to working with a bigger team, like yours."

"Unfortunately, a recurring issue in my current company is employee tenure. It's just really part of the industry as we need some holiday and seasonal associates and they typically don't want to stay on, or we don't have the budget to keep them on. This turnover means we are continually becoming a new team and learning how to work with our new coworkers. Scheduling often has a learning curve with a new team, too, because you have to take into account the availability of all parties, and who works well together. That said, it's something I'm used to. I make it a bit of a personal challenge or game for myself. How quickly I can learn their available days, how fast I can learn who works best together."

"A recurring issue at my current job is lack of reliable inventory that my clients are requesting, which can be incredibly frustrating. I am working hard to land a client, get them to buy into our program, both literally and figuratively, and then we fall short of expectations when our inventory doesn't meet their standards. That said, I continue to go out, land new clients, and try to source the proper inventory for them."

"A recurring issue revolves around my lack of a classroom and the friction that can arise at times because of it. Without the flexibility of my own classroom, I sometimes find myself in an awkward situation since I have to abide by the other teacher's rules, which sometimes conflict with mine. I do my best to follow the teachers' class rules, and make sure that we have a good understanding."

11. Tell me about a time when you failed to solve a problem. How did you overcome the failure?

'Success is bouncing from failure to failure without losing momentum,' or so they say. Your resilience shines through when you can learn from your mistakes and keep going. Give an example that shows you can accept fault and learn from challenging experiences.

"I failed to meet an important deadline in my first job out of college because I didn't know how to prioritize properly. I kept letting other menial tasks get in the way rather than focusing on finishing the project. I learned how to manage my time wisely by setting reasonable goals and reminders on my calendar. This technique helped me to manage my time more effectively."

"Last month we were having issues with our GoToMeeting application, and it was right before a major client meeting. I was on a call with the service provider, trying to troubleshoot and unfortunately, did not deliver a fix on time. After the initial frustration, I decided to talk to my boss about having backups in place. Now, we have Skype, and Google Hangouts set up for these emergency situations."

"I was asked to solve our issue of employee turnover which ended up being much more difficult than I originally thought. My initial goal was to improve turnover by 70% but in the end, only reached 40% improvement. Although I did not reach my goal, I am still happy that my action plan made a difference."

"I had a customer who was not happy with my delivery, and I chose to take care of the situation without involving my boss. It wasn't that I was trying to sweep the situation under the rug, I just honestly thought I had been successfully dealing with the situation on my own. Unfortunately, I was wrong because the client sent a nasty email to my boss a short time after. I should have gone to my boss right away and filled him in. It's something that I've learned from, and I'm ready to involve my boss with every sticky client situation."

"In a previous role as a personal shopping assistant, I was tasked with taking on a notoriously difficult client. She spent a lot of money in the store in the past but was very demanding. This challenge seemed like the perfect opportunity to prove myself. A few months in, I made the misstep of mentioning something she'd complained about at an earlier date. Apparently, she was offended that I brought it up, even though I meant it very innocently. I owned up to it immediately to my manager and came up with a plan to win her back. I wrote a snail-mail card apologizing to her and let several weeks pass before reaching out in any other way. By the time I did, two months later, she was perfectly lovely, dismissed my apology as though she didn't know what I was talking about, and we moved along in a better fashion than we had prior."

"In my first role out of college, I was working to solve a lane issue with a carrier that kept falling through. I went through every solution I could come up with including pitching consistency, to leveraging my current relationships, and asking for favors. Those favors and workarounds ran out, and we fell short of client expectations. While I did all that I knew how at the time, I still fell short, and it was disappointing. In retrospect, I would have involved more people in supervisory positions earlier on in the process to learn from their shared experiences."

"The problem I've failed to solve that still keeps me up at night is a successful inclusion of one of my students with an IEP. He loves Spanish and in a one-on-one setting excels at it, but cannot handle the behavior expectations in class because he gets too excited. I've tried multiple approaches to get him to regulate, and participate, but so far nothing has allowed him to participate in the class without disrupting the other students and causing a meltdown for himself. This fact weighs on me since I want him to experience inclusion at all times. As a result, he comes to my office a few days each week, and we have our Spanish class together. I feel this exemplifies who I am as a teacher. I will go the extra mile for my students to make sure they get their fair shake at life."

12. What sources do you look to when you need to solve a complicated problem?

The interviewer wants to know that you can think outside the box, or even ask for help when you are stuck on a complicated problem. Maybe you look to a mentor or boss for advice. Perhaps you have handbooks, manuals and systems you turn to for help. Offer some relevant examples based on your industry. If you work in the medical field, you may turn to textbooks, online research, colleagues or even patient's history to find the right solution. If you work in customer service, you may ask the customer what they need to find the best way to solve the problem. Show the interviewer that you are knowledgeable and equipped to handle these types of scenarios.

"When I am faced with a complicated problem, I will look to the resources that my current company has provided me. The answer is almost always in there. If it's more of a moral dilemma vs. a knowledge-based dilemma, I will ask my supervisor for his thoughts and opinion since I value him as a mentor and expert in our industry."

"I have a variety of manuals and online tutorials that I lean to when I need to solve a complicated problem. Usually, the issues are surrounding Excel troubleshooting, so it is easy to find answers without involving anyone else and interrupting their day."

"I have a business mentor that I turn to for significant problems. She and I are in the same industry; however, she is much more tenured than I am. I recommend that everyone have a mentor. Even though I run a team of my own now, there are times when I do not have the answers."

"When I need to solve a complicated problem I will turn to marketing forums and blogs that I follow. There is a plethora of information on the internet, and it would be a shame not to take advantage of them!"

"To solve a complex issue, I will reach out to a manager or mentor from a previous role to ask them how they've handled such issues in the past. I am always ready to dive back into our handbook, but these types of scenarios are often not covered there, which is why I value a human, experiential approach. I know that there are so many folks in the industry who have so much to teach me and have probably already "been-there-done-that," so I love to utilize them as a resource."

"If there's a complicated problem, I'll write out what I think the possible solutions would be. Then, I will weigh those potential solutions against one another and list the complications that may arise as a result of each choice. Also, I am always open to input or suggestions from those with more experience than I. I will often turn to my organizations' training resources, as well as talk the problem out with coworkers or my boss."

"I have a vast cohort of teachers with whom I work currently, or have worked in the past, so if I am stuck on a problem or feel I need some additional help, I reach out to these educators. If nothing else, they're there to lend an ear and let me bounce my ideas off of them. They almost always have some real-life experience in a very similar situation. I value this collaborative, supportive group that I've amassed over the years."

13. After you implement a solution to a problem, how do you test the effectiveness of that solution?

The interviewer wants to see that you have strong follow-through skills and the ability to use data and analytics to support your decisions. The only way to test the effectiveness of a new solution is to keep a close eye on the immediate, and often longer-term, results! Depending on the situation, you can use data, run reports, and compare/contrast your findings. If you have records of the data before your problem-solving solution, you can track the results of your new solution and analyze in a month, or beyond. It can take time to see the results, so having a method for measuring them is essential. Give an example of a time you implemented a solution and found a way to measure the results to check its efficacy.

"Last year, our company was having a very high rate of turnover due to employee burnout during overtime hours worked. I implemented a third shift which alleviated the need for excessive overtime. Yes, it did increase our payroll costs by 33%; however, it decreased our turnover which was costing us more and more every year. From the analytics I have been watching, the change will pay for itself by the end of year two."

"One solution that I recently implemented was the use of Google calendar with the executive that I support. She was rarely updating her Outlook calendar because she found it to be too difficult to do on her smartphone. Since this implementation, we have minimized our crossed wires significantly! I have measured the effectiveness of this new calendar strategy by marking down any appointments that need to be rescheduled. So far, for the month, the number of reschedules is down by 80%."

"I always look at the data to gauge the efficacy of policy or new solution. I am big on numbers as they do tell the full, and true, story. I love the reliability of spreadsheets and numbers!"

"Once our team comes up with a new marketing strategy for a client we will conduct two focus groups. One test group will be on the original marketing plan and the second, on the plan that we want to pitch. The use of focus groups is the best way for us to measure if our new strategy will be effective enough to justify the changes for the client."

"I like to collect data, as well as anecdotal assessments of new policies. It's great to have data to confirm if it was or was not effective, but I am a firm believer, too, in getting the team on board. Plus, as you implement a solution, sometimes those doing the actual day-to-day work with customers or in the actual implementation have a more accurate understanding of what's going on or what could be improved. Therefore, I am sure to ask the staff how they think it's going, if it's impactful, or what they still see as an area for growth."

"To test the effectiveness of any solution, you have to be objective and see if it genuinely addressed the problem it set out to solve. Everything in our business runs on KPIs, so when we introduce any initiative, we can see how it is or is not impacting those measurements. One example of this was when I assigned specific accounts to my team of buyers, instead of just attaching as they came up. The idea was to get a buyer to become an expert on that account, their buying habits, and therefore be more effective in the long term at sourcing for their needs. At first, it didn't seem all that impactful, as the close rate was still around 42% overall. However, over the course of 10 weeks, we saw an uptick in close ratios on the assigned, dedicated accounts versus the randomly distributed ones, resulting in 53% close ratio. It's something that became so effective that other sales pods adopted it as their practice as well."

"For me, numbers play an important part in teaching but do not paint the full picture. So, after implementing a change, it is certainly important for me to collect data from our unit tests to gauge the efficacy of the lessons we're teaching and the lesson plans we are using. However, I also am sure to check-in with the students on a more regular basis to check for comprehension. Testing is only truly reflective of the way some students learn, whereas others are terrible test takers, even though they've learned the material. That is why I like to take a two-pronged approach."

14. When a major problem arises, what is your first reaction?

The interviewer wants to know if your reactions to problems reflect maturity and professionalism. How you react will significantly determine how you fit with their existing team. Perhaps your computer crashes, and you realize you may have just lost all of your hard work. Or maybe you are limited on time and have a deadline rapidly approaching. Demonstrate to the interviewer that you take a very methodical approach to problem-solving, rather than reacting impulsively when a problem occurs.

"When a major problem arises, my first instinct is to take a step back and absorb what just happened. I then go into 'brainstorm' mode, jotting down potential ways to resolve the issue. From there, I can use a pros and cons list to determine the best course of action for a fast and amicable resolution."

"I have taught myself to become much calmer with my first reactions when problems arise. Now, I will step back and review my options for solving the problem rather than allow myself to become frustrated. If I feel that I cannot solve the issue on my own, I will ask for help from my superiors."

"Depending on the situation, I will gather my resources and team and collaborate on making the necessary happen on a shortened timeline so that we can deliver our results in the most efficient manner possible. Usually, we learn something about ourselves, the team, or a more effective approach to the next problem in the process."

"When a major problem arises, my first instinct is to jump in and fix the issue. I am a do-er and also think in a reverse-engineering manner. I start with the desired result, and work my way backward from there, figuring out where the snag occurred."

"I am resistant to stress but cannot completely avoid it. When a major issue arises, I will take a quick walk, if possible, so I can best assess how to address the issue while clearing my head. Then, I get to work. I delegate whenever possible so that I can oversee the effectiveness, but am not at all afraid to jump in and do the dirty work myself."

"In the event of a significant problem or setback, my first reaction is to freeze in disbelief for a moment or two while I gather myself, then I jump into action. I know that I need to work harder and faster to recover the time and effort lost. My salesperson mind goes into overdrive until the issue comes to a resolution."

"My first inclination in the event of a major problem is to roll up my sleeves and jump in to fix it or help mitigate some of the potential blow out. This initial reaction is especially true when the problem involves a student's feelings or wellbeing."

15. What steps do you take when you have to make an immediate decision without all the relevant information?

Sometimes we have to make decisions without all of the pertinent information at our fingertips. The interviewer wants to know that you are capable of taking educated guesses and that you are confident enough in your abilities that you can make a firm decision without all pieces of the problem being present.

"When I need to decide without all of the information, I weigh the pros and cons and come up with a solution that makes the most sense. Common sense can take you a long way! Next, I may ask the opinion of someone I trust to see what they think. Even though I trust my decision-making ability, I still think it's important to get a second opinion when it comes to situations involving money or decisions that make a significant impact on others."

"Being organized, I do have a checklist that I follow on all policy-related decisions and changes. If I do not have all necessary information to make an important decision, I can usually find answers in our company resource database, or I will consult an administrator more tenured than I."

"Immediate decisions are required of me on a daily basis. For instance, what do I do when a forklift driver doesn't show up for their shift? How do I react to a chemical spill in the warehouse? I find that the most effective method for making immediate decisions is to forget about what you don't know and focus on what you do know. That's the best anyone can do, and there is no sense wasting time on the what ifs, especially in my industry when the safety of others could be at risk."

"In my current company, we have a rule always to do what will make the client happiest. So, when I am in a situation where I need to make an immediate decision on a client file, I will ask myself what I would want if I were the client. Then, I jump into action to make that happen."

"Often when a customer is worked up, I only have a piece of the puzzle to go off of, whether because they haven't given the full story, or I'm pulled in by the associate who heard the full story. In either case, it's something I'm accustomed to and deal with daily. As far as customer problems go, they tend to follow the same general pattern, so I assess quickly what category the problem seems to fall in, and go from there."

"I am a strong believer in following my gut, and for the most part, it has not steered me wrong. I try to gather as much information as possible, but when all of the pieces are not accessible, I assess the situation using my prior knowledge of similar situations, and I follow my intuition. If I'm not certain or feel conflicted, I don't hesitate to bring in another person to help me come to the best decision for the company."

"I feel comfortable making an immediate decision, even if I don't have all of the relevant information, for the most part. I have great confidence in my situational knowledge as an experienced educator. One example that comes to mind was the class when there was a behavior outburst. I immediately leaped into action to diffuse the situation the best way I knew. By acting quickly, I can prevent the situation from further escalating."

16. How do you deal with distracting coworkers who stand in the way of your progress?

Even the most well-meaning coworkers can distract you from getting things done at work from time to time. The funny and entertaining coworkers who like to chat online and send YouTube videos are often the ones who can get in the way of your productivity if you let them. How do you respond? Show off your ability to set professional boundaries, when needed.

"I typically just set a kind, but clear, boundary and tell my coworker that I need to focus. I will offer an alternate time for a catch-up, over lunch for example. It is important for the sake of workplace culture to set aside time to be social with coworkers, so I usually just let them know when I'll be available for a quick break in the day."

"I understand working relationships are significant, and I'm sure to make time for them so that I can be useful but also enjoy myself at work. With that said, I know where these relationships fall regarding prioritization of my day. I make sure that others know that, too, without being off-putting."

"I am always interrupted by my team - that is par for the course being a manager. To deal with any lost time, I will simply stay late or come to work a bit earlier the next day. My day is unpredictable, and I have accepted that fact."

"I am very open with my colleagues and will let them know if they are a distraction. Currently, I can take my work home as well so if there is a part that I cannot get past due to distractions; I will take a day to work from my home office."

"I try to make the workplace as fun as possible, within reason. I love to make it a place people want to go to, instead of dread. That said, there are always the people that ruin it for the rest of the team by taking advantage. To combat this, I make it very clear what the expectations of allowed and prohibited behaviors are, and am sure to reinforce those expectations."

"There are always going to be co-workers who are there for the gab, rather than the work, or who are content just being in their position with no intent of advancing through the ranks. Early in my career, this bothered me. Why weren't they motivated to grow and learn? Then, I realized that it's important to have those people since a company can't have all its people always vying for the top. If there's a distracting coworker, I try to make my priorities clear and engage kindly and courteously with them as humans, and then get back to work. I am sure to remain friendly, while also firm, as needed, to communicate that I am here for work first as a priority."

"Very rarely do I find that my coworkers successfully distract me- even in a department meeting, I find I'm able to remain on task. I was always taught to ignore the behavior you wish to cease. If my coworkers are distracting and seeking attention, I try to ignore it as much as possible and only address it when it's detracting from a productive work environment."

17. Tell me about a time when you had to troubleshoot to solve a problem.

Troubleshooting is like reverse engineering - it takes skill, effort, and patience. You have to understand the problem to know how to work backward from it to find a solution. Knowing how to solve problems with technical equipment is always a solid skill, and a great way to demonstrate your example. Show that you are insightful in your approach.

"Last week, while operating the ultrasound machine, I was receiving a repeated error. I entered in a few different codes, but that didn't solve the issue. I then did a hard reset, removing all power sources. Then, I referred to the online manual for additional suggestions. It took a little time and patience, but I was able to resolve the issue without calling a technician."

"We do not have an IT department in my current office so whenever an issue arises, I am the person that my team calls. Troubleshooting is fun for me - it's like a new challenge every time. Google and IT forums are often my best friend!"

"We had a major complication in our system and our entire production line shut down. Our network administrator could not be reached so I had to go old-school and manually enter the orders so that my team could continue with production. The entire debacle lasted half of a day, and my system worked well as a placeholder."

"One of our clients called me in a panic, saying that Facebook rejected their ad campaign that we so carefully crafted. I researched on ad policy forums and learned that it was not approved because we did not set our demographic targets to people only over the age of 21. The ad was for a craft beer company, and we did not put into consideration the legal age in most states. Once I was able to narrow down the issue, I tweaked the ad, and it was approved."

"One horrific day at work, our systems went down entirely. We had no backup for how to check customers out, so I had to dig in the deep recesses of the back room and find the card imprint machines, and we wrote out tickets by hand and made imprints of the cards. I tried all the usual tricks to get our registers up, but couldn't get them to come online as it was a network error. I found the way around it with the handprint cards and then opening the cash drawer with a key."

"In a troubleshooting situation, I approach it like a maze and work backward. There are usually multiple factors contributing to any one issue, so I try to discern what they are, weigh those out and try to conclude what the potential best solution is. As far as technically speaking, my go-to in many situations, as rudimentary and childish as it may be, is often turn it off and turn it back on. Ha. I know it sounds too simple, but it often works best."

"I do everything I can to test out the technology before I bring it into the classroom- the day is so packed that we don't have any time to spare on figuring out technology if it acts up. I also always have a backup plan in mind in case the smart board or whatever we're utilizing that day doesn't cooperate, so we don't lose precious learning time." However, I believe that troubleshooting applies to more than just technology. Problems that occur offline also need troubleshooting as they arise, including figuring out a lesson plan and how it works or doesn't. It's all about working backward to see what issues, if any, may arise in its implementation during a dry run. By preparing in advance and being aware of what issues may come up, I'm able to flush out problems that would have otherwise arisen during the class time. "

18. Tell me about a time when your analysis of a problem was deemed to be incorrect. What would you have done differently?

Everyone makes mistakes when analyzing a situation. The interviewer isn't concerned with perfection; instead, they want to know how you deal the aftermath of rejection! Sometimes you can't correct your mistakes, but you can certainly learn from them. Highlight your ability to learn from your mistakes and move on, professionally.

"It was my first job as a physician's assistant, and I was trying to diagnose a patient who had severe pain in her abdomen. After running some tests, the doctor and I believed she was suffering from a gallbladder problem. We treated her, but she came back to the ER a week later. It turned out she had a problem with her pancreas. Even though we misdiagnosed her initially, we were able to use this mistake to help us identify the real problem. I've learned that sometimes making a mistake is a part of the process of solving a more complicated problem."

"I was asked recently to work on balancing an accounts receivable report. Math is not my strongest suit; however, I was confident in my ability to make it happen. Through a bit of research, I carefully worked on the document and was quite proud of my result. It turns out, I skipped a few important steps, and my work was, in fact, incorrect. I took it as a learning opportunity but also realized that my strengths are in other areas of business. I should have asked for the project to be placed with someone else, but I do not regret trying."

"We had incredibly high turnover rates when I first started in my current role. Going in guns blazing, wanting to make a strong first impression, I did a complete overhaul of the training manual thinking that was the problem. It turns out the training manual was just fine. The culprit to the turnover was one employee who was a complete bully on the job. The moment I terminated that person, the issue was solved. At least now I have a fancy new training manual! Moving forward, I now poll my team regularly for job satisfaction. I encourage a transparent workplace culture where people feel safe bringing their issues to me."

"I had a client, earlier in my career, who was not seeing the same results from their Facebook advertising as they once did. I changed the headlines, increased the budget, and even did multiple A-B tests. What I failed to see were the strategic algorithm changes that Facebook had made, which directly affected the visibility of my clients' ads. Now, I have alerts and subscribe to a couple of blogs solely dedicated to these changes, so I never miss a beat."

"Unfortunately, this happened not too long ago where I misjudged a customer complaint. The associate needed to escalate the conflict to a manager but did not accurately portray the customer's concern, and I jumped into action based off of the limited information given. Due to not gathering enough information from the customer herself or clarifying the misunderstanding with the associate, I took a misstep with the customer and did not resolve the issue as quickly as I would have liked. Ultimately, I was able to clarify the situation and get to a resolution that worked for everyone, leaving the customer happy. However I have some regrets. It was a learning process, and something I have been sure not to repeat since. Were I to do it again, I would clarify the situation with the customer, rather than taking the associate's word for it."

"When pitching an existing client on increased volume next year, I had made a recommendation on the most effective carrier for a lane. I based this recommendation on historical data and projected future rates. However, a merger occurred after the time of the pitch, and their prices skyrocketed since they were the only viable carrier for that area. Without competition, they didn't have to remain competitive in their rates. While I could not have predicted the merger, I could have quoted out with a higher margin on our part so that if there were some snag like this, we are covered. Since we lock in the rates for the customer, we took a loss each time they moved freight this way. As a backup method, whenever possible, I attempted to send the freight another way, so that we would lose some money but not take as large of a hit. That was a big learning experience for me and has helped me be better prepared to pitch other customers in a more effective, CYA type way."

"While teaching, the kids told me that I needed to quiet down at one point. I assumed it was the teacher whom I shared a wall with, that planted the seed, which was irksome. This type of situation had happened before. This time, however, I was wrong. I asked her to avoid delivering messages to me through the students, and she said she had not. Apparently, the students knew she had a headache that day, so they were all watching their volume level. I was glad that I did address the situation with the teacher, but made sure not to be accusatory or make assumptions about motives again."

19. Tell me about the most challenging aspect of your previous job. How did you overcome it?

Sometimes the most significant workplace challenge is a difficult task that puts you outside of your comfort zone. It could be something that requires skills you haven't mastered yet or qualities where you are not the strongest. Explain to the interviewer why it was difficult but be sure to spend more time highlighting the actions you took to overcome the challenge.

"The most challenging aspect of my last job was troubleshooting some of the older technology. We needed some serious upgrades, but they weren't in the budget. Learning how to work around this problem was quite a challenge, but I learned how by referring to old manuals and online forums. I ended up to become one of the stronger users of this program, in our entire office! I quickly became the go-to person when anyone had questions about the technology."

"In my current role, we have global offices that span four time zones. It is an incredible challenge to be continually calculating the difference in my mind when I call or email on updates for projects, for instance. I now have each locations time added to my desktop, my smartphone, and four individual clocks on my wall. These small and inexpensive changes made all the difference."

"The most challenging aspect of my previous job was the constant need to pivot when it came to trends in the industry. We would gain footing, and then the next greatest product would arrive. It made it difficult to feel loyalty to any of it. I started to express loyalty to the company's ability to discern great products instead of narrowing in on the products themselves. This shift in thought helped with mine and my teams' performance when it came to sales."

"My previous role was with a small agency where budgets were always a concern by the clients. Although I liked the clients, they were usually independent businesses with less than ten employees. They had a hard time thinking big-picture. I overcame this by coming up with a questionnaire that would address their greatest pain points and needs for their business. I would then focus on their small goals versus what I felt their company could be. Some business owners are more comfortable being comfortable, versus ruling the world, and that's okay! I just needed to wrap my marketing brain around that."

"The most challenging part of my prior role was navigating the landscape as the newest manager on the team. I needed to work on gaining the trust and respect despite my being green. I worked hard to build individual relationships with each associate and forge a bond with them. I also shared information about myself, including my experience in the industry, and who I am as a person. I know that this made me more human, approachable, and also solidified my credentials, so I know how to get the team on my side."

"The most challenging part of my previous job was relying on another team to be efficient. I am all for teamwork, but for me to be paid, this team needed to deliver timely and quality work. Meanwhile, their goals and metrics remained disconnected to any sales outcomes. This situation made it tough to motivate them. In the short term, I sat down with them and explained why it was vital for myself them, and the company that we work together on the same timeline. I incentivized them with coffee or store gift cards. Bigger picture, I sat down with the management of both teams and shared the issues we were having, suggesting a solution that would tie their KPIs and financial incentives to our outcomes, to make them invested. In the end, the short- and long-term approaches proved useful."

"I think the most challenging aspect of my current job is the fact that I share a classroom with another educator. Without having my space, other obstacles come up such as teachers trying to influence how I run my class, or them holding small conversations with their aides during my teaching time. I make sure to address this up front with the classroom teachers- that while it is also their room, please treat it as though it were mine during the 40 minutes that I am teaching. If there is ever an issue, I am sure to address it quickly and directly, so we can move past it."

20. When faced with a problem, are you more likely to jump into solving it, or are you the type to carefully assess the issue first?

The interviewer would like to know more about your problem-solving skills, and your personality. Discuss how you tackle problems when they arise, and keep your answer work-related if you can. Whether you are the type to jump right into solving a problem or you are more methodical in your approach, highlight to the interviewer that you are capable of handling issues professionally while using sound judgment.

"When faced with a problem, I am more likely to jump right into solving it. I believe that you cannot leave a problem to fester or become bigger than it already is. You have to take ownership of the issue, and involve yourself in the resolution right away. With that said, I am responsible for my decision making and certainly don't jump in blind. If I am unsure of what action to take, I will ask my leader for advice."

"I am careful and calculated in every step taken when it comes to problem-solving. This effort is because as an administrator, one error in judgment can throw off the timing of an entire project. I would say that I am the particular type who thoroughly assesses situations."

"As a manager, responsible for a team of 18 individuals, I need to be very calculated in most decisions that I make. I cannot act on the fly, or by emotion alone because others are relying on me."

"In marketing, I feel that I often have to do both. Some smaller decisions simply cannot be over-thought and others, especially when it comes to strategy, will need extra thought. I can provide both sides when appropriate."

"I think it depends on the situation at hand, honestly. In a familiar situation, I am ready to jump right in and tackle the problem. However, when the stakes are high, or tension is high, I am more inclined to take a step back, slow down, and be more tactful in my approach."

"I'm a "roll up my sleeves" kind of person. I see a problem, envision a solution, and begin to tackle it, figuring it out as I go and asking for help along the way. I think it can become a 'bury your head in the sand' issue, or the team will have the bystander effect, thinking someone else is going to take care of it, so I jump in and take action. I rally the troops, gather the appropriate supplies or resources as needed, and get to work."

"I'm the type of teacher who jumps in, head first and gets the work done. I know that the longer I wait to address a problem, the bigger it becomes, so I make sure to get right to it. This approach applies to interpersonal issues as well as curriculum missteps."

21. Give me a recent example of a valuable lesson you learned from a problem you faced at work.

One of the best aspects of problem-solving is that you always have the opportunity to learn from the experience. Seeing problems as opportunities to grow, is what makes you an excellent employee! Show the interviewer that you can learn valuable lessons when there is a problem at hand. Use a work-related example, if you can.

"Last month our sales team was facing a major challenge when we lost one of our primary distributors. I took action and started cold-calling, other potential distributors. I brainstormed with my team in some other ways that we could avoid a negative impact on our bottom line. We were quite successful in our recovery, and I would say that the biggest lesson I learned from the experience is that you are often only successful if you have motivated people in your corner."

"The most valuable lesson I learned from problem-solving at work is that not everyone will see your solution as the best one. Accepting change is difficult for some people, so I have found that not everyone will be on board right away."

"I recently had an employee express their disinterest in the job and the company. Rather than coach them out, I selfishly wanted to 'save' the employee. I put in extra hours mentoring and training her, just to see her quit anyways. It's a valuable lesson as a manager to put your energy into those who want to be there. Other efforts are often just a temporary fix for the inevitable."

"Marketing is always shifting so I often learn new, valuable lessons. One lesson I recently learned was to double check the documents that I send out for any needed updates. A lot of the manuals and how-tos that we send clients are evergreen; however, some are not. I accidentally sent an old social media guide to a client, and they ended up being incredibly confused. My lack of attention to detail at that moment was a bit embarrassing but lesson learned!"

"A recent learning experience was when I misjudged what the customer was upset about, and I didn't take the time to learn what it was that she was looking for. It reminded me to slow down, go back to the basics, and not assume that all situations fit the mold of the 'typical' customer. It was a perfect reminder that though I've seen most everything, I need to remember that each person and situation is unique."

"A recent valuable lesson for me has been not putting all of my eggs in one basket, as the old saying goes. Over 64% of my sales came from one group of stores, and they've always been a big contributor to the entire company's sales numbers. However, they were put on "hold" recently by their corporate, due to some restructuring issues. This event threw me for a loop. I was in real danger of not hitting my monthly sales target, and therefore I would have fallen short on my quarterly quota as well. I had to work extra long hours and hustle my other clients and fence-sitters to get them into "buy" mode to make up for the void in my numbers. It took a ton of effort, long nights, and creative pitches, but I was able to make up for the gap. I learned just how important it is to diversify my portfolio so that I don't find myself, or the company, in this position again."

"When working on curriculum development, I learned an important lesson. Two of our teachers wanted to keep a lesson in, because of personal connections to the lesson, but the other three were quite against it, with me being the uncertain one. I saw the validity in both sides. So, rather than find ourselves with a divisive issue on our hands, I proposed that we have a "freebie" lesson when we each got to pick one that we thought would culturally enrich our students. I learned that by thinking outside of the box, the team and our students would all benefit."

22. When change occurs in the workplace, it can create new problems. Do you see these as inconvenient problems, or opportunities to learn?

When a change occurs in the workplace, often problems arise due to new implementations and procedures, or unforeseen kinks needing to be worked out. Do you approach these problems positively or do you resist the change? Talk to the interviewer about how you can adapt to the inevitable issues that come with the change in the workplace.

"I fully understand that when the change occurs in the workplace, some new problems may arise because of it. I embrace workplace change because it often gives me the opportunity to learn a new skill or even teach a colleague a new skill."

"As an executive assistant, I see change all the time. Policy changes, travel changes, issues in scheduling, and the like. Although they are often inconvenient or threaten to throw my day off, I am always prepared with a Plan B. Each time these situations occur, I learn something new."

"Change is inevitable when you work with people because you cannot control everyone's reactions in a day, or whether they even show up to work. Recently I had a major shift in my team and, overnight, went from being completely confident in my team to the need of reassessing our strategy. I saw this as an opportunity to stretch outside of my comfort zone. I embrace change and learning opportunities."

"One change that we always go through in this industry are shifts related to social media platforms and online trends. These tools are ever evolving, and when you think you have it - poof - changes are made. I don't mind this, however. I believe that each shift is a chance to learn something new."

"I like to approach every day and situation as an opportunity to learn and grow, so even though it's uncomfortable, I like to think that there's something valuable to take away from any situation that involves change."

"I'm all about taking everything in stride and jumping on opportunities for growth and improvement. My latest job has been a year-long exercise in that: a start-up that pivoted entirely from the direction it had been going in when I was brought on, with an entirely new team and even intended client base. I decided to take it as a growth opportunity. I took a deep breath, rolled up my sleeves, and got to work learning and adapting to the new product, clients, and management. I think that the experience will serve me well in the future since I became quite flexible and learned a lot about myself and sales in the process."

"I am adaptable to change. As a teacher, I have to be open to change! Nothing stays the same in education and students challenge everything. I am capable of pivoting when needed and am not thrown off my game, easily."

23. Rate your problem solving skills from 1-10. How do you justify your rating?

The interviewer wants to know how you would rate your problem-solving skills. Of course, you want to give yourself a healthy rating; however, it's crucial that you remain realistic. Try to avoid giving yourself a 10, and nobody is perfect, and you do not want to come across as overly confident or someone who has no room for feedback and improvement. Alternately, avoid giving yourself too little credit. You do not want to paint the picture that you are a problem-solving dud! Try to remain in the 7.5-9.5 range while staying honest and accurate. Everyone has room to learn and improve! Be sure to justify your score as well.

"I rate my problem-solving skills as an 8/10. I will, on occasion, have times when I am not as efficient as I would like to be but all in all, I do feel that my problem-solving skills are above average. My supervisor and co-workers will attest to my fast reflexes when a problem arises, and they would also say that I remain calm under pressure."

"I will rate myself an eight because I value problem-solving but, just like most people, I have things to learn. Some ways to ensure that I can effectively solve issues are by utilizing multiple knowledge resources when looking for answers."

"I will rate myself an 8.5 because I consider myself a strong problem solver, especially when it comes to important matters that affect my team. Solid problem-solving skills are the foundation of success in business. I am always striving to be a better problem solver, so I leave the rest of the scale as an aspirational measure."

"Problem-solving is at the heart of what we do in marketing. We have to solve branding and sales issues for our clients all the time. I am an exceptional problem-solver, and quite creative with my strategies. For that reason, I will rate myself as a 9/10 and always improving."

"I'd rate my problem-solving skills as an 8/10. I believe I'm always a willing learner who brings creativity to the table, no matter what the situation. I am still full of ideas on how to solve a problem, and yet I am also open to the opinion and input of others. I like to collaborate but am not afraid to take charge and make it happen. There's room for growth, which is why I give myself only an 8!"

"I would say I get a solid 8.3 on a scale of 10. Seems weird to give myself something like a .3, but I think of it as an 83%, which is a B minus teetering on a solid B. It's a solid grade, with definite room for improvement, since I'm certainly not perfect. The reason for the B-/B grade would be that I'm quick to take action and figure out the solution as I go, but sometimes I could benefit from taking a moment to pause and reflect or gather other contributors before taking action. That said, I believe I get the best outcome possible when faced with a challenge."

"I would say I'm a strong problem solver and would rate myself an 8/10. I follow my gut and problem solve creatively, but know there is still room for improvement. I think my teamwork and problem strategies highlight my strengths in problem-solving. I can hear what people find essential and flush out the things on which we can compromise. Then, I come up with a great outcome that makes the teachers happy and is in the best interest of our students."

24. What do you think might be the greatest challenges faced in this job? How will you overcome these challenges?

Even though it may seem like a dream job, the interviewer wants to know that you have realistic expectations of the role and that you will not be blindsided if problems or challenges present themselves. Keep your answer simple. It is okay to ask for clarification on the position if you do not fully understand what challenges are in store for you.

"I think the greatest challenges in this role will be to learn the proper operation of the equipment. Another challenge will be the physical aspect of the position as I will be required to stand and walk around most of the day. I will be sure to pay keen attention to training and ask questions along the way. In regards to the physical component - I will get used to the additional activity after just a couple of days, I'm sure."

"I believe that the greatest challenge in this job will be to learn the ins and outs of your systems. I am familiar with SAP; however, will need to navigate some modules that will be new to me. If you don't mind, I would like to gain a head start on these by studying online for the next weekend or so."

"As a new manager, the biggest challenge is always to earn the trust of my new team. I plan to do this by getting to know everyone through genuine interest and conversation. I do understand that solid trust develops over time, but it's important to me to get started on the right foot."

"The greatest challenge is going to be getting to know your clients and their preferences. Every client has their quirks that need to be kept the top of mind during projects. I plan to read as many project notes as possible before diving into face-to-face meetings. I intend to come across to your clients as well-prepared and earnest."

"I think the added responsibility of running one of the highest volume departments in the store will be an adjustment, but it's a welcome challenge. I am looking forward to tackling it head on and growing through the challenges, because I know on the other side of those challenges, of that responsibility, lies the biggest opportunity yet."

"I would say the greatest challenge I'll face in this role is learning the industry ins and outs to be perceived as an expert when making the pitch to new clients. I want to be sure to immerse myself in the industry jargon, attend as many seminars and conventions as possible, and I've already begun subscribing and reading the leading industry publications so that I can get into the nitty-gritty of how it all works. Of course, I will also seek out mentorship opportunities where I can learn from folks who have been in the industry for years. I find they love to share their knowledge and it gives me a leg up."

"I believe the greatest challenge faced in this new position would be getting accustomed to the new curriculum. I am accustomed to my lesson plans and the curriculum I've had a hand in developing over the last ten years, so something new will have a bit of a learning curve, but welcomed. I am looking forward to a new challenge and to tackle a new set of lessons!"

25. What steps do you take to solve a problem?

The interviewer would like to know that you understand the importance of taking calculated steps when problem-solving in the workplace. Most candidates want to sound like go-getters, and their first instinct would be to say that they jump right in. Jumping right in can cause costly mistakes and oversights. Assure the interviewer that you will workshop the issue before diving in! Here are some steps to take: 1. Identify The Problem. Proper problem solving involves ensuring that you are very clear on the nature of the problem. Be sure that you fully understand the core of the problem before trying to repair it. 2. Identify The Stakeholders. Ask yourself, what the best case resolution will be for all stakeholders, not just for yourself. Ask yourself what is best for the company, your coworkers, and your clients. 3. List Your Options. The third step is to figure out what your options are when it comes to your course of action. Write them down if you need to. 4. Evaluate Your Options. Take a look at your list of potential actions and see if you can solve the problem using just one, or a blend of them. 5. Execute! Finally, execute your well-researched action plan. Be sure to set up a follow-up time to ensure that your solution worked.

"When I need to solve a problem, I first stop to ensure that I understand the issue at hand. Once I do, I will think of potential fixes and the pros and cons of each. Whichever solution or a blend of solutions is best for the customer; I will choose that option."

"My current company is very team-focused, and we train everyone to problem-solve with "what is best for team morale" being the question at hand. I have been with the company for twelve years so most problems I have a pretty clear idea of what will work for us, but when I need to workshop an idea, I will call in my team and have a brainstorm session."

"Problem-solving in Marketing can be unique because you have to truly balance the customers' pain point with the solutions that are currently available. Also, some clients like trying new marketing methods and others want to remain conservative, using only tried and true advertising methods, for instance. When I approach a problem, I first identify the personality of the client and their business and research options from there."

"Problem-solving in a retail environment is challenging in the sense that the issue is often something that needs to be fixed immediately, like a faulty product or an upset customer. When faced with a problem, I ask questions first, to ensure that I fully understand the core of the issue. Once I fully understand the core of the problem, I can more easily troubleshoot from there."

"Every customer is different, with unique needs, so when I need to problem-solve, I am often coming across a brand new problem or a different version of a problem I have seen before. Our company is big on chasing the money, and so I have been trained that every solution I choose must have the business' bottom line top of mind. My process is to understand the issue, address who the stakeholders are, and create a solution where everyone feels they won in some small way."

"Problem-solving in the classroom is a challenge because it is often on the fly. Or, a student will ask a question in a new way and I won't necessarily have the answer! When a problem arises, I like to involve my class, have a brainstorm session, and discuss as a group what we could do. This method turns an issue into a conversation where we have the opportunity to come up with some unique solutions."

Need to start saving with a new ATS? Learn how to calculate the return on investment of your ATS Calculate ROI now

  • HR Toolkit |
  • HR Templates |
  • Interview questions |

Problem-solving interview questions and answers

Use these sample problem-solving interview questions to discover how candidates approach complex situations and if they can provide effective solutions.

Christina Pavlou

An experienced recruiter and HR professional who has transferred her expertise to insightful content to support others in HR.

problem-solving interview questions

10 good problem-solving interview questions

  • Describe a time when you had to solve a problem without managerial input. How did you handle it, and what was the result?
  • Give an example of a time when you identified and fixed a problem before it became urgent.
  • Share a situation where you predicted a problem with a stakeholder. How did you prevent it from escalating?
  • Describe a time when you faced challenges in doing your job efficiently. How did you overcome them?
  • Recall a time when you successfully used crisis-management skills.
  • How would you handle a new project with great revenue potential but potential legal implications for the company?
  • How do you determine when to solve a problem on your own or ask for help?
  • Describe a time when you faced a difficult situation at work that required critical thinking and decision-making under pressure.
  • Have you ever used intuition or prior experience to anticipate and address a problem effectively? Provide an example.
  • Share an example of a project or task that initially seemed overwhelming. How did you approach it, and what strategies did you use to ensure successful completion?

1. Describe a time when you had to solve a problem without managerial input. How did you handle it, and what was the result?

This question assesses candidates’ ability to take initiative and solve problems independently. Look for their problem-solving approach, the actions they took, and the outcome of their efforts.

Sample answer:

“In my previous role, we encountered a sudden technical issue that disrupted our operations. As the team lead, I gathered all available information, analyzed the root cause, and facilitated a brainstorming session with the team. We implemented a temporary workaround and collaborated with the IT department to resolve the issue. Our proactive approach ensured minimal disruption, and we were able to restore normal operations within 24 hours.”

2. Give an example of a time when you identified and fixed a problem before it became urgent.

This question evaluates candidates’ ability to anticipate and address problems proactively. Look for their ability to identify potential issues and take preventive measures.

“While working as a project manager, I noticed a potential bottleneck in our production process that could have led to delays if left unaddressed. I conducted a thorough analysis, identified the root cause, and proposed process improvements. By implementing these changes proactively, we eliminated the bottleneck and increased efficiency. As a result, we consistently met project deadlines, and our team’s productivity significantly improved.”

3. Share a situation where you predicted a problem with a stakeholder. How did you prevent it from escalating?

This question assesses candidates’ ability to identify and mitigate potential conflicts. Look for their communication and problem-solving skills in managing stakeholder relationships.

“While working on a cross-functional project, I anticipated a miscommunication issue that could arise with a key stakeholder due to conflicting expectations. I scheduled a meeting with the stakeholder, listened to their concerns, and facilitated a discussion among the team members. By proactively addressing the issue, we established clear communication channels, built trust, and ensured a smooth collaboration throughout the project.”

4. Describe a time when you faced challenges in doing your job efficiently. How did you overcome them?

This question evaluates candidates’ ability to handle challenges and find solutions to improve efficiency. Look for their problem-solving strategies and their adaptability to overcome obstacles.

“In a previous role, I faced a situation where the workload significantly increased due to unexpected circumstances. To maintain efficiency, I assessed the situation, identified tasks that could be delegated, and communicated with my colleagues to seek their support. By redistributing responsibilities and fostering a collaborative environment, we successfully managed the increased workload without compromising quality or missing deadlines.”

5. Recall a time when you successfully used crisis-management skills.

This question assesses candidates’ ability to remain calm and make effective decisions under pressure. Look for their problem-solving approach and their ability to handle high-stress situations.

“In a previous role as a customer service representative, we experienced a sudden surge in customer complaints due to a product quality issue. I quickly coordinated with relevant departments, identified the root cause, and developed an action plan. By prioritizing urgent cases, maintaining open communication with affected customers, and providing timely updates, we regained customer satisfaction and prevented further damage to our brand reputation.”

6. How would you handle a new project with great revenue potential but potential legal implications for the company?

This question assesses candidates’ ability to balance potential risks and rewards. Look for their ethical considerations, problem-solving approach, and willingness to seek guidance when faced with legal implications.

“If faced with a project that carries both revenue potential and potential legal implications, I would approach it with caution and thorough evaluation. I would research and seek legal guidance to fully understand the implications and compliance requirements. I would then collaborate with legal experts, cross-functional teams, and stakeholders to develop a comprehensive plan that minimizes legal risks while maximizing revenue potential.”

7. How do you determine when to solve a problem on your own or ask for help?

This question assesses candidates’ judgment and collaboration skills. Look for their ability to assess situations and make decisions about when to seek assistance.

“When faced with a problem, I first evaluate its complexity and impact on the project or task at hand. If it’s within my capabilities and doesn’t significantly hinder progress, I take the initiative to solve it on my own. However, if the problem is complex or could have a significant impact, I believe in seeking help from relevant team members or subject matter experts. Collaboration often leads to more comprehensive and effective solutions.”

8. Describe a time when you faced a difficult situation at work that required critical thinking and decision-making under pressure.

This question assesses candidates’ ability to think critically and make sound decisions in challenging situations. Look for their problem-solving approach, decision-making process, and the outcomes of their decisions.

“In a previous role, I faced a tight deadline for a project with limited resources. It required careful resource allocation and prioritization. I gathered all available data, analyzed the project requirements, and consulted with team members. Through strategic planning and effective delegation, we managed to complete the project successfully within the given timeframe, exceeding client expectations.”

9. Have you ever used intuition or prior experience to anticipate and address a problem effectively? Provide an example.

This question assesses candidates’ ability to leverage intuition and past experiences to navigate problem-solving situations. Look for their ability to reflect on past situations, apply lessons learned, and make informed decisions.

“In a previous role, I noticed a recurring issue in our supply chain that had caused delays in the past. Drawing upon my prior experience, I anticipated the problem and suggested process improvements to streamline the supply chain. By implementing these changes, we minimized delays and improved overall efficiency, resulting in cost savings for the company.”

10. Share an example of a project or task that initially seemed overwhelming. How did you approach it, and what strategies did you use to ensure successful completion?

This question assesses candidates’ ability to tackle complex projects and break them down into manageable tasks. Look for their problem-solving approach, organization skills, and ability to persevere in the face of challenges.

“I once undertook a project that involved a significant amount of data analysis and reporting within a tight deadline. Initially, it felt overwhelming, but I broke it down into smaller tasks and created a detailed timeline. I prioritized the most critical aspects and sought assistance from colleagues with specialized skills. Through effective time management, collaboration, and diligent effort, we successfully completed the project on time and delivered high-quality results.”

Why you should ask candidates problem-solving interview questions

Employees will face challenges in their job. Before you decide on your next hire, use your interview process to evaluate how candidates approach difficult situations.

Problem-solving interview questions show how candidates:

  • Approach complex issues
  • Analyze data to understand the root of the problem
  • Perform under stressful and unexpected situations
  • React when their beliefs are challenged

Identify candidates who are results-oriented with interview questions that assess problem-solving skills. Look for analytical and spherical thinkers with the potential for technical problem solving.

Potential hires who recognize a problem, or predict one could potentially occur, will stand out. Candidates should also demonstrate how they would fix the issue, and prevent it from occurring again.

These sample problem-solving interview questions apply to all positions, regardless of industry or seniority level. You can use the following questions to gauge your candidates’ way of thinking in difficult situations:

Tips to assess problem-solving skills in interviews

  • During your interviews, use hypothetical scenarios that are likely to occur on the job. It’s best to avoid unrealistic problems that aren’t relevant to your company.
  • Examine how candidates approach a problem step-by-step: from identifying and analyzing the issue to comparing alternatives and choosing the most effective solution.
  • Pay attention to candidates who provide innovative solutions. Creative minds can contribute fresh perspectives that add value to your company.
  • When problems arise, employees should show commitment and a can-do attitude. Test candidates’ problem-solving skills in past situations. If they were determined to find the best solution as soon as possible, they will be great hires.
  • Most complex situations require a team effort. Candidates’ previous experiences will show you how they collaborated with their colleagues to reach decisions and how comfortable they felt asking for help.
  • If you’re hiring for a technical role, ask questions relevant to the work your future hires will do. Technical problem-solving interview questions, like “How would you troubleshoot this X bug?” will reveal your candidates’ hard skills and their ability to effectively address problems on the job.
  • No answer. If a candidate can’t recall an example of a problem they faced in a previous position, that’s a sign they may avoid dealing with difficult situations. Canned answers. A generic answer like “Once, I had to deal with a customer who complained about the pricing. I managed to calm them down and closed the deal,” doesn’t offer much insight about the candidate’s thought process. Ask follow-up questions to get more details.
  • Focus on the problem, not the solution. Identifying the problem is one thing, but finding the solution is more important. Candidates who focus too much on the problem may be too negative for the position.
  • Feeling stressed/uncomfortable. It’s normal to feel slightly uncomfortable when put on the spot. But, if candidates are so stressed they can’t answer the question, that’s an indicator they don’t handle stressful situations well.
  • Superficial answers. Candidates who choose the easy way out of a problem usually don’t consider all aspects and limitations of the situation. Opt for candidates who analyze the data you’ve given them and ask for more information to better dig into the problem.
  • Cover up the problem or minimize its significance. Unaddressed problems could quickly escalate into bigger issues. Employees who leave things for later mightn’t be result-oriented or engaged in their jobs.

In conclusion, problem-solving interview questions provide valuable insights into candidates’ abilities to approach challenges, think critically, and provide effective solutions. By asking these questions and considering the tips provided, you can assess candidates’ problem-solving skills and make informed hiring decisions that align with your organization’s needs.

Frequently asked questions

Want a custom interview kit, related interview questions.

  • Critical-thinking interview questions and answers
  • Decision-making interview questions and answers
  • Analytical interview questions and answers

Related Topics

  • How to assess soft skills in an interview
  • Interview process and strategies: a comprehensive FAQ guide
  • Structured interview questions: Tips and examples for hiring

Jump to section

Let's grow together.

Explore our full platform with a 15-day free trial. Post jobs, get candidates and onboard employees all in one place.

Share on Mastodon

Forage

What Are Problem-Solving Skills? Definition and Examples

Zoe Kaplan

  • Share on Twitter Share on Twitter
  • Share on Facebook Share on Facebook
  • Share on LinkedIn Share on LinkedIn

person sitting at desk with headphones thinking

Forage puts students first. Our blog articles are written independently by our editorial team. They have not been paid for or sponsored by our partners. See our full  editorial guidelines .

Why do employers hire employees? To help them solve problems. Whether you’re a financial analyst deciding where to invest your firm’s money, or a marketer trying to figure out which channel to direct your efforts, companies hire people to help them find solutions. Problem-solving is an essential and marketable soft skill in the workplace. 

So, how can you improve your problem-solving and show employers you have this valuable skill? In this guide, we’ll cover:

Problem-Solving Skills Definition

Why are problem-solving skills important, problem-solving skills examples, how to include problem-solving skills in a job application, how to improve problem-solving skills, problem-solving: the bottom line.

Problem-solving skills are the ability to identify problems, brainstorm and analyze answers, and implement the best solutions. An employee with good problem-solving skills is both a self-starter and a collaborative teammate; they are proactive in understanding the root of a problem and work with others to consider a wide range of solutions before deciding how to move forward. 

Examples of using problem-solving skills in the workplace include:

  • Researching patterns to understand why revenue decreased last quarter
  • Experimenting with a new marketing channel to increase website sign-ups
  • Brainstorming content types to share with potential customers
  • Testing calls to action to see which ones drive the most product sales
  • Implementing a new workflow to automate a team process and increase productivity

Problem-solving skills are the most sought-after soft skill of 2022. In fact, 86% of employers look for problem-solving skills on student resumes, according to the National Association of Colleges and Employers Job Outlook 2022 survey . 

It’s unsurprising why employers are looking for this skill: companies will always need people to help them find solutions to their problems. Someone proactive and successful at problem-solving is valuable to any team.

“Employers are looking for employees who can make decisions independently, especially with the prevalence of remote/hybrid work and the need to communicate asynchronously,” Eric Mochnacz, senior HR consultant at Red Clover, says. “Employers want to see individuals who can make well-informed decisions that mitigate risk, and they can do so without suffering from analysis paralysis.”

Showcase new skills

Build the confidence and practical skills that employers are looking for with Forage’s free job simulations.

Problem-solving includes three main parts: identifying the problem, analyzing possible solutions, and deciding on the best course of action.

>>MORE: Discover the right career for you based on your skills with a career aptitude test .

Research is the first step of problem-solving because it helps you understand the context of a problem. Researching a problem enables you to learn why the problem is happening. For example, is revenue down because of a new sales tactic? Or because of seasonality? Is there a problem with who the sales team is reaching out to? 

Research broadens your scope to all possible reasons why the problem could be happening. Then once you figure it out, it helps you narrow your scope to start solving it. 

Analysis is the next step of problem-solving. Now that you’ve identified the problem, analytical skills help you look at what potential solutions there might be.

“The goal of analysis isn’t to solve a problem, actually — it’s to better understand it because that’s where the real solution will be found,” Gretchen Skalka, owner of Career Insights Consulting, says. “Looking at a problem through the lens of impartiality is the only way to get a true understanding of it from all angles.”

Decision-Making

Once you’ve figured out where the problem is coming from and what solutions are, it’s time to decide on the best way to go forth. Decision-making skills help you determine what resources are available, what a feasible action plan entails, and what solution is likely to lead to success.

On a Resume

Employers looking for problem-solving skills might include the word “problem-solving” or other synonyms like “ critical thinking ” or “analytical skills” in the job description.

“I would add ‘buzzwords’ you can find from the job descriptions or LinkedIn endorsements section to filter into your resume to comply with the ATS,” Matthew Warzel, CPRW resume writer, advises. Warzel recommends including these skills on your resume but warns to “leave the soft skills as adjectives in the summary section. That is the only place soft skills should be mentioned.”

On the other hand, you can list hard skills separately in a skills section on your resume .

answer in problem solving

Forage Resume Writing Masterclass

Learn how to showcase your skills and craft an award-winning resume with this free masterclass from Forage.

Avg. Time: 5 to 6 hours

Skills you’ll build: Resume writing, professional brand, professional summary, narrative, transferable skills, industry keywords, illustrating your impact, standing out

In a Cover Letter or an Interview

Explaining your problem-solving skills in an interview can seem daunting. You’re required to expand on your process — how you identified a problem, analyzed potential solutions, and made a choice. As long as you can explain your approach, it’s okay if that solution didn’t come from a professional work experience.

“Young professionals shortchange themselves by thinking only paid-for solutions matter to employers,” Skalka says. “People at the genesis of their careers don’t have a wealth of professional experience to pull from, but they do have relevant experience to share.”

Aaron Case, career counselor and CPRW at Resume Genius, agrees and encourages early professionals to share this skill. “If you don’t have any relevant work experience yet, you can still highlight your problem-solving skills in your cover letter,” he says. “Just showcase examples of problems you solved while completing your degree, working at internships, or volunteering. You can even pull examples from completely unrelated part-time jobs, as long as you make it clear how your problem-solving ability transfers to your new line of work.”

Learn How to Identify Problems

Problem-solving doesn’t just require finding solutions to problems that are already there. It’s also about being proactive when something isn’t working as you hoped it would. Practice questioning and getting curious about processes and activities in your everyday life. What could you improve? What would you do if you had more resources for this process? If you had fewer? Challenge yourself to challenge the world around you.

Think Digitally

“Employers in the modern workplace value digital problem-solving skills, like being able to find a technology solution to a traditional issue,” Case says. “For example, when I first started working as a marketing writer, my department didn’t have the budget to hire a professional voice actor for marketing video voiceovers. But I found a perfect solution to the problem with an AI voiceover service that cost a fraction of the price of an actor.”

Being comfortable with new technology — even ones you haven’t used before — is a valuable skill in an increasingly hybrid and remote world. Don’t be afraid to research new and innovative technologies to help automate processes or find a more efficient technological solution.

Collaborate

Problem-solving isn’t done in a silo, and it shouldn’t be. Use your collaboration skills to gather multiple perspectives, help eliminate bias, and listen to alternative solutions. Ask others where they think the problem is coming from and what solutions would help them with your workflow. From there, try to compromise on a solution that can benefit everyone.

If we’ve learned anything from the past few years, it’s that the world of work is constantly changing — which means it’s crucial to know how to adapt . Be comfortable narrowing down a solution, then changing your direction when a colleague provides a new piece of information. Challenge yourself to get out of your comfort zone, whether with your personal routine or trying a new system at work.

Put Yourself in the Middle of Tough Moments

Just like adapting requires you to challenge your routine and tradition, good problem-solving requires you to put yourself in challenging situations — especially ones where you don’t have relevant experience or expertise to find a solution. Because you won’t know how to tackle the problem, you’ll learn new problem-solving skills and how to navigate new challenges. Ask your manager or a peer if you can help them work on a complicated problem, and be proactive about asking them questions along the way.

Career Aptitude Test

What careers are right for you based on your skills? Take this quiz to find out. It’s completely free — you’ll just need to sign up to get your results!

Step 1 of 3

Companies always need people to help them find solutions — especially proactive employees who have practical analytical skills and can collaborate to decide the best way to move forward. Whether or not you have experience solving problems in a professional workplace, illustrate your problem-solving skills by describing your research, analysis, and decision-making process — and make it clear that you’re the solution to the employer’s current problems. 

Looking to learn more workplace professional skills? Check out Two Sigma’s Professional Skills Development Virtual Experience Program .

Image Credit: Christina Morillo / Pexels 

Zoe Kaplan

Related Posts

6 negotiation skills to level up your work life, how to build conflict resolution skills: case studies and examples, what is github uses and getting started, upskill with forage.

answer in problem solving

Build career skills recruiters are looking for.

More From Forbes

Computational thinking is a key problem-solving skill in the ai era.

Forbes Human Resources Council

  • Share to Facebook
  • Share to Twitter
  • Share to Linkedin

Neil Morelli, Ph.D., Chief I-O Psychologist, Codility .

A May 2024 Reuters Institute and University of Oxford survey, which included more than 12,000 people from six countries, found that 21% of respondents on average have used ChatGPT professionally. This will surely grow as companies ramp up their AI adoption efforts and look for job candidates with generative AI skills. In fact, Microsoft's 2024 Work Trend Index Annual Report estimates that 71% of leaders are now more likely to hire a candidate with AI skills, even if they have less overall experience.

As AI becomes more prevalent in the workplace, there's one vital skill that can help determine a worker's success in learning, deploying and working with AI tools: computational thinking.

What Is Computational Thinking?

You might not have heard of computational thinking before, but computer scientists and educators have discussed similar concepts since the 1960s. But it was Jeannette Wing, an MIT-trained computer scientist and Columbia professor, who popularized computational thinking in 2006 . She argued that it's a "universally applicable attitude and skill set" that can benefit anyone.

Similar to general problem-solving, computational thinking involves breaking down problems, pulling out key information and forming solutions. But it goes a step further and turns solutions into repeatable instructions that a computer can understand and automate. Computational thinking has five parts: breaking problems into more solvable pieces, abstracting how a problem works into underlying rules, designing a series of steps that take inputs and produce an output (an algorithm), testing a solution over multiple cycles and generalizing the solution to similar problems.

There Is Only One Actor Who Can Take Over As MCU Wolverine From Hugh Jackman

Today’s nyt mini crossword clues and answers for tuesday, july 23, trump: ‘crazy’ that secret service didn’t have more manpower at rally.

Wing emphasized that computational thinking is a fundamental skill for problem-solvers, not just a rote mechanical skill for computer literacy or programming. It helps solve problems by finding patterns, understanding new systems and improving solutions with feedback.

To borrow a simple illustration , what do you do when your mobile phone stops working? If you're thinking computationally, you'll form a mental model of how the phone works and hypothesize that a manual reset may refresh the device's internal state to get it working again. Similarly, computational thinking helps AI users create mental models of the systems so they can work more effectively with these tools.

3 Ways Computational Thinking Improves Effectiveness With AI

Let's explore three fundamental aspects of computational thinking and their relevance to AI.

1. Breaking Problems Into Solvable Parts

Computational thinking allows you to divide up problems into more manageable, solvable components. Once broken down, you'll begin abstracting the parts into a working model. When their components are isolated and understood, big problems become less menacing and easier to solve.

When applied to AI, this skill translates into using effective prompting techniques to guide the model toward better solutions. This might involve breaking down complex queries into smaller parts (chain of thought) or providing examples to illustrate the desired outcome ( few-shot learning ).

2. Understanding A System

Computational thinking helps problem-solvers understand systems and get a sense of which AI tool works best for the task. With this thought process, they can better identify the appropriate AI model to employ—whether public, private, fine-tuned or foundational—or if an AI tool is necessary at all. For instance, programming thought leaders like Dave Farley argue that developers save time with generative AI coding tools when they understand whether it's easier to write the code manually or prompt the AI and modify the output.

3. Iterating And Improving Outputs

Recognizing patterns and thinking in loops to improve solutions continuously is a key component of computational thinking. The best problem-solvers use trial and error to improve solutions each time. Computational thinkers can build on AI-generated outputs with additional instructions and refinements using their knowledge and creativity over multiple cycles to increase quality and usability.

Computational Thinking: Beyond Programming

Computational thinking is a foundational skill that underpins effective software engineering. But it extends beyond simply knowing a programming language. It involves thinking like a computer scientist to solve problems.

Wing predicted that as technical systems become more integrated into our daily lives, this way of thinking would become essential for everyone. Today, AI assistants are making this prediction a reality. These tools provide access to advanced computing power through natural language interactions rather than requiring users to communicate in machine language. This shift acts as a force multiplier for computational thinking skills, and more people can supercharge their problem-solving capabilities.

As AI reshapes the modern economy, organizations must identify and nurture problem-solving skills using computing systems, in addition to assessing and developing pure technical skills within a limited set of roles. The most successful workforces will be those that blend human creativity and judgment with AI capabilities. By developing and applying computational thinking skills, professionals across all fields can enhance their effectiveness and adapt to the changing landscape.

Forbes Human Resources Council is an invitation-only organization for HR executives across all industries. Do I qualify?

Neil Morelli

  • Editorial Standards
  • Reprints & Permissions

For IEEE Members

Ieee spectrum, follow ieee spectrum, support ieee spectrum, enjoy more free content and benefits by creating an account, saving articles to read later requires an ieee spectrum account, the institute content is only available for members, downloading full pdf issues is exclusive for ieee members, downloading this e-book is exclusive for ieee members, access to spectrum 's digital edition is exclusive for ieee members, following topics is a feature exclusive for ieee members, adding your response to an article requires an ieee spectrum account, create an account to access more content and features on ieee spectrum , including the ability to save articles to read later, download spectrum collections, and participate in conversations with readers and editors. for more exclusive content and features, consider joining ieee ., join the world’s largest professional organization devoted to engineering and applied sciences and get access to all of spectrum’s articles, archives, pdf downloads, and other benefits. learn more about ieee →, join the world’s largest professional organization devoted to engineering and applied sciences and get access to this e-book plus all of ieee spectrum’s articles, archives, pdf downloads, and other benefits. learn more about ieee →, access thousands of articles — completely free, create an account and get exclusive content and features: save articles, download collections, and talk to tech insiders — all free for full access and benefits, join ieee as a paying member., how good is chatgpt at coding, really, study finds that while ai can be great, it also struggles due to training limitations.

Illustration of ghostly hands with 0s an 1s hovering over a keyboard

This article is part of our exclusive IEEE Journal Watch series in partnership with IEEE Xplore.

Programmers have spent decades writing code for AI models , and now, in a full circle moment, AI is being used to write code. But how does an AI code generator compare to a human programmer?

A study published in the June issue of IEEE Transactions on Software Engineering evaluated the code produced by OpenAI’s ChatGPT in terms of functionality, complexity and security. The results show that ChatGPT has an extremely broad range of success when it comes to producing functional code—with a success rate ranging from anywhere as poor as 0.66 percent and as good as 89 percent—depending on the difficulty of the task, the programming language, and a number of other factors.

While in some cases the AI generator could produce better code than humans, the analysis also reveals some security concerns with AI-generated code.

Yutian Tang is a lecturer at the University of Glasgow who was involved in the study. He notes that AI-based code generation could provide some advantages in terms of enhancing productivity and automating software development tasks—but it’s important to understand the strengths and limitations of these models.

“By conducting a comprehensive analysis, we can uncover potential issues and limitations that arise in the ChatGPT-based code generation... [and] improve generation techniques,” Tang explains.

To explore these limitations in more detail, his team sought to test GPT-3.5’s ability to address 728 coding problems from the LeetCode testing platform in five programming languages: C, C++, Java, JavaScript, and Python .

“A reasonable hypothesis for why ChatGPT can do better with algorithm problems before 2021 is that these problems are frequently seen in the training dataset.” —Yutian Tang, University of Glasgow

Overall, ChatGPT was fairly good at solving problems in the different coding languages—but especially when attempting to solve coding problems that existed on LeetCode before 2021. For instance, it was able to produce functional code for easy, medium, and hard problems with success rates of about 89, 71, and 40 percent, respectively.

“However, when it comes to the algorithm problems after 2021, ChatGPT’s ability to generate functionally correct code is affected. It sometimes fails to understand the meaning of questions, even for easy level problems,” Tang notes.

For example, ChatGPT’s ability to produce functional code for “easy” coding problems dropped from 89 percent to 52 percent after 2021. And its ability to generate functional code for “hard” problems dropped from 40 percent to 0.66 percent after this time as well.

“A reasonable hypothesis for why ChatGPT can do better with algorithm problems before 2021 is that these problems are frequently seen in the training dataset,” Tang says.

Essentially, as coding evolves, ChatGPT has not been exposed yet to new problems and solutions. It lacks the critical thinking skills of a human and can only address problems it has previously encountered. This could explain why it is so much better at addressing older coding problems than newer ones.

“ChatGPT may generate incorrect code because it does not understand the meaning of algorithm problems.” —Yutian Tang, University of Glasgow

Interestingly, ChatGPT is able to generate code with smaller runtime and memory overheads than at least 50 percent of human solutions to the same LeetCode problems.

The researchers also explored the ability of ChatGPT to fix its own coding errors after receiving feedback from LeetCode. They randomly selected 50 coding scenarios where ChatGPT initially generated incorrect coding, either because it didn’t understand the content or problem at hand.

While ChatGPT was good at fixing compiling errors, it generally was not good at correcting its own mistakes.

“ChatGPT may generate incorrect code because it does not understand the meaning of algorithm problems, thus, this simple error feedback information is not enough,” Tang explains.

The researchers also found that ChatGPT-generated code did have a fair amount of vulnerabilities, such as a missing null test, but many of these were easily fixable. Their results also show that generated code in C was the most complex, followed by C++ and Python, which has a similar complexity to the human-written code.

Tangs says, based on these results, it’s important that developers using ChatGPT provide additional information to help ChatGPT better understand problems or avoid vulnerabilities.

“For example, when encountering more complex programming problems, developers can provide relevant knowledge as much as possible, and tell ChatGPT in the prompt which potential vulnerabilities to be aware of,” Tang says.

  • What to Do When the Ghost in the Machine Is You ›
  • How Coders Can Survive—and Thrive—in a ChatGPT World ›
  • Coding Assistant - ChatGPT ›

Michelle Hampson is a freelance writer based in Halifax. She frequently contributes to Spectrum's Journal Watch coverage, which highlights newsworthy studies published in IEEE journals.

Floch Forster

That's yesterday's news, try it with version 4o, it's free.

Richard Wickens

"struggles due to training limitations" isn't that EVERYONE's problem with EVERYTHING.

"I could be an awesome guitar playing, but I struggle due to training limitations."

"I could be a great Opera singer, but I struggle due to training limitations."

"I could be a great jockey, but I am 6'4"...." Ok, well maybe not everything.

ChatGPT sucks at coding because it's not an AI - it's a big ass word predictor.

Sam Sperling

I actually think the key here is writing good test suits to ensure AI does the right thing...

Here is the full argument: https://medium.com/@samuel.sperling/software-2-1-ai-is-coding-now-why-test-mastery-is-your-new-job-security-31a65e792f7f

Test Flight Demonstrates Navigation by Cellphone Signals

How olympics officials try to catch "motor doping", irobot's autowash dock is (almost) automated floor care, related stories, what to do when the ghost in the machine is you, chatgpt’s new upgrade teases ai’s multimodal future, chatgpt may be a better improviser than you.

What happened at Possum Trot? Remarkable story shows how we can solve America's problems.

In possum trot, texas, a single church changed the lives of dozens of children in foster care..

answer in problem solving

We’re looking for the wrong solutions to America’s biggest problems. It’s true of poverty, homelessness, mental health, addiction and so much more. Given the scale of these challenges, we tend to assume they need even bigger answers − massive programs and taxpayer support. But after decades of taking that approach, America’s problems have gotten worse.

What if the solutions are bigger  and  smaller than most people think?

What happened at Possum Trot

That’s the message of “ Sound of Hope: The Story of Possum Trot ,” a movie released this week. Named after a town in East Texas and based on a true story, it follows the story of dozens of local children who were taken from their families and put in foster care. Their lives were hard, and foster care made them harder. The kids were seemingly destined for a life of violence, jail, addiction, poverty and, in many cases, death.

But the town didn’t let that happen. A local pastor, Bishop W.C. Martin , and his wife, Donna Martin , adopted two of the kids, then called on their friends and fellow parishioners to step up.

All told, 22 families adopted 77 of the hardest-to-place children in the local foster system.

That was nearly three decades ago. While many of the former foster children are still dealing with the trauma of being taken from their families, the majority of them have grown to become thriving adults.

Children languish for years in foster care

It wouldn’t have happened had they stayed in the system. Each year, more than 600,000 kids spend time in foster care. Even one day in foster care is proven to be disastrous for a kid’s long-term outcomes. But 60% of children in foster care are in the system for more than a year.

Up to  4 out of 5 foster kids  have mental health issues, and 1 in 5 who age out of foster care become homeless before age 20.

I survived the foster care system. Dismantling it altogether is the only path forward.

These horrible outcomes happen despite decades of reform efforts and large-scale funding. Combined federal, state and local spending for child welfare services, which is largely about foster care, soared by about 120% between  the mid-1990s  and  2020 .

The federal government is paying for  more programs  on everything from mental health to substance abuse to skills training. Despite this massive investment, foster care still looks like an unsolvable problem.

But this problem is solvable. The town of Possum Trot showed how.

Strong communities hold key to solving big problems

The solution to foster care − and all of America’s biggest problems − is the power of relationships. Social scientists call it “social capital,” while others call it “strong communities." Whatever name you use, we’re talking about people helping people on an individual basis. And while that seems small, when you scale up relationships nationwide, it’s the biggest solution imaginable.

In Possum Trot, a single church changed the lives of dozens of families and children. But there are  nearly 400,000 churches  in America. If each one helped just one of the roughly 400,000 kids in foster care at any one time, the problem could be largely solved overnight.

And it’s not just about religious institutions. There are accessible ways for people to save kids from the lifelong struggles of foster care, all through the power of people helping people.

Americans are running away from church. But we don't have to run from each other.

I’ve seen the proof in communities across the nation. 

National Angels provides local support to foster families, making them 50% more likely to avoid burnout while providing more stability for the kids. Another group, Safe Families , helps kids avoid foster care altogether, pairing trusted families with moms and dads who are struggling to provide for their kids. They take care of the kids for a few weeks or months while the parents get back on their feet.

Remarkably, most of the families are reunited, compared with less than half in foster care . In both cases, the relationships made all the difference.

Opinion alerts: Get columns from your favorite columnists + expert analysis on top issues, delivered straight to your device through the USA TODAY app. Don't have the app? Download it for free from your app store .

Then there’s CarePortal . It connects local churches and community members with struggling families in real time, getting them the resources they need to provide for their kids. If a family can’t afford food, car seats or anything else their kids urgently need, neighbors swoop in so that child services doesn’t need to.

Surely there are many other groups I don’t know about that are making a big difference by applying the same insight.

Can something similar happen with other seemingly unsolvable problems? Absolutely. Homelessness, addiction, gang violence, poverty − they’re all made worse by the loss of community. The solution is to create community, which all of us can do. No amount of government spending or programming has done that, nor can it, for the simple reason that government action isn’t big enough. The bigger and better solution starts somewhere much smaller yet much more powerful: everyday Americans.

Evan Feinberg is senior vice president of Stand Together and chair of the Stand Together Foundation.

You can read diverse opinions from our USA TODAY columnists and other writers on the Opinion front page , on X, formerly Twitter, @usatodayopinion and in our Opinion newsletter .

  • Small Language Model
  • Computer Vision
  • Federated Learning
  • Reinforcement Learning
  • Natural Language Processing
  • New Releases
  • AI Webinars
  • 🔥 Promotion/Partnership

Logo

In conclusion, researchers have introduced integrating a reliable, deductive reasoning module into their inference pipeline. In this paper, the inherent limitations of LLMs are highlighted in performing reliable and general reasoning. The neurosymbolic approach prompts the LLM to convert the information encoded by problem statements into logical code statements, and this division of labor significantly improves the LLMs’ performance on mathematical reasoning tasks. Moreover, the proposed NLR dataset provides a strong benchmark for testing LLMs’ ability to handle problems that need unique nonlinear reasoning and challenge the usual linear next-word prediction approach of LLMs.

Check out the Paper . All credit for this research goes to the researchers of this project. Also, don’t forget to follow us on  Twitter and join our  Telegram Channel and  LinkedIn Gr oup . If you like our work, you will love our  newsletter..

Don’t Forget to join our  46k+ ML SubReddit

Find Upcoming AI Webinars here

answer in problem solving

Sajjad Ansari

Sajjad Ansari is a final year undergraduate from IIT Kharagpur. As a Tech enthusiast, he delves into the practical applications of AI with a focus on understanding the impact of AI technologies and their real-world implications. He aims to articulate complex AI concepts in a clear and accessible manner.

  • DiT-MoE: A New Version of the DiT Architecture for Image Generation
  • Make-An-Agent: A Novel Policy Parameter Generator that Leverages the Power of Conditional Diffusion Models for Behavior-to-Policy Generation
  • MACAROON: Enhancing the Proactive Conversation Abilities of Large Vision-Language Models LVLMs
  • RTMW: A Series of High-Performance AI Models for 2D/3D Whole-Body Pose Estimation

RELATED ARTICLES MORE FROM AUTHOR

Wtu-eval: a new standard benchmark tool for evaluating large language models llms usage capabilities, huggingface researchers introduce docmatix: a dataset for document visual question answering containing 2.4 million pictures and 9.5 million q/a pairs, agentpoison: a novel red teaming approach and backdoor attack targeting generic and rag-based llm agents by poisoning their long-term memory or rag knowledge base, proctag: a data-oriented ai method that assesses the efficacy of document instruction data, meet zeropath: a github app that detects, verifies, and issues pull requests for security vulnerabilities in your code, merlinn: an open-source llm-powered-on-call copilot ai engineer that automatically listens to production incidents and resolves it for you, huggingface researchers introduce docmatix: a dataset for document visual question answering containing 2.4 million pictures..., agentpoison: a novel red teaming approach and backdoor attack targeting generic and rag-based llm..., meet zeropath: a github app that detects, verifies, and issues pull requests for security....

  • AI Magazine
  • Privacy & TC
  • Cookie Policy
  • 🐝 Partnership and Promotion

🐝 FREE AI WEBINAR: A Synthetic Data Deep Dive (July 30 2024)

Thank You 🙌

Privacy Overview

  • ENVIRONMENT

How do you solve a problem like glitter?

Glitter is made to be tiny and adhere loosely to surfaces, contributing to microplastic pollution. Biodegradable solutions might still get everywhere, but won't stick around forever.

two purple gloved hands holding a luminescent strip

It’s hard to resist a little glitz. For centuries, people have coveted and adorned themselves with precious stones, metals, and anything else that catches the light. Glitter, a collection of tiny, reflective particles originally from natural sources like mineral dust has long been an easy way to add some dazzle.  

As early as A.D. 400, for example, the Maya people used mica in paint which shimmered in the sun. Other ancients in the Americas used the same mineral to adorn gravesites and craft figurines.

Modern technology, particularly the development of synthetic plastics, has given us endless and cheap supplies of glitter. Plastic-based glitter can be seen twinkling in everything from cosmetics and candles to wrapping paper and sunscreen.

But those sparkling flecks don’t just disappear when they slough off your face or birthday card. In fact, a growing body of research is pointing to serious environmental consequences of microplastic pollution.  

“One of the bad things is that it’s intentionally made to be small…and also very mobile,” says Robert Hale, an environmental scientist and professor at the Virginia Institute of Marine Science. Reports have found microplastics everywhere from the Antarctic to the deep ocean to, more recently, the placenta, feces, and blood vessels of humans.  

With glitter on the chopping block, experts discuss just how much damage a bit of plastic glitz can do and whether non-plastic alternatives offer viable solutions.  

Summer Flash Sale – Up to $20 off!

Glitter in our ecosystem affects all life, including ours.

Measuring less than five millimeters in length, most glitter is considered a microplastic and typically consists of three layers: a plastic core, a reflective coating, and a final thin, plastic layer. Like other microplastics—largely byproduct from the breakdown of bigger manufactured plastic—glitter has raised alarm because of how easily it can spread.

“It's oftentimes applied to a surface somewhat weakly…so it by nature falls off and is easily distributed,” Hale says. Over time, glitter particles fragment further making them smaller and more easily ingested. “When it gets small enough, it actually can enter cells…and participate or interfere with various essential biochemical reactions.”

( Microplastics are hidden in your home. Here’s how to avoid them .)

Microplastics are a known environmental hazard that are rampant in marine and terrestrial environments and, because of their size, nearly impossible to remove. High concentrations of glitter microplastics appear to hinder the growth of aquatic organisms , like phytoplankton and zooplankton, which form the base of the food chain and play a critical role in oxygen production and carbon dioxide consumption.

“This microplastic is so small, at the nanoscale, that [zooplankton] could eat this up and actually tear up their internal organs,” says study author Rafael Barty Dextro, a research biologist at the Center for Nuclear Energy and Agriculture in Brazil.  

microscopic image of a water droplet with living organisms and plastic

Small amounts of glitter can add up quickly, accumulating in cells and larger animals higher up the food chain. Microplastics have become ubiquitous and highly concentrated in bodies of water , particularly marine sediments , where they get into the bodies of small fish and vertebrates, which are then consumed by larger species.

In a study on viral mortality in fish , Hale found that microplastics amplified the effects of the virus, likely by causing physical damage to the gills and allowing the pathogen to enter more easily. Testing has found microplastics in the gastrointestinal tracts and feces of terrestrial animals, including snakes, birds, and even livestock.  

You May Also Like

answer in problem solving

In a first, microplastic particles have been linked to heart disease

answer in problem solving

What BPA can do to our bodies—and how to limit your exposure

answer in problem solving

Microplastics are in our bodies. How much do they harm us?

( Microplastics have moved into virtually every crevice on Earth .)

It’s not just the plastic itself that can leach into and contaminate the environment—glitter is often made with all sorts of toxic, chemical additives for color and shine. “So all of this, once in contact with water and air, starts degrading and liberating all of those compounds into the water,” Dextro says.  

Though the worst impacts have been found in marine life and ecosystem, humans too are at risk, particularly from cosmetic glitters.

Hale warns that directly applying glitter to skin can result in absorption or inhaling of tiny plastic particles. Humans may ingest between 74,000 and 121,000 microplastic particles per year, according to a 2019 estimate . Though the physiological impacts are still being studied, he says, these “foreign bodies” have been linked to a host of issues , including DNA damage, organ dysfunction, and cardiovascular problems . One recent study found a correlation between high levels of microplastics in arterial plaque and heart and circulatory issues, while another study linked inflammatory bowel disease with higher concentrations of fecal microplastics.  

( Microplastics are in our bodies. How much do they harm us ?)

Even if the quantities of glitter are fairly small relative to other microplastic sources, Hale describes it as an unnecessary risk. “[Glitter is] one of these voluntary heavy exposures…and easily avoided.”

strip of gold glitter on black background

Solving the glitter problem

Growing fears about the dangers of microplastic build-up have driven some authorities to try to eliminate them outright.

In 2022, California considered, but did not pass, a bill that would have banned cosmetics and other products containing “intentionally added microplastics.” Glitter is prohibited within the annual confines of Burning Man’s Black Rock City , as well as dozens of British music festivals . So-called “glitter bans”—which prohibit the manufacture and sale of certain microplastic-containing products—are in effect in New Zealand and the European Union .

Others are trying to keep the makeup artists and festival-goers happy with alternative solutions, such as biodegradable glitter. Non-plastic options are starting to hit the market, offering glitters made from the likes of plant cellulose, cassava, and mica.

Bioglitter, the first manufacturer of non-plastic glitter, produces its eco-friendly glitz with cellulose from wood pulp, primarily eucalyptus. This extract is then made into a film, cut into hexagons, and coated with pigments, stabilizers, and, most importantly, shellac—a glossy secretion from the lac beetle that adds shine. In as little as four to six weeks, the glitter will degrade naturally in environments that contain microorganisms, which “consume the glitter and turn it into harmless substances: water, carbon dioxide and biomass,” says Lauren Jones, founder of Luminosity Glitter, a Bioglitter retailer.  

But even biodegradable glitter may have its own problems. In preliminary studies , researchers found that cellulose and mica glitter was more damaging to duckweed and phytoplankton growth than conventional glitter.   “Ecotoxicological research is needed to test for impacts of any new types of glitter so we don’t end up just creating a new problem,” says author Dannielle Green, ecologist and plastic pollution expert at Anglia Ruskin University.

Others have seen more promising results. Though analysis is still ongoing, Mauricio Junior Machado, an agricultural microbiologist at the University of São Paulo, says that initial observations have not found any acute effects of cassava- and mica-based biodegradable glitter on the cell growth of freshwater cyanobacteria.  

In the meantime, Green emphasizes the value of individual actions, namely: don’t scatter or wash glitter down the drain. “We can make the decision to avoid glitter altogether or to dispose of it responsibly.”

Related Topics

  • MICROPLASTICS
  • PLASTIC WASTE
  • WATER POLLUTION

answer in problem solving

Microplastics are hidden in your home. Here’s how to avoid them.

answer in problem solving

California’s sweeping new plastics law could be a game changer

answer in problem solving

How a dramatic win in plastic waste case may curb ocean pollution

answer in problem solving

The world’s nations agree to fix the plastic waste crisis

answer in problem solving

We got rid of BPA in some products—but are the substitutes any safer?

  • Environment
  • History & Culture

History & Culture

  • Mind, Body, Wonder
  • Paid Content
  • Terms of Use
  • Privacy Policy
  • Your US State Privacy Rights
  • Children's Online Privacy Policy
  • Interest-Based Ads
  • About Nielsen Measurement
  • Do Not Sell or Share My Personal Information
  • Nat Geo Home
  • Attend a Live Event
  • Book a Trip
  • Inspire Your Kids
  • Shop Nat Geo
  • Visit the D.C. Museum
  • Learn About Our Impact
  • Support Our Mission
  • Advertise With Us
  • Customer Service
  • Renew Subscription
  • Manage Your Subscription
  • Work at Nat Geo
  • Sign Up for Our Newsletters
  • Contribute to Protect the Planet

Copyright © 1996-2015 National Geographic Society Copyright © 2015-2024 National Geographic Partners, LLC. All rights reserved

COMMENTS

  1. 26 Expert-Backed Problem Solving Examples

    The example interview responses are structured using the STAR method and are categorized into the top 5 key problem-solving skills recruiters look for in a candidate. 1. Analytical Thinking. Situation: In my previous role as a data analyst, our team encountered a significant drop in website traffic.

  2. 50 Interview Questions About Problem Solving (With Answers)

    Demonstrating your ability to tackle challenges effectively can set you apart from other applicants. Here are five tips to help you showcase your problem-solving skills during an interview: 1. Use the STAR Method. Structure your responses using the Situation, Task, Action, and Result (STAR) method.

  3. 8 Common Problem-Solving Interview Questions and Answers

    Related: 125 Common Interview Questions and Answers (With Tips) Problem-solving questions with sample answers Let's take a look at a few of the most common problem-solving interview questions that you may encounter during an interview and some sample answers. When preparing for your interview, consider a few different examples of when you ...

  4. Top 20 Problem Solving Interview Questions (Example Answers Included)

    MIKE'S TIP: When you're answering this question, quantify the details. This gives your answer critical context and scale, showcasing the degree of challenge and strength of the accomplishment. That way, your answer is powerful, compelling, and, above all, thorough. 2. Describe a time where you made a mistake.

  5. Problem-Solving Strategies: Definition and 5 Techniques to Try

    In insight problem-solving, the cognitive processes that help you solve a problem happen outside your conscious awareness. 4. Working backward. Working backward is a problem-solving approach often ...

  6. What is Problem Solving? (Steps, Techniques, Examples)

    The problem-solving process typically includes the following steps: Identify the issue: Recognize the problem that needs to be solved. Analyze the situation: Examine the issue in depth, gather all relevant information, and consider any limitations or constraints that may be present. Generate potential solutions: Brainstorm a list of possible ...

  7. 40 problem-solving techniques and processes

    7. Solution evaluation. 1. Problem identification. The first stage of any problem solving process is to identify the problem (s) you need to solve. This often looks like using group discussions and activities to help a group surface and effectively articulate the challenges they're facing and wish to resolve.

  8. Top 20 Problem Solving Interview Questions & Answers

    8. Detail how you've handled a scenario where team members disagreed on the solution to a problem. Navigating team disagreements can reflect deeper dynamics such as power struggles, communication breakdowns, or diverse perspectives. This question helps understand how a candidate resolves conflicts and builds consensus.

  9. The Problem-Solving Process

    Problem-solving is a mental process that involves discovering, analyzing, and solving problems. The ultimate goal of problem-solving is to overcome obstacles and find a solution that best resolves the issue. The best strategy for solving a problem depends largely on the unique situation. In some cases, people are better off learning everything ...

  10. Top 20 Problem Solving Skills Interview Questions & Answers

    Common Problem Solving Skills Interview Questions. 1. Describe a situation where you had to solve a problem with no obvious solution. Having strong problem-solving skills is crucial in many job roles, particularly in positions where unexpected challenges are common.

  11. Top Answers for Problem Solving Interview Questions

    Brainstorm and Analyze Options. Implementation. Emphasize Soft Skills. Top 8 Problem Solving Interview Questions and Example Answers. 1. Describe a Situation When You Faced a Difficult Problem at Work and How You Solved It. 2. Explain a Time When You Had to Make a Decision Without All the Information You Needed. 3.

  12. 10 Problem-solving strategies to turn challenges on their head

    2. Break the problem down. Identifying the problem allows you to see which steps need to be taken to solve it. First, break the problem down into achievable blocks. Then, use strategic planning to set a time frame in which to solve the problem and establish a timeline for the completion of each stage. 3.

  13. 5 Steps (And 4 Techniques) for Effective Problem Solving

    4. Implement the Solution. At this stage of problem solving, be prepared for feedback, and plan for this. When you roll out the solution, request feedback on the success of the change made. 5. Review, Iterate, and Improve. Making a change shouldn't be a one time action.

  14. What Are Problem-Solving Skills? (Examples Included)

    Before we dig into any examples, let's focus first on an important question: what are problem-solving skills. To answer that question, let's start with the barebones basics. According to Merriam-Webster, problem-solving is "the process or act of finding a solution to a problem. ...

  15. Problem-Solving Strategies and Obstacles

    Problem-solving is a vital skill for coping with various challenges in life. This webpage explains the different strategies and obstacles that can affect how you solve problems, and offers tips on how to improve your problem-solving skills. Learn how to identify, analyze, and overcome problems with Verywell Mind.

  16. Problem-Solving Interview Questions & Answers

    A job interview is a great moment for interviewers to evaluate how candidates approach challenging work situations.They do this by asking problem-solving questions. These types of questions are commonly asked during interviews since problem-solving skills are essential in most jobs. In any workplace, there are challenges, and when hiring new personnel, hiring managers look for candidates who ...

  17. What is Problem Solving? Steps, Process & Techniques

    Finding a suitable solution for issues can be accomplished by following the basic four-step problem-solving process and methodology outlined below. Step. Characteristics. 1. Define the problem. Differentiate fact from opinion. Specify underlying causes. Consult each faction involved for information.

  18. 25 Problem-Solving Interview Questions And Sample Answers

    Problem-Solving Exercises . Some interviewers also like to throw in a couple of weird interview questions, aimed at challenging your on-the-stop problem-solving skills.For example, Jeff Bezos once asked an interviewee to try counting the number of windows in Seattle. While the question may sound absurd, it gives the interviewer a good idea of how you structure your reasoning and employ logical ...

  19. 25 Problem Solving Interview Questions & Answers

    Answer Example "I rate my problem-solving skills as an 8/10. I will, on occasion, have times when I am not as efficient as I would like to be but all in all, I do feel that my problem-solving skills are above average. My supervisor and co-workers will attest to my fast reflexes when a problem arises, and they would also say that I remain calm ...

  20. How to Develop Problem Solving Skills: 4 Tips

    How to Develop Problem Solving Skills: 4 Tips. Learning problem-solving techniques is a must for working professionals in any field. No matter your title or job description, the ability to find the root cause of a difficult problem and formulate viable solutions is a skill that employers value. Learning the soft skills and critical thinking ...

  21. 10 Proven Problem-solving Interview Questions [+Answers]

    Problem-solving interview questions show how candidates: Approach complex issues. Analyze data to understand the root of the problem. Perform under stressful and unexpected situations. React when their beliefs are challenged. Identify candidates who are results-oriented with interview questions that assess problem-solving skills. Look for ...

  22. What Are Problem-Solving Skills? Definition and Examples

    Problem-Solving Skills Definition. Problem-solving skills are the ability to identify problems, brainstorm and analyze answers, and implement the best solutions. An employee with good problem-solving skills is both a self-starter and a collaborative teammate; they are proactive in understanding the root of a problem and work with others to ...

  23. Mastering Problem-Solving: Essential Steps and Skills

    Problem-solving skills are essential in both personal and work situations. They help you handle challenges well and get the results you want. Good problem-solving improves decision-making and resilience in handling conflicts and helps businesses succeed. Continue reading to learn the essential steps and skills required for efficient problem ...

  24. Computational Thinking Is A Key Problem-Solving Skill In The ...

    Similar to general problem-solving, computational thinking involves breaking down problems, pulling out key information and forming solutions. ... It helps solve problems by finding patterns ...

  25. How Good Is ChatGPT at Coding, Really?

    Overall, ChatGPT was fairly good at solving problems in the different coding languages—but especially when attempting to solve coding problems that existed on LeetCode before 2021.

  26. Math Message Boards FAQ & Community Help

    Art of Problem Solving AoPS Online. Math texts, online classes, and more for students in grades 5-12. Visit AoPS Online ‚ Books for Grades 5-12 ...

  27. What happened at Possum Trot? Remarkable story shows how we can solve

    Despite this massive investment, foster care still looks like an unsolvable problem. But this problem is solvable. The town of Possum Trot showed how. Strong communities hold key to solving big ...

  28. Which Of The Following Is A Heuristic Commonly Used In Problem Solving

    Means-end analysis is a method of problem-solving that is frequently employed.. What is meant by heuristic problem solving? A problem-solving heuristic is an informal, speculative method that occasionally results in a solution but not always. The consequences of using a heuristic are unpredictable, therefore the technique can either be more or less successful than using an algorithm.

  29. This AI Paper from UC Berkeley Shows How Interfacing GPT with Prolog

    This problem likely comes from the way transformers, the underlying architecture, work. They solve problems step-by-step, predicting the next word in a sequence, which limits their ability to backtrack and fix errors. Moreover, LLMs are trained statistically, which creates challenges in handling problems outside their training distribution.

  30. How do you solve a problem like glitter?

    How do you solve a problem like glitter? Glitter is made to be tiny and adhere loosely to surfaces, contributing to microplastic pollution. Biodegradable solutions might still get everywhere, but ...